Sei sulla pagina 1di 100

Universita di Pisa

Corso di laurea in Fisica

Esercitazioni di Elettromagnetismo
www.df.unipi.it/astrumia/fisica2.html
Ultimo aggiornamento: 4 maggio 2005 (capitolo 14)

2 Conduttori 21
Es 31 Piano conduttore . . . . . . . . . . . 21
Es 32 Lastra conduttrice . . . . . . . . . . . 21
Es 33 Metodo delle immagini . . . . . . . . 22
Es 34 Piano carico fra 2 piani conduttori . . 22
Es 35 Carica fra 2 piani conduttori . . . . . 23
Indice Es 36 1 lastre conduttrice carica . . . . . . 23
Es 37 2 lastre conduttrici cariche . . . . . . 23
Es 38 Capacitatore cilindrico . . . . . . . . 23
Es 39 Capacitatore di dimensioni variabili . 24
I Elettrostatica 4 Es 40 Conduttore in capacitatore . . . . . . 25
Es 41 Sfera conduttrice a terra . . . . . . . 25
1 Campi e potenziali elettrici 5
Es 42 Sfera conduttrice isolata . . . . . . . 26
Es 1 Gravita vs elettromagnetismo . . . . 5
Es 43 Sfera conduttrice in E costante . . . 26
Es 2 Rompere una bacchetta . . . . . . . . 5
Es 44 Tetraedro conduttore . . . . . . . . . 27
Es 3 Sistemi stabili? . . . . . . . . . . . . 5
Es 45 Condensatore sferico . . . . . . . . . 27
Es 4 Campo elettrico di un filo . . . . . . 7
Es 46 Condensatori in serie . . . . . . . . . 27
Es 5 Campo elettrico di un piano . . . . . 7
Es 47 Effetto delle punte . . . . . . . . . . . 27
Es 6 Campo elettrico di una sfera . . . . . 8
Es 7 Energia di un guscio sferico . . . . . . 8 Es 48 Sfera conduttrice bucata . . . . . . . 28
Es 8 Modelli dellatomo . . . . . . . . . . . 9 Es 49 Carica dentro sfera . . . . . . . . . . 28
Es 9 Energia di una sfera . . . . . . . . . . 9
3 Dielettrici 29
Es 10 Raggio classico dellelettrone . . . . . 10
Es 50 2 dielettrici in condensatore piano . . 29
Es 11 Masse dei nuclei . . . . . . . . . . . . 11
Es 51 N dielettrici in condensatore piano . 29
Es 12 Differenza di massa protone-neutrone 11
Es 13 Nucleo che si spezza . . . . . . . . . . 12 Es 52 Condensatore in acqua . . . . . . . . 30
Es 14 Energia di due cariche . . . . . . . . 12 Es 53 Carica davanti a semipiano dielettrico 30
Es 15 Forza su cariche superficiali . . . . . 12 Es 54 Dielettrico in condensatore . . . . . . 31
Es 16 Scattering debole . . . . . . . . . . . 13 Es 55 Forza di conduttore su dielettrico . . 31
Es 17 Scattering Rutherford . . . . . . . . . 13 Es 56 Dielettrico in campo esterno . . . . . 32
Es 18 Esplosione Coulombiana . . . . . . . 14 Es 57 Buco in dielettrico . . . . . . . . . . . 32
Es 19 Sfera polarizzata . . . . . . . . . . . . 15 Es 58 Sfera dielettrica in dielettrico . . . . . 33
Es 20 Cilindro polarizzato . . . . . . . . . . 16 Es 59 Attrazione fra dielettrici . . . . . . . 33
Es 21 Formule di base sui dipoli . . . . . . 16
Es 22 Paradosso sui dipoli I . . . . . . . . . 16 4 Correnti 35
Es 23 Paradosso sui dipoli II . . . . . . . . 16 Es 60 Capacitatore piano imperfetto . . . . 35
Es 24 Paradosso sui dipoli III . . . . . . . . 17 Es 61 Scarica di sfera carica . . . . . . . . . 35
Es 25 Paradosso sui dipoli IV . . . . . . . . 17 Es 62 Resistenza fra sfere concentriche . . . 35
Es 26 Allineamento di dipoli elettrici . . . . 17 Es 63 Sonda marina . . . . . . . . . . . . . 36
Es 27 Coordinate polari . . . . . . . . . . . 18 Es 64 Semipiano dielettrico imperfetto . . . 36
Es 28 Laplaciano . . . . . . . . . . . . . . . 19 Es 65 Diodo termoionico . . . . . . . . . . . 36
Es 29 Potenziale di Yukawa . . . . . . . . . 19 Es 66 Piatto dielettrico . . . . . . . . . . . 37
Es 30 Atomo di idrogeno quantistico . . . . 20 Es 67 Sfera dielettrica . . . . . . . . . . . . 37

1
2 Indice

5 Circuiti 39 9 Forze magnetiche fra circuiti 63


Es 68 Resistenze in parallelo . . . . . . . . 39 Es 116 Due circuiti lunghi . . . . . . . . . . 63
Es 69 Resistenze su cubo . . . . . . . . . . 39 Es 117 Rotazione di due spire circolari . . . 63
Es 70 Ponte di Wheatstone . . . . . . . . . 39 Es 118 Una spira ed un dipolo . . . . . . . . 64
Es 71 Impedenze . . . . . . . . . . . . . . . 40 Es 119 Monopolo magnetico . . . . . . . . . 65
Es 72 Potenza dissipata . . . . . . . . . . . 40 Es 120 Traslazione di due spire circolari . . . 66
Es 73 Filtro che taglia frequenze alte . . . . 40
Es 74 Filtro che taglia frequenze basse . . . 41 10 Campi magnetici nella materia 69
Es 75 Pendolo accoppiato . . . . . . . . . . 41 Es 121 Sbarra magnetizzata . . . . . . . . . 69
Es 76 Attenuatore . . . . . . . . . . . . . . 41 Es 122 Materiali ferromagnetici . . . . . . . 69
Es 77 Catena LC . . . . . . . . . . . . . . . 42 Es 123 Due bacchette . . . . . . . . . . . . . 70
Es 124 Correnti parassite . . . . . . . . . . . 70
Es 125 Correnti parassite . . . . . . . . . . . 70
II Magnetostatica 44 Es 126 In pratica . . . . . . . . . . . . . . . 71
6 Campi magnetici 45
Es 78 Forza fra 2 cariche . . . . . . . . . . . 45 III Elettrodinamica 72
Es 79 Disco di Rowland . . . . . . . . . . . 45
Es 80 Attrazione o repulsione? . . . . . . . 46 11 Corrente di spostamento 73
Es 81 Filo rettilineo . . . . . . . . . . . . . 46 Es 127 Scarica di un filo . . . . . . . . . . . . 73
Es 82 Cavo coassiale . . . . . . . . . . . . . 46 Es 128 Piano con carica ondulata . . . . . . 73
Es 83 Spira circolare . . . . . . . . . . . . . 46 Es 129 Sfera radioattiva . . . . . . . . . . . . 74
Es 84 Due spire circolari . . . . . . . . . . . 47 Es 130 Scarica di un condensatore . . . . . . 74
Es 85 Filo a U . . . . . . . . . . . . . . . . 47 Es 131 Condensatore in alternata . . . . . . 75
Es 86 Piano a U . . . . . . . . . . . . . . . 47 Es 132 Cavita risuonante . . . . . . . . . . . 76
Es 87 Solenoide rettilineo infinito . . . . . . 47 Es 133 Effetto pelle . . . . . . . . . . . . . . 76
Es 88 Solenoide rettilineo finito . . . . . . . 48 Es 134 Filo conduttore interrotto . . . . . . 77
Es 89 Solenoide toroidale . . . . . . . . . . 48 Es 135 Due cilindri cavi . . . . . . . . . . . . 78
Es 90 Sfera ruotante . . . . . . . . . . . . . 48 Es 136 Carica in moto . . . . . . . . . . . . . 79
7 Moto in campo magnetico esterno 50
12 Onde e oscillazioni 80
Es 91 Trottola magnetica . . . . . . . . . . 50
Es 137 Sorgenti di onde . . . . . . . . . . . . 80
Es 92 Cilindro su piano inclinato . . . . . . 50
Es 138 Ricevitore di onde . . . . . . . . . . . 80
Es 93 Ago magnetico . . . . . . . . . . . . . 51
Es 139 Luce solare . . . . . . . . . . . . . . . 81
Es 94 Carica in B costante . . . . . . . . . 51
Es 140 Vettore di Poynting . . . . . . . . . . 81
Es 95 Campo magnetico galattico . . . . . . 51
Es 141 Rilfessione di onde in una corda . . . 82
Es 96 Spettrometro . . . . . . . . . . . . . . 52
Es 142 Rifrazione . . . . . . . . . . . . . . . 83
Es 97 Carica in B ed E costanti . . . . . . 52
Es 98 Fotomoltiplicatore in B, E . . . . . . 52 Es 143 Rifrazione . . . . . . . . . . . . . . . 83
Es 99 Ciclotrone . . . . . . . . . . . . . . . 53 Es 144 Riflessione da un metallo . . . . . . . 84
Es 100 Carica in B con direzione non uniforme 54 Es 145 Onde adiabatiche . . . . . . . . . . . 86
Es 101 Carica in B con modulo non uniforme 54 Es 146 Guida donda . . . . . . . . . . . . . 86
Es 102 Carica in B(t) uniforme . . . . . . . . 55 Es 147 Cavita risuonante . . . . . . . . . . . 87
Es 103 Atomo in B(t) uniforme . . . . . . . 56 Es 148 Pressione di radiazione . . . . . . . . 87
Es 104 Carica in B non uniforme . . . . . . . 56 Es 149 Velocita di gruppo . . . . . . . . . . . 88
Es 105 Carica in quadrupolo magnetico . . . 56 Es 150 Pulsar . . . . . . . . . . . . . . . . . 88
Es 106 Carica in quadrupoli magnetici . . . . 57 Es 151 Interferenza . . . . . . . . . . . . . . 88
Es 107 Ottica geometrica matriciale . . . . . 58
13 Irraggiamento 90
8 Induzione magnetica 59 Es 152 Atomo di idrogeno . . . . . . . . . . . 90
Es 108 Circuito allungato . . . . . . . . . . . 59 Es 153 Scattering elettrone/nucleo . . . . . . 90
Es 109 Cilindro ruotante . . . . . . . . . . . 59 Es 154 Scattering ee . . . . . . . . . . . . . . 91
Es 110 Generatore . . . . . . . . . . . . . . . 59 Es 155 Onde gravitazionali . . . . . . . . . . 91
Es 111 Generatore in orbita . . . . . . . . . 60 Es 156 Scattering elettrone/fotone . . . . . . 92
Es 112 Trasformatore . . . . . . . . . . . . . 60 Es 157 Polarizzazione della CMB . . . . . . . 92
Es 113 Trasformatore con due spire . . . . . 60 Es 158 Unantenna . . . . . . . . . . . . . . . 92
Es 114 Induzione . . . . . . . . . . . . . . . . 61 Es 159 Due antenne . . . . . . . . . . . . . . 92
Es 115 Trapano . . . . . . . . . . . . . . . . 61 Es 160 Dipolo magnetico . . . . . . . . . . . 93
Indice 3

14 Relativita 94 Es 169 Riflessione da specchio in moto . . . 97


Es 161 Contrazione di Lorentz . . . . . . . . 94 Es 170 Aberrazione relativistica . . . . . . . 97
Es 162 Che cosa e lelettromagnetismo . . . 94 Es 171 0 2 . . . . . . . . . . . . . . . . 97
Es 163 Forza fra 2 cariche bis . . . . . . . . . 95 Es 172 GZK . . . . . . . . . . . . . . . . . . 98
Es 164 Scattering debole bis . . . . . . . . . 95
Es 173 Effetto Compton . . . . . . . . . . . . 99
Es 165 Carica in E e B ortogonali bis . . . . 96
Es 166 Filo in moto . . . . . . . . . . . . . . 96 Es 174 Esperienza d Fizeau . . . . . . . . . . 99
Es 167 Forza prodotta da filo in moto . . . . 96 Es 175 Iraggiamento da elettroni relativistici 99
Es 168 Onda vista da sistema in moto . . . . 97 Es 176 Miraggi . . . . . . . . . . . . . . . . . 100
Parte I

Elettrostatica
Capitolo 1

Campi e potenziali elettrici

Forza di Coulomb: F = krq1 q2 /r2 dove k = 8.9875 109 N m2 /C 2 , riscritto in termini della costante dielettrica
del vuoto 0 come k = 1/40 con 0 = 8.8542 1012 C 2 / N m2 . Altre unita usate altrove sono k = 1, k = 1/4.
E utile introdurre il campo elettrico E, ed il potenziale elettrico . La forza di Coulomb F rp ha importanti
proprieta speciali vere solo per p = 2, che rendono possibile reinterpretarla come teorema di Gauss (E) =
Qin /0 e poi come   2
E = /0 = /0

E = 0 E =
(Analogamente a come x = a e equivalente ma piu utile di x = 12 at2 ). Energia elettromagnetica e sua densita
X qi qj Z Z Z
1 X qi qj 1 (x1 )(x2 ) 1 0
U= = = dV1 dV2 = dV = dV E 2
i>j
4 r
0 ij 2 4 r
0 ij 2 4 |x
0 1 x 2 | 2 2
i6=j

Un Coulomb sono 6.24 1018 elettroni. Una unita di misura molto usata e lelettron-volt eV = J(qe /C) =
J/6.24 1018 , che e lenergia che un singolo elettrone acquista passando per una differenza di potenziale di un
Volt.

Esercizio 1: Gravita vs elettromagnetismo


Un atomo di idrogeno e composto da un elettrone e da un protone (con massa me = 0.911 1030 kg e mp
1836me ) a distanza circa A = 108 cm. Calcolare la forza elettrica e gravitazionale.
2
bSoluzione: La forza elettrica ha un valore quasi macroscopico FC ke2 /A 108 N. La forza gravitazionale
e invece trascurabile:
FN GN me mp
= = 4.4 1040 N
FC ke2
Sebbene esistano motivi plasusibili non si sa da dove un rapporto cos grosso esca fuori.
Le particelle si combinano formando materia neutra in modo da cancellare, in media, lenorme forza elettrica.
Lenergia di legame vale
me 2 e2 k e2
E= ve k = = 13.6 eV
2 r 2 r
per r = 0.53 A. Lelettrone ha velocita v = c/137.036 e quindi e non relativistico.

Esercizio 2: Rompere una bacchetta


Una bacchetta ha sezione di 1 cm2 . Che forza bisogna avere per romperla?

bSoluzione: Se la materia e costituita da atomi di dimensione A legati dalla forza di Coulomb, uno deve
rompere n = ( cm/A)2 = 1016 legami e quindi serve una forza nFC 108 N. Tenendo conto che non distinguiamo
idrogendo da gesso da acciaio, la stima non e male; avrebbe potuto venire una cosa sbagliata di 100 ordini di
grandezza. Il legame vero e piu debole; non occorre ionizzare gli atomi.

5
6 Capitolo 1. Campi e potenziali elettrici

Esercizio 3: Sistemi stabili?


Assemblare un sistema di cariche elettriche in equilibrio stabile.

bSoluzione: Presentiamo tre tentativi fallimentari: la cosa interessante e capire perche non funzionano.

2) Usando solo due cariche non e possibile assemblare un sistema stabile.

3) Proviamo con tre cariche: due cariche q lungo lasse x

P1 = `(1, 0, 0), P2 = `(1, 0, 0)

ed una q 0 in mezzo a P3 = (0, 0, 0) dove E = 0. Scegliendo q 0 = q/4 si ha E = 0 anche sulle cariche q.


Quindi abbiamo realizzato un sistema in equilibrio, e rimane da vedere se si tratta di equilibrio stabile o
instabile. E facile vedere che q 0 e in equilibrio instabile: il campo elettrico per X = (x, y, z) 0 e
2 ~ P~i
~ q X X q
E(x, y, z) = ' (4x, 2y, 2z) + O(x, y, z)2 .
~ P~i |
40 i=1 |X 3/2 40 L3

Come intuitivamente atteso lequilibrio e instabile lungo x e stabile lungo y e z. E facile verificare che il
~ calcolato su di un cubetto attorno a 0 vale zero: 4 + 2 + 2 = 0.
flusso di E
Attorno alla carica 2 si ha
~ q xL y z
E(x, y, z) ' ( , , )
40 L3 4 8 8
~ calcolato su di un
che e stabile lungo z ed y ma instabile lungo x. E facile verificare che il flusso di E
cubetto attorno a P2 vale zero: 41 18 18 = 0.

4) Con quattro cariche: tre q ai vertici di un triangolo equilatero di lato `:

1 1 1 1 1
P1 = `(0, , 0), P2 = `( , , 0), P3 = `( , , 0),
3 2 2 3 2 2 3
2 2

Ciascuna
risente una forza F = 2(kq /` )( 3/2) diretta verso lesterno. In mezzo al triangolo (a distanza

d = `/ 3 dalle altre) il campo elettrico vale zero, quindi provo ad aggiungere una carica q 0 = q/ 3 in
modo che le altre cariche risentano forza zero. Lequilibrio einstabile. Ad esempio il campo elettrico per
X = (x, y, z) 0 e

q X X ~ P~i q 9 3 x y
~ =
E ' ( , , z) + O(x, y, z)2
~ P~i |3/2
40 i |X 40 L3 2 2

che e instabile nel piano (x, y) e stabile lungo la direzione z. In pratica uno puo calcolare le componenti
meno laboriose Ez ed Ey , ed ottenere Ex sapendo che il flusso vale zero: infatti si ha 1/2 1/2 + 1 = 0.
Il campo elettrico attorno alla carica 1 e

q 5 7
~ '
E 3
( x, (y P1y ), z)
40 L 2 2

che e stabile lungo z e lungo x ma non lungo y. Di nuovo il flusso su di un cubetto attorno alla carica 1
vale zero.
Capitolo 1. Campi e potenziali elettrici 7

E importante notare che per altri potenziali V 1/rp con p 6= 1 sarebbe possibile costruire sistemi stabili.
Nel caso speciale di V 1/r il problema non ha soluzione, in quanto il fenomeno che abbiamo verificato e del
tutto generale: il flusso del campo elettrico generato dalle cariche esterne ad una superficie e zero (teorema di
Gauss). Lo si puo verificare in generale espandendo il campo generato da una singola carica q il campo di
tante cariche e la sovrapposizione dei campi delle singole cariche, ciascuna delle quali da flusso zero. Mettendo
la carica q in (0, 0, 0), nella regione attorno a X = (r, 0, 0) + (x, y, z) si ha

q X~ q x y z
~ =
E ' (1 + (1 p) , , ) + O(x2 , y 2 , z 2 )
~
40 |X| p/2 40 r p1 r r r

che ha flusso zero solo per p = 2. Quindi il campo elettrico non puo essere solo entrante o solo uscente, come
sarebbe necessario per avere una forza attrattiva in qualunque direzione.
Questo fallimento ha una conseguenze fisica importante: lelettromagnetismo da solo non puo spiegare la
stabilita della materia.

Esercizio 4: Campo elettrico di un filo


Si calcoli il campo e potenziale di un filo rettilineo infinito con densita lineare di carica .

bSoluzione: A distanza d dal filo


Z +
kr 2k
Er = dz = , E = 0
(r2 + z 2 )3/2 r

Il potenziale elettrico consente di calcolare facilmente E = di un filo di lunghezza finita 2`


+`z  +`z `2 r2 2z 2
Z  
0 k 0
p
02
kQ ` 4
(z, r) = dz 2
= k ln z + z + r ' 1+0 + O(` )
`z r2 + z02 `z r2 + z 2 d 6 (r2 + z 2 )2

dove Q = 2`.
k k
Ez = z = p p , Er = r = ...
r2 + (z `)2 r2 + (z + `)2
Espandendo in serie di Taylor per `  r, z, il primo termine corrisponde alla carica totale. Il termine successivo
e di quadrupolo (il dipolo vale zero).
Per ` = viene = . Avendo cariche allinfinito non e possibile mettere () = 0. Linfinito e tutto
nella costante addittiva arbitraria in : fissando (r0 ) = 0 viene
Z +  
1 1 r
(r) = k dz p = 2k ln = 2k ln r + costante

2
r +z 2 2
r0 + z 2 r 0

Se il mondo avesse 2 dimensioni invece di 3, questi sarebbero il campo ed il potenziale di Coulomb. Ver-
ificherebbero ancora 2 = 0. Siccome cresce per r , in 2 dimensioni non esisterebbero cariche
libere.

Esercizio 5: Campo elettrico di un piano


Calcolare il campo elettrico di un piano infinito con densita superficiale di carica

bSoluzione:

1) Calcolo diretto. Per motivi di simmetria E ha solo la componente Er ortogonale al piano. Passiamo
attraverso il campo generato da un disco circolare di raggio R.

Z R
1 r r R
Er (r) = 2 d = (1 ) = sgn r
40 0 (r2 + 2 )3/2 20 r 2 + R2 20
8 Capitolo 1. Campi e potenziali elettrici

Per r  R in termini della carica totale Q = R2 viene il risultato ovvio (espansione di multipolo)

3R2
 
Q 4
Er ' 1 + O(R )
40 r2 4r2

Per R  r si ottiene il piano cairico: Er (r) = /20 . Per motivi dimensionali E r.


2) Teorema di Gauss: flusso = carica interna/0 . Prendendo un cilindretto schiacciato che attraversa una
qualunque superficie con densita di carica variabile
1 2
= S(E E ) = /0 in qualunque punto

Nel caso del piano, aggiungendo considerazioni di simmetria, si riottiene il risultato precedente.
3) Il potenziale lungo lasse di un disco uniformemente carico di raggio R vale

p 2 Q R2
(r, 0) = ( R + r2 r2 ) ' (1 2 + O(R4 ))
20 40 r 4r
Per R  r Z

(r) = p p = (r0 r)
20 0 2 + r 2 2 + r02 20

Esercizio 6: Campo elettrico di una sfera


Calcolare il campo elettrico generato da una densita superficiale di carica = Q/4R2 distribuita su di un
guscio sferico di raggio R.

bSoluzione: Il teorema di Gauss darebbe immediatamente il risultato, ma qui lo vogliamo ottenere tramite
un calcolo a testa bassa. Mettiamo la sfera nellorigine e calcoliamo E in (r, 0, 0). Per motivi di simmetria E
ha solo la componente radiale:
Z 
r R cos 0 r<R
Ex = R d 2R sin =
40 0 | {z } [(r R cos )2 + R2 sin2 ]3/2 Q/40 r2 r > R
dS

Esercizio 7: Energia di un guscio sferico


Si calcoli il lavoro necessario per comprimere un palloncino sferico contenente una carica Q uniformemente
distribuita da un raggio r1 ad r2 .

bSoluzione: A raggio r generico il campo elettrico e radiale

Q/40 r2 fuori

Er (r) =
0 dentro

Le cariche vorrebbero espandersi: in generale una densita di superficie risente una forza E medio dove E medio =
(E 1 + E 2 )/2. In questo caso il campo interno e zero e = Q/4r2 . Per comprimere da r1 ad r2 occorre
esercitare un lavoro Z r2 Z r2
Er (r) Q2 dr Q2 1 1
L = 2
r (r) dr = 2
= ( )
r1 2 8 0 r1 r 8 0 r 1 r 2
2
E
Sostituendo == 0 Er e S dr = dV ottengo anche L = S E2r dr = 0 2r dV . In generale il campo elettrico
R R

contiene una densita di energia u = 0 E 2 /2. Le dimensioni sono giuste. In questo caso la verifica e semplice
perche mano a mano che si contrae varia solo il campo nella zona di contrazione
Z r2
E2 Q2 1
Z
1
U = u dV = 0 4r2 = ( )
r1 2 8 0 r 1 r 2


Capitolo 1. Campi e potenziali elettrici 9

Come ulteriore verifica calcoliamo anche lenergia totale per ri = R


Z Z
1 ? 0
U= dV = E 2 dV
2 2
Inserendo nella prima espressione = Q/(40 R) e S = Q, si ottiene subito
1 Q2
U= S =
2 80 R
Inserendo nella seconda E = Q/(4r2 0 ) viene lo stesso risultato:
0
Z
Q2 Q2
Z
2 2 dr
U= 4r E dr = = .
2 R 80 R r2 80 R

Esercizio 8: Modelli dellatomo


Nel modello di Thomson per latomo di idrogeno, la carica positiva e e distribuita uniformemente in una sfera
di raggio a0 . Lelettrone di carica e e considerato puntiforme e si muove allinterno della sfera.
a) Calcolare il campo elettrico ed il potenziale generati dalla carica positiva e la posizione dequilibrio per
lelettrone (assunto in uno stato di momento angolare nullo).
b) Determinare lenergia di ionizzazione UI (ovvero lenergia necessaria ad estrarre lelettrone dallatomo).
Trovare il valore di a0 consistente col valore sperimentale UI = 2.18 1018 Joule.
c) Determinare il periodo di oscillazione dellelettrone intorno alla posizione dequilibrio e confrontarlo col
valore sperimentale T = 3.04 1016 sec1 .
d) Si calcoli il momento di dipolo elettrico p indotto nellatomo da un campo esterno E 0 , la polarizzabilita
dellatomo e la costante dielettrica  dellidrogeno allo stato solido (cioe nello stato in cui tutti gli atomi
sono adiacenti fra loro a formare un reticolo).

bSoluzione: Lenergia di ionizzazione si puo anche riscrivere come 13.6 eV dove eV = e mN/C = 1.60 1019
Joule.
a) Il campo e radiale e si ha equilibrio stabile in r = 0:
( (  2 
e r
40 a30 r < a0 r
4e0 a0 2a 3
2 2 r < a0
Er (r) = e 1 (r) = e 1
0

40 r 2 r > a0 + 4 r > a0


0 r

b) UI = e(0) = 3e2 /80 a0


c) Dallequazione del moto
e e r
r = = 2 r
me 40 a30
si ha 2 = e2 /40 me a30 e quindi T = 2/ = 7.9 1016 s.
d) La nuova posizione dequilibrio req e data da E(req ) = E 0 da cui r eq = 40 a30 E 0 /e; p = er eq = E 0
dove = 40 a30 ; la densita di atomi e n = 1/(2a0 )3 ; quindi  = 1 + n/0 = 1 + /2.
Nel modello di Rutheford lenergia di ionizzazione vale
me v 2 1 e2
UI = (a0 ) = ( + 1)
2 2 40 a0
e la frequenza di rotazione
v2 F/me e2
2 = 2 = =
a0 a0 40 me a30
Modello 2 Energia di ionizzazione
Rutherford e /40 me a30
2
e2 /80 a0 = 8.5 eV
Thomson e2 /40 me a30 3e2 /80 a0 = 26 eV
Per fissare 2 serve a = 8.4 1011 m in entrambi i casi; dopodiche entrambi i modelli non azzeccano UI (la
frequenza e data dalla differenza di energia tra i livelli quantistici).
10 Capitolo 1. Campi e potenziali elettrici

Esercizio 9: Energia di una sfera


Calcolare lenergia potenziale di una sfera di raggio R contenente una carica Q distribuita uniformemente.

bSoluzione:
1) Integrando la densita di energia
Z R Z  2
Q2
Z  
0 2 0 2 Qr 2 2 Q 2 1 Q
U= E dV = 4r ( ) + 4r ( ) = +
2 2 0 40 R3 R 40 r2 40 10R 2R

2) Calcolo doppiamente sbagliato. Densita di carica: = Q/V = 3Q/4R3 . Carica dentro una sferetta di
raggio r < R: qin (r) = Q(r/R)3 , quindi = qin (r)/40 r (soddisfa () = 0 e continutita ad r = R)
Z R
1 3 Q2
Z
qin (r)
U = dV = 4r2 dr =
0 | {z } |4 R
{z0 }
40 5 R
dq

20 ) Calcolo giusto. Il potenziale dentro la sfera vale (r) = Er dr = cte 21 qin (r)/(40 r). Imponendo
R

continuita ad r = R
Q(3R2 r2 )/80 R2 per r < R

(r) =
Q/40 r per r > R
Integrando
R
3Q2 r2 (3R2 r2 ) 3 Q2
Z Z
1 1
U= dV = 6
=
2 40 0 4R 5 R
I plot delle funzioni Ee e giuste sono:

E

r r

3) Il calcolo sbagliato al punto 2) non da il risultato giusto per caso. Partendo dai principi primi, costruisco
la sfera aggiungendo mano a mano carica dq facendo crescere il suo raggio r da 0 a R a densita costante
. La usata in 2) non e il potenziale della sfera finale, ma lenergia necessaria per portare cariche da
a r nel modo appena descritto.
1 q(r)dq 4r3 r
dU = q(r) = Q( )3 dq = 4r2 dr
40 r 3 R
quindi
42 4 42 R5 1 3Q2
Z
dU = r dr U= dU = =
30 150 40 5R


Le stesse cose valgono per la gravita. I fisici credevano che la terra fosse piu giovane di 30 Myr, siccome
lenergia del sole E GM 2 /R emessa con potenza d2 K K = 8.53 1011 MeV cm2 s1 dura solo per
(GM 2 /R)/(d2 Ksun ) 30 Myr. I biologi (Darwin) sostenevano che almeno 300 Myr erano necessari per lerosione
e levoluzione delle specie. Teologi (like Lightfoot) sostenevano che la terra era stata creata il 23 ottobre 4004,
alle nove del mattino. I fisici successivamente scoprirono lenergia nucleare.
Capitolo 1. Campi e potenziali elettrici 11

Esercizio 10: Raggio classico dellelettrone


Approssimando lelettrone come una sferetta di raggio R, calcolare il valore di R tale che U = me c2 .

bSoluzione: Approssimare le particelle come puntiformi e un limite singolare. Ad esempio: (1) Quando si
calcola la forza su di un elettrone non si include nel campo elettrico quello infinito generato dallelettrone stesso.
(2) Lenergia elettromagnetica U diverge per R 0.
Nel secolo scorso ci sono stati tentativi di migliorare questa situazione proponendo teorie dellelettrone in
cui lelettrone veniva approssimato con una palletta di raggio R finito. Lenergia elettromagnetica U allora e
finita: U = cqe2 /40 R dove c dipende da quale distribuzione di carica viene assunta. c = 3/5 per una densita
uniforme; c = 1/2 per una densita superficiale uniforme. Assumiamo c = 1.
Lenergia elettromagnetica contribuisce alla massa dellelettrone, secondo m = U/c2 . Assumendo che tutta
la massa sia di origine elettromagnetica si determina
qe2
R = re = = 2.82 1015 m
me 0 c2
chiamato raggio classico dellelettrone, sebbene non abbia niente a che vedere con le dimensioni di un elettrone
(solo una piccola parte dellenergia dellelettrone e di origine elettromagnetica). Numericamente, re e simile alla
dimensione di un protone: si tratta di un accidente fuorviante.
Una aspettazione qualitativa piu corretta e U < me c2 (altrimenti U eccederebbe la massa dellelettrone):
questa implica R > re . Oggi si sa quello che succede: a scale R 1000re iniziano a farsi sentire gli effetti del
positrone, una particella identica allelettrone ma con carica positiva. Ripetendo il calcolo di U in teorie di
campo quantistiche relativistiche si trova che il contributo elettromagnetico alla massa dellelettrone esiste ma
e piccolo, circa 1/100 della massa totale.

Esercizio 11: Masse dei nuclei


Si puo approssimare un nucleo come una sfera a densita costante, contenente Z = A/2 protoni e circa A/2
neutroni con raggio R = A1/3 rN con rN = 1.2 1015 m. Calcolare lenergia elettromagnetica e discutere la
stabilita dei nuclei.

bSoluzione: Aggiungiamo allenergia eletromagnetica un termine che descrive lenergia di legame media fra
nuclei contigui e la differenza di massa mn mp 1
U 3 (Ze)2
mnucleo = Zmp + (A Z)mn + , U= + Elegame A.
c2 5 40 R
Esistono nuclei stabili fino a Z 100: imponendo dU/dZ|Z100 = 0 si trova lenergia di legame per nucleone:
2/3
e2

100
Elegame = 10 MeV
4 40 rN
A grandi Z la repulsione Coulombiana diventa leffetto principale ed impedisce di formare nuclei grossi. Mini-
mizzando U/Z si trova che il nucleo che ha la maggior energia di legame per nucleone ha Z 20, ed infatti e il
ferro (Z = 26).

Esercizio 12: Differenza di massa protone-neutrone


Stimare la differenza di massa protone-neutrone approssimandoli come 3 quarks fermi ai vertici di un triangolo
equilatero come p = uud e n = udd (qu = 2e/3, qd = e/3).

bSoluzione: Lenergia elettrostatica vale


e2 e2 e2 1
Ep = (qu2 + 2qu qd ) = 0, En = (qd2 + 2qu qd ) =
40 r 40 r 40 3r
1I nuclei sul bordo sono meno legati. Trascuriamo tale effetto, legato alla tensione superficiale, che e importante solo per nuclei
piccoli.
12 Capitolo 1. Campi e potenziali elettrici

Convertendo energia in massa tramite E = mc2 lelettromagnetismo tende a rendere il protone (carico) piu
pesante del neutrone (neutro): mp mn e3 /120 r = MeV 0.5 1015 m/r. Lordine di grandezza e giusto
(mn mp = 1.3 MeV, rN = 1.2 1015 m) ma il segno e sbagliato. Se fosse mp > mn non esisterebbero atomi.
Tenere conto che i quark ruotano con v c (e quindi ceanche una energia magnetica) non cambia il segno. Il
neutrone n ' ddu pesa piu del protone p ' uud perche i quark d hanno massa maggiore dei quark u:

mn mp = md mu + O(e2 /40 r)
| {z } | {z } | {z }
1.3 MeV 3 MeV 1.7 MeV

Il fatto che il neutrone sia poco piu pesante del protone e essenziale per avere una chimica complessa. Il neutrone
libero decade: se fosse mp < mn il protone (e quindi latomo di idrogeno) decadrebbe. Se mn mp fosse un
poco piu grande supererebbe lenergia di legame e non esisterebbero nuclei.2

Esercizio 13: Nucleo che si spezza


Si puo approssimare un nucleo come una sfera a densita costante. Un nucleo si spezza in due nuclei di carica
Q0 = Q/2 e raggio R0 = R/21/3 . Di quanto cambia lenergia elettromagentica?

bSoluzione: Ricordando che U = 3Q2 /5R/40 , lenergia elettromagnetica liberata da un nucleo che si spezza
e
0 3 Q2  Q2
1 22/3 = 0.22

Eem = Eem 2Eem =
5 40 R 40 R
Per Q = 100e e R = rN N 1/3 1014 m viene U 250 MeV: lordine di grandezza e giusto. Leffetto
elettromagnetico e proporzionale a Z 2 , ed a grande Z diventa piu importante delleffetto dovuto alla differenza
di massa protone/neutrone ed alla loro energia di legame, proprozionale a Z.
Un kg di uranio contiene circa 4 moli: quindi fissionandolo si libera unenergia 4NA 250 MeV = NA GeV
1014 J 20kton (kg/1000)c2 (dove kton = 4.2 1012 J e un unita di energia usata per bombe e corrisponde
allenergia rilasciata nellesplosione di 1000 tonnellate di TNT).

Esercizio 14: Energia di due cariche


Per due cariche q1 e q2 ad x = 1.

bSoluzione: Usando il potenziale si ottiene subito U = q1 q2 /40 d. Integrando E 2 i calcoli sono troppo difficili,
ma interessanti perche occorre rinormalizzare

r2 1
Z Z Z
1 2 2 2 1 q1 q2
U= (E 1 + E 2 ) E 1 E 2 = E1 E2 =
8k 4k 4k (r2 2x + 1)3/2 (r2 + 2x + 1)3/2

Lintegrale in r da 4/x2 se x2 > 1 e 0 altrimenti. Lintegrale in x e banale


q1 q2
U =k
d
La cosa qualitativa importante e che due cariche opposte hanno E = 0 nel mezzo: quindi si attraggono in quanto
avvicinandosi minimizzano U . La stessa cosa accade per due fili: siccome il campo B e rotazionale invece che
radiale si ha B = 0 nel mezzo con correnti uguali, che quindi si attraggono.

Esercizio 15: Forza su cariche superficiali


Dimostrare che una generica densita di carica superficiale induce una variazione E1 E2 = /0 e subisce
una forza F = (E 1 + E 2 )/2
2 Lenergia
di legame nucleare e indirettamente dovuta alle masse dei quark che controllano la massa dei (che sono le forze di
van der Waals nucleari) ed e di ordine mu + md .
Capitolo 1. Campi e potenziali elettrici 13

bSoluzione: Applicando il teorema di Gauss ad un cilindretto schiacciato infinitesimo che attraversa perpen-
dicolarmente la superficie si ottiene la variazione di E . E= e continuo.
Per calcolare la forza occorre vedere spessore zero come limite di uno spessore finito. Chiamando z lasse
alla superficie si ha dEz /dz = /0 (relazione che segue dalla dimostrazione Rprecedente, ed e un caso particolare
di E = /0 ). Intergrando in dz si riottiene Ez2 Ez1 = /0 con = dz. La densita di forza vale
E 2 Ez1
2
Z Z
dFz dEz Ez2 + Ez1
pz = = Ez dz = 0 Ez dz = 0 z2 = = Ezext
dS dz 2 2
La media geometrica viene per un motivo fisico semplice: la forza deve essere generata solo dal campo elettrico
esterno, non da quello generato dalla stessa.

Esercizio 16: Scattering debole


Una carica q urta su di un sistema di cariche totali Q. Calcolare il piccolo angolo di deflessione assumendo
simmetria cilindrica (o che sia possibile osservare soltanto uno scattering medio, come capita in esperimenti che
utilizzano un fascio di molte particelle) e che q e ciascuna delle cariche in Q vengano perturbate poco dallurto.

bSoluzione:
R Z
p F dv/x 1 q qQ(r < b) m 2
(b) = = = E dS = K v
p= mv mv 2 2b 40 bK 2
Quindi misurando (b) si fa una tomografia della distribuzione di cariche.

Esercizio 17: Scattering Rutherford


Una particella (di massa me  m  mN , carica +2e, ed energia E = 4 MeV) viene fatta collidere su atomi
contenti nuclei di carica Ze e massa mN . Calcolare langolo di deflessione in funzione del parametro dimpatto
b  A e la sezione durto.

bSoluzione: Siccome m  me ed E  (energia di ionizzazione) gli Z elettroni hanno effetto trascurabile, se


la particella entra nella zona b  A dove gli elettroni non schermano il nucleo. Conta solo il nucleo, che
approssimativamente rimane fermo. Facciamo il conto in 3 modi.

1. Adattando le note formule per le orbite dei pianeti. Tenendo conto che in questo caso la forza F = /r2
( = 2kZe2 ) e repulsiva
` 2EL2 m2 v 4 b2
r= , e2 = 1 + 2
=1+
1 + e cos m 2
Si ha r > 0 per cos < 1/e e cioe in un range dato da cos /2 = 1/e. Langolo di deflessione e
definito come d = e vale quindi
d 1 d 1
sin = cos = o anche tan = = .
2 2 e 2 e2 1 bmv 2
Il parametro dimpatto che produce una data deflessione e
kZe2 d kZe2 Z MeV
b= tan1 dove = 1013 m .
E 2 E 70 E
14 Capitolo 1. Campi e potenziali elettrici

2. Procedendo in modo diretto, senza usare tecniche sofisticate. Usando coordinate polari (r, ) e mettendo
lasse x lungo la linea di simmetria, lequazione del moto e

mvx = cos = m cos
r2 L
dove L = mr2 e il momento angolare rispetto al nucleo, che e una costante del moto, uguale a L = mv0 b.
Siccome F 1/r2 , e sparita la dipendenza da r. Diventa banale integrare ottenendo vx = (/L) sin e
quindi v = vx / cos = (/L) tan . Ad r = tan 0 = mv02 b/. Langolo di deflessione e d = 20 .
3. Approssimazione perturbativa.
dp dx dp 1 1 b /v0
= ' F = = 2
dt dt dx v0 v0 r2 r (x + b2 )3/2
Quindi Z +
p b dx 2
d = = =
p= mv02 (x2 2
+b )3/2 bmv02
p
che Re corretta per d  1. La primitiva e 1/ 1 + b2 /x2 . Usando il teorema di Gauss verrebbe
2b E = 4 dove E = b/(x2 + b2 )3/2 .

Il parametro dimpatto in un singolo urto non e misurabile sperimentalmente. E invece nota la distribuzione
di probabilita dei parametri dimpatto in un numero n  1 di urti. Secondo la meccanica quantistica questa
cosa e vera non solo in pratica, ma anche in linea di principio. Il punto dincontro convenzionale fra teoria ed
esperimento e la sezione durto per collisioni su di un singolo nucleo.
numero di particelle deflesse
=
flusso di particelle incidenti
caratterizza gli effetti misurabili prodotti da un nucleo; spetta allo sperimentale tenere conto che i nuclei sono
tanti (e che ci possono essere scattering multipli). la dimensione di unarea, e dice quanto e grosso un nucleo,
quando viene visto tramite interazioni elettromagnetiche. Ad esempio, la sezione durto totale per eventi con
angolo di deflessione maggiore di un qualunque valore e
2
kZe2

1
(d > ) = b2 () =
E tan2 /2

E.g. (d > /2) dice quante particelle rimbalzano allindietro. La probabilita che una particella rimbalzi
indietro, quando viene inviata perpendicolarmente su di un piano di atomi a distanza d A fra di loro, e
/d2 106 . Se viene mandata su di una targhetta lunga ` composta da atomi con densita n, e n` < 1.
La sezione durto totale e (d > 0) = : a differenza di altre forze, lelettromagnetismo e una interazione
a lungo raggio3 , che deflette tutte le particelle (anche quelle con b ). In realta quando b > A gli elettroni
schermano il campo elettrico del nucleo, e poi ci sono altri nuclei.
Di solito si preferisce descrivere lo scattering usando la sezione durto differenziale

b db 1 kZe2 2
 
numero di particelle deflesse in d d d 1
d = = d cos = =
flusso di particelle incidenti d cos 2 sin d 4 E sin4 /2
Avendo scritto langolo solido in coordinate polari d = d d cos e tenendo conto che lo scattering non dipende
dallangolo polare

Esercizio 18: Esplosione Coulombiana


Una nuvola sferica di raggio R e carica totale Q e costituita da N particelle di carica q = Q/N e massa m,
inizialmente (t = 0) distribuite con densita uniforme.

a) Calcolare lenergia potenziale di una carica posta a distanza r dal centro della nuvola.
particella massiva genererebbe una forza di Yukawa F er/r0 /r 2 (dove m 1/r0 ). Essa darebbe, in approssimazione
3 Una

perturbativa d dCoulomb eb/r0 e quindi ( > 0) r02 .


Capitolo 1. Campi e potenziali elettrici 15

Figura 1.1: Linee di campo (linee continue) e superfici equipotenziali (linee tratteggiate) generate da una sfera
polarizzata.

Per effetto della repulsione coulombiana la nuvola inizia ad espandersi radialmente, mantenendo la simmetria
sferica. Nel corso del moto radiale le particelle non si scavalcano (cioe se inizialmente due strati di particelle si
trovano alle distanze r1 (0) e r2 (0) > r1 (0) dal centro, ad ogni istante successivo r2 (t) > r1 (t).)
b) Sia r = r(t) la posizione al tempo t delle particelle che a t = 0 sono a distanza r0 = r(0) < R dal centro.
Mostrare che lequazione del moto per r = r(t) e

d2 r qQ  r0 3
m = ()
dt2 40 r2 R

c) Si dica a che distanza dal centro si trovano inizialmente le particelle che acquistano la massima energia
cinetica durante lespansione, e si dia il valore di tale energia massima.
d) Si mostri che per ogni strato di particelle si muove secondo la legge oraria r(t) = r0 (t) dove (t) non
dipende da r0 e che di conseguenza la densita di carica rimane uniforme durante lespansione della nuvola.

bSoluzione:
a) (
Q r2 3
V (r) = 40 ( 2R3 + 2R ) per r < R
Q 1
40 r per r > R

b) Poiche le particelle non si scavalcano, la carica contenuta entro una sfera di raggio r(t) rimane costante.
c) Lenergia potenziale corrispondente allequazione del moto (*) e Ur0 = (Q/40 )(r0 /R)3 /r. Lenergia
cinetica massima viene acquistata a distanza infinita ed e uguale a Ur0 , che e massima per r0 = R.
d) Inserendo lansatz nellequazione del moto (*) si trova d2 /dt2 = qQ/40 2 R2 nella quale r0 non compare
piu. Quindi la nuvola si dilata in modo omogeneo.

Esercizio 19: Sfera polarizzata


Calcolare il campo elettrico generato da una sfera di raggio R con carica superficiale () = cos .
16 Capitolo 1. Campi e potenziali elettrici

bSoluzione: Conviene usare il principio di sovrapposizione e vederla come la la distribuzione di carica generata
da tanti piccoli dipoli allineati. Separando le cariche positive da quelle negative, lo si puo anche vedere come
sovrapposizione di due sfere con densita uniformi e con i centri a distanza d tale che d = . La
corrispondenza diventa esatta per d 0 (e quindi ).
Come calcolato precedentemente una singola sfera genera al suo interno un campo elettrico E = r/30 .
Quindi due sfere di carica sovrapposte a distanza d generano al loro interno E = d/30 = P /30 dove
P d viene chiamata densita di polarizzazione.
Allesterno della sfera si ha il campo di un dipolo p = Qd = V P , dove V e il volume della sfera.
Il potenziale in tutto lo spazio, in coordinate sferiche e
p cos /40 r2 per r > R
  3 2
cos R /r per r > R
(r, ) = =
Er cos per r < R 30 r per r < R
La seconda espressione permette di verificare che e continuo a r = R.

Esercizio 20: Cilindro polarizzato


Calcolare il campo elettrico generato da una cilindro polarizzato trasversalmente con carica superficiale () =
cos .

bSoluzione: Si procede in modo analogo alla sfera, sovrapponendo due cilindri con densita uniformi e .

Esercizio 21: Formule di base sui dipoli


Due cariche q a distanza d formano un dipolo p = qd.

bSoluzione: Ricavo le formule di base sui dipoli.


Campo elettrico generato da un dipolo. Espandendo r i = r d/2 (d punta verso la carica positiva)
q 1 1 1 pr
= ( ) ' kqd = k 3
40 r1 r2 r r
   
1 3(p r)r p 1 2(p r)r r (r p)
E = = = +
40 r5 r3 40 r5 r5
In coordinate polari = p cos /40 r2 e quindi
2kp cos 1 kp sin kp p
Er = = 3
, E = = , E = 0 E= 1 + 3 cos2
r r r r3 r3
diverso da zero per ogni . A grande distanza E 1/r3 .
Forza sentita da un dipolo in un campo elettrico esterno:
F = q(d )E = (p ) = U dove U = p E = pE cos
Lenergia e minima quando p si allinea ad E.
Il momento delle forze vale M = p E, M = U = qdE sin .
Negli esercizi successivi discutiamo alcune sottigliezze nelluso di queste formule.

Esercizio 22: Paradosso sui dipoli I


Un dipolo p, obbligato ad orientarsi lungo lasse z e libero di muoversi lungo lasse x in un campo elettrico
esterno Ez = x. Calcolare la forza sul dipolo.

bSoluzione: Secondo le formule precedenti U = px, quindi F = U = px.


Tuttavia e ovvio che la forza totale su due cariche q poste una sopra laltra lungo lasse z e zero.
La formula non si applica perche il campo elettrico proposto non e irrotazionale, come si vede da un circuitino
o da E = y.4
4 Volendo complicare le cose si puo rifare lo stesso esercizio con un campo elettrico E = 1/r, che ha rotore zero in tutti i punti
eccetto 0
Capitolo 1. Campi e potenziali elettrici 17

Esercizio 23: Paradosso sui dipoli II


Calcolare lenergia di un dipolo prodotto da un campo elettrico esterno

bSoluzione: Se p = E il lavoro necessario per portare il dipolo da zero a p vale


pE ?
Z Z
L = F ds = E dp = = Utot
2
Lenergia di interazione fra il dipolo ed il campo elettrico esterno vale U = p E. Affinche Utot = U + U 0 sia
giusta ci deve essere unaltra energia potenziale U 0 = + 12 p E, dovuta al fatto che lesistenza stessa del dipolo
e dovuta alla forza esterna.
Come verifica del risultato generale consideriamo il sistema particolare piu semplice possibile: due cariche
q a distanza x tenute assieme da una forza elastica. Il valore della costante k dovrebbe essere irrilevante. Il
dipolo vale p = qx = qE/k. Lenergia di legame vale
k 2 pE
U0 = x =
2 2
Questo sistema e realizzato fisicamentente dallatomo di Thomson, nel quale avevamo visto che = 40 a30 .
Lenergia U 0 di interazione fra elettrone e protone e di tipo elastico e vale
e2 2 pE
U 0 (r) U 0 (0) = e[(r) (0)] = r =
2 2
(verifico che e giusto usando il laplaciano in coordinate polari: 2 = 3e/ = /0 ).

Esercizio 24: Paradosso sui dipoli III


Calcolare la forza fra due dipoli p e p0 a distanza x, orientati parallalelamente alla loro separazione. Come
cambia la risposta se p0 e indotto da p come p0 = x?

bSoluzione: Abbiamo due formule generali che danno la forza fra dipoli. Vediamo come applicarle.
1. La formula generale e F = (p )E = px E dove
1 3(p0 r)r p0 1 p0
 
E= 5
3
= x r = (x, y, z)
40 r r 20 x3
e il campo elettrico generato da p0 . Quindi la forza e radiale e vale Fx = 3pp0 /20 x4 .
E ovvio che la risposta non cambia se p0 e indotto.
2. Una formula meno generale ma piu semplice e F = U con
1 pp0
U = p E =
20 x3
Nel caso di p0 costante si riottiene il risultato precedente. Nel caso di dipolo indotto, p0 = x, viene
un risultato che differisce di un fattore 2/3: per utilizzare correttamente questa formula meno generale
occorre prima calcolare il come se p0 fosse costante e poi inserire il valore del dipolo indotto p0 (x).

Esercizio 25: Paradosso sui dipoli IV


Due dipoli sono orientati rispettivamente lungo gli assi z ed x. Verificare che le forze sono uguali ed opposte
(ma non radiali). Verificare che i momenti non lo sono.

bSoluzione: Il momento totale e zero se calcolato rispetto ad un polo fisso. Una situazione analoga piu banale:
un dipolo nel campo elettrico di una carica. Il dipolo sente un momento delle forze (che tende ad allinearlo con
il campo elettrico), ma anche una forza.
18 Capitolo 1. Campi e potenziali elettrici

Esercizio 26: Allineamento di dipoli elettrici


Come si dispongono due dipoli a distanza r fissata, liberi di ruotare su loro stessi?

bSoluzione: Lenergia fra due dipoli pi = qi di a distanza r fissata (senza vincoli non esistono configurazioni
di equilibrio stabile)

k
U= [p p 3(p1 r)(p2 r)] cos(1 2 ) 3 cos 1 cos 2
R3 1 2
che e minima a 1 = 2 = 0 (piu configurazioni simmetriche), cioe per dipoli stesi nella stessa direzione. Tenerli
verticali e contrapposti richiede energia maggiore.
Un dipolo puo essere la molecola H2 O (lO attira gli elettroni piu di H), oppure latomo di idrogeno in un
campo esterno.

Esercizio 27: Coordinate polari


Calcolare gradiente, divergenza, rotore e 2 in coordinate cilindriche e polari ed in un generico sistema di
coordinate ortogonali.

bSoluzione: Il e sia un operatore differenziale che un vettore. Questo significa e.g. che (f E) = ... e che
f = 0, (f ) (g) 6= 0, ( E) = 0. In linea di principio per passare a coordinate polari si procede
come per altri vettori, rispettando le proprieta delloperatore derivata. In pratica si fa molto prima usando i
teoremi in cui compaiono gradienti, rotori e divergenze: i teoremi del gradiente (linee), Stokes (superifci), Gauss
(volumi) sono casi particolari di integrali di forme asimmetriche a n indici
Z I
A(n) dx(n+1) = A(n) dx(n)
X X

e cioe (in 3 dimensioni):


Z I Z I Z I
f dx = f = f ( E) n dS = E dx ( E)dV = E n dS
L L=P S S=L V V =S

Gradiente In un generico sistema di coordinate ortogonali xi si ha


X f
df = f (x + dx) f (x) = dxi dx f
i
xi

1
Siccome dx = gii dxi xi allora = gii xi i . E.g. in coordinate polari grr = 1, g = r e g = r sin in quanto
dx = dr r + r d + r sin d e ds = dr2 + r2 d2 + r2 sin2 d2
2

1 1
= x + y + z = r + +
x y z r r r sin

Divergenza Per calcolare la divergenza di un generico vettore E usiamo il teorema di Gauss


Z Z
( E)dV = E dS

applicato ad un volumetto elementare di lati dxi . La differenza dei flussi sui lati lungo x1 vale

dV
(E1 g22 g33 )+ dx2 dx3 (E1 g22 g33 ) dx2 dx3 = dx1 dx2 dx3 (E1 g22 g33 ) = (E1 g22 g33 )
x1 g11 g22 g33 x1

Quindi  
1
E = (E1 g22 g33 ) + (E2 g11 g33 ) + (E3 g11 g22 )
g11 g22 g33 x1 x2 x3
Capitolo 1. Campi e potenziali elettrici 19

In coordinate polari la divergenza di un vettore con solo componente radiale vale

1 1
E = Er r2 sin = 2 r2 Er
r2 sin r r r
Riotteniamola procedendo in un altro modo, senza usare il teorema di Gauss: Applicato ad un vettore radiale
E = rEr vale

Er r r Er 1
E = + Er ( + )= + (1 + 1)Er = 2 r2 Er
r r r sin r r r

Su di una funzione V che dipende solo da r

1 2 1 2
2 V = ( )V = r V = rV
r2 r r r r2
In d dimensioni (d = 3 coordinate polari, d = 2 coordinate cilindriche, etc)

1 d1 2V d 1 V
E = r Er 2 V = +
rd1 r r2 r r

Rotore Utilizzando in modo analogo il teorema di Stokes si ottiene unespressione esplicita per il rotore. Il
2 e banale

g11 x1 g22 x2 g33 x3  
1 2 1 g22 g33
E = det 1 2 3 , V = 1 1 V +
g11 g22 g33 g11 g22 g33 g11
g11 E1 g22 E2 g33 E3

Ad esempio in coordinate polari

2 (rV ) (sin V ) 2 V
 
1 r sin r
2
V = 2 2
r (r sin r V ) + ( V ) + V = r + +
r sin r r sin r r2 sin r2 sin2

ed in coordinate cilindriche

r (rr V ) 2 V
 
2 1 1
V = r (rr V ) + ( V ) + z (rz V ) = + 2 + z2 V
r r r r

Esercizio 28: Laplaciano


2 p
Calcolare r in d dimensioni spaziali.

bSoluzione: In 1 dimensione r2 = x2 . In 2 dimensioni r2 = x2 + y 2 . In 3 dimensioni r2 = x2 + y 2 + z 2 . In


generale
x rp = px rp2 , x2 rp = p rp2 + p(p 2)x2 rp4 , 2 rp = p[d + p 2]rp2
Il caso d = 3 corrisponde a coordinate polari; il caso d = 2 a coordinate cilindriche.
Il potenziale generato da una carica in d dimensioni e la soluzione singolare a r = 0 di 2 = 0, e cioe per
r2d . Quindi
d=1 d=2 d=3 d=4 d=5
r ln r r1 r2 r3
E r0 E r1 E r2 E r3 E r4
(il campo generato da una carica puntiforme in d = 1 e d = 2 corrispondono rispettivamente al campo di un
filo e di un piano in d = 3).
La cosa fondamentale non e 1/r ma la conservazione del flusso, e cioe E 1/S o 2 = 0.
Solo d = 3 da fisica interessante. Per d < 3 non esistono cariche libere ( cresce con r), per d > 3 lenergia
cinetica di rotazione (potenziale effettivo V = L2 /2mr2 ) non basta ad impedire che si spiaccichino ad r = 0.
20 Capitolo 1. Campi e potenziali elettrici

Esercizio 29: Potenziale di Yukawa


2 2
Risolvere = = 0

bSoluzione: La soluzione a simmetria sferica (r) e


1
(r)00 = 2 : r = er
r
Per r  e come lelettromagnetismo, per r  la forza va a zero esponenzialmente. Lo Z ha 1016 cm.
La gravita potrebbe avere 1010 anni luce.

Esercizio 30: Atomo di idrogeno quantistico


Risolvere lequazione di Schroedinger

bSoluzione: Secondo Schroedinger uno deve: scrivere lenergia H, rimpiazzare p ih, risolvere H = E,
e ||2 e la probabilita. Per un elettrone in un atomo di idrogeno

p2 qe2 h2 2 e2
H=
2m 40 r 2m r

avendo usato e2 = qe2 /40 . Quindi, usando u = r

h2 e2
(r)00 = (E + )(r)
2m r
risolto da er/a0 ed E = e2 /2a0 dove a0 h2 /me2 . Quindi secondo la meccanica quantistica la carica
dellelettrone si distribuisce come (r) = e e2r/a0 /a30 .
Capitolo 2

Conduttori

Le cariche elettriche dentro un conduttore sono libere di redistribuirsi; e finche E 6= 0 continuano a spostarsi.
Quindi si riaggiustano (dissipando energia termicamente) fino a raggiungere la condizione di equilibrio stabile:
E = 0 dentro il conduttore ed E= = 0 lungo la sua superficie (cioe = costante nel conduttore). Quindi il
campo elettrico subito fuori da un conduttore vale E = /0 . La pressione sentita da un conduttore e E /2.

Esercizio 31: Piano conduttore


Studiare una carica q puntiforme posta a distanza d da un piano conduttore infinito posto a potenziale zero.

bSoluzione: Scelgo le coordinate in modo che il piano e a x = 0 e la carica a (x, y, z) = (d, 0, 0). Usando il
metodo delle immagini si trova il potenziale
 
q 1 1
per x > 0
(x, y, z) = 40 |x d| |x + d|
0 per x < 0

La densita superficiale di carica indotta sul piano e

1+1 dq
(x, y) = 0 E = 0 =
x 4 (x + y + d2 )3/2
2 2

R
e la carica totale e dx dy = q, come si puo verificare o facendo esplicitamente lintegrale, o applicando il
teorema di Gauss ad una superficie chiusa ottenuta chiudendo il piano a r = (lunico contributo al flusso
e sul piano a r d).
La carica q sente una forza attrattiva Fx = kq 2 /(2d)2 . Il piano sente una forza opposta, come si puo
verificare integrando dF/dS = E /2 (non dimenticando il fattore 2: E /2 e la media del campo elettrico
subito fuori e subito dentro il conduttore)

kq 2
Z
E
F = dx dy =
2 (2d)2
0
R la carica2 da x = d ad x = (o piu in generale, ad un punto x = d ) occorre compiere un lavoro
Per portare
L = d F dx = kq /4d, uguale a meta della variazione dellenergia potenziale fra carica e carica-immagine
V () V (d) = kq 2 /2d, in quanto non serve lavoro per spostare la carica immagine.
Se il piano conduttore non e posto a = 0 ma e isolato non cambia niente. Infatti, se e finito ma grosso
(dimensioni D  d) ed ha carica totale zero, la carica q induce una carica q nella zona vicina, e quindi una
carica +q concentrata ai bordi lontani (che non si vede se D ).

Esercizio 32: Lastra conduttrice


Si calcoli il campo elettrico in presenza di una carica q situata a distanza d da una lastra conduttrice piana di
spessore finito s.

21
22 Capitolo 2. Conduttori

Figura 2.1: Tentativi di usare il metodo delle immagini.

bSoluzione: E immediato verificare che



come prima dal lato dove ce q, i.e. x > 0
(x, y, z) = 0 dentro il conduttore, i.e. s < x < 0
0 dal lato opposto, i.e. x < s

e una soluzione, e quindi e la soluzione. Se d > s la carica immagine q viene fuori dalla lastra, ma questo e
irrilevante in quanto e scritto in termini di q solo sul lato dove ce q. Dal lato opposto dove non ce la carica
E = 0. Se il conduttore dal lato opposto non fosse piano ma avesse una forma artistica, rimarrebbe sempre
E = 0.
Questo fenomeno e piu generale: un conduttore scherma da altre cariche lo spazio che circonda. Se un
conduttore contiene un buco vuoto, allora dentro E = 0. Infatti la ovvia unica soluzione dellequazione di
Poisson con condizioni al bordo (bordo) = 0 e (buco) = 0 . Questo accade perche F 1/rn con n = 2, e
consente di verificare sperimentalmente quanto n e veramente vicino a 2.
Per schermare un campo elettrico (stazionario) non serve racchiudere tutto con un conduttore: una griglia
conduttrice a maglie piccole basta a fare un buon lavoro (per lo stesso motivo discusso in un esercizio analogo
a pag. 73).

Esercizio 33: Metodo delle immagini


Trovare e studiare altri casi simili.

bSoluzione: Usando la linearita e immediato studiate altri casi: se ci sono due cariche q1 e q2 basta sommare
le soluzioni. Se ce un dipolo, si aggiunge un dipolo immagine. Se si vuole studiare un filo carico sospeso sopra
il terreno, si considera un filo immagine sottoterra.
Con una carica posta vicino a semipiani che si intersecano ad un dato angolo, il metodo funziona solo per
angoli speciali. Usualmente riflettendo si trova che servirebbe mettere cariche nella zona vuota (vedere fig. 2.1)
per cui si ottiene una soluzione per 2 o piu cariche messe in posti speciali
Il metodo funziona per angolo di 90 , e servono 3 cariche immagini (fig. 2.1a). E interessante studiare in
che modo il lavoro necessario a spostare la carica e legato allenergia potenziale fra q e cariche immagini. Per
semplicite mettiamo la carica lungo lasse di simmetria a distanza d dai piani: risente una forza attrattiva diretta
lungo lasse con modulo
Z
(4 + 2)kq 2
 
2 1 1
F = kq 2
+ , L = F ds =
2 (2d)2 (2 2d)2 2d 8d

Il lavoro e uguale allenergia potenziale della sola carica vera, e quindi ad 1/4 dellenergia potenziale di tutte
le cariche (vera ed immagini)

kq 2 2
 
k X q1 qi 1 k X qi qj
V (1) = = + = L, V (1) + V (2) + V (3) + V (q) = = 4V.
2 r1i 2 2d 2 2d 2 rij
i6=1 i6=j
Capitolo 2. Conduttori 23

Esercizio 34: Piano carico fra 2 piani conduttori


Due piani conduttori paralleli a distanza ` sono tenuti allo stesso potenziale. Una carica q, distribuita uniforme-
mente lungo un piano, viene messa a distanze e 0 = ` dai piani. Calcolare le cariche totali indotte q e
q0 .

bSoluzione: Ovviamente q + q 0 = q. I campi elettrici sono costanti. Siccome i due conduttori sono allo stesso
potenziale, E = E 0 (s ). Le cariche indotte sono legate ai campi da E = /0 e quindi
q E `
= 0 = 0 =
q0 E
Quindi q = q(1 /`) e q 0 = q/`.

Esercizio 35: Carica fra 2 piani conduttori


Due piani conduttori paralleli a distanza ` sono tenuti allo stesso potenziale. Una carica puntiforme q viene
messa a distanze e 0 = ` dai piani. Calcolare le cariche totali indotte q e q 0 .

bSoluzione: Si potrebbe usare una serie infinita di cariche immagini q situate a x = 2n` , ma il conto
sarebbe troppo difficile.

PR RP
(la carica indotta non dipende dalla distanza, ma 6= ). Visto che il problema chiede solo la carica totale
indotta, convene usare un altro trucco. Immaginiamo che q invece di essere puntiforme sia distribuita lungo un
piano parallelo ai due conduttori, come nellesercizio precedente. La carica totale indotta resta la stessa. Con
lo stesso trucco si potrebbe anche calcolare la forza sentita dalla carica.

Esercizio 36: 1 lastre conduttrice carica


Una lastra di superficie S ha carica totale q. Calcolare i campi elettrici indotti.

bSoluzione: Fisicamente uno si aspetta che la carica q si divida equamente fra le due superfici, generando un
campo elettrico esterno E = (q/2)/S0 ortogonale alla lastra ed uguale sui due lati.
Questo accade non perche alle cariche piace disporsi simmetrimecamente, ma perche questa e la configu-
razione di minima energia, come discusso nel meno semplice problema successivo.

Esercizio 37: 2 lastre conduttrici cariche


Due lastre di superficie S hanno cariche totali q e q 0 . Calcolare i campi elettrici indotti.

bSoluzione: Il problema consiste nel trovare come le cariche si ripartiscono fra le superfici destra e sinistra
delle lastre. In generale le cariche sulle 4 superfici (da sinistra a destra) possono essere
q Q, Q Q q0 + Q
dove Q e incognito. I campi elettrici sono
qQ q q0 + Q
Esinistra = , Emezzo = , Edestra =
0 S 0 S 0 S
Lenergia totale e proporzionale allintegrale di E 2 , dominato dal grande spazio a sinistra ed a destra. Quindi
2
le cariche minimizzano Esinistra 2
+ Edestra . Questo accade nella configurazione simmetrica, Q = (q q 0 )/2:
q + q0 q q0
Esinistra = Edestra = , Emezzo =
20 S 20 S
24 Capitolo 2. Conduttori

Esercizio 38: Capacitatore cilindrico


Un cavo coassiale e fatto di un filo conduttore interno di diametro d circondato da un guscio metallico di diametro
D. Calcolare la capacita. Sapendo che laria puo sostenere Emax = 3MV/ m (rigidita dellaria) trovare quale
valore di d/D consente di avere la massima differenza di potenziale, e quale d/D consente di immagazzinare la
massima energia.

bSoluzione: Usando il teorema di Gauss possiamo immediatamente calcolare il campo elettrico nello spazio
vuoto fra le due armature: 2r Er = /0 da cui
2k
Er = = : = 2k ln r.
r r
Quindi la differenza di potenziale vale V = = 2k ln(D/d) e la capacita per unita di lunghezza vale
20 /ln D/d. Imponendo che il massimo campo elettrico E(r = d/2) = 4k/d sia uguale a Emax si trova che la
massima differenza di potenziale vale
d D
V = Emax ln
2 d
Per esempio V = 3.45 kV se d = 1mm e D = 1cm. Fissato D, V e massimizzato scegliendo d = D/e.
Lenergia immagazzinata in una lunghezza L vale

CV 2 2 d D
U= = L0 Emax ( )2 ln
2 2 d
ricalcolabile anche come
D/2
0 E 2
Z Z
0 d r dr
U= dV = L2( )2 Emax
2
.
2 2 2 d/2 r2

Fissato D, lenergia immagazzinata e massimizzata scegliendo d = D/ e.

Esercizio 39: Capacitatore di dimensioni variabili


Si raddoppia la distanza fra i piatti di un capacitatore di capacita C. Quanto lavoro meccanico occorre fare se
(a) le cariche sui piatti sono tenute costanti? (b) una batteria mantiene costante la differenza di potenziale V ?

bSoluzione: Ricordo che Q = CV . Per due piatti conduttori di area A a piccola distanza d

V = Ed = d/0 = Q d/A0 cioe C = 0 A/d

Raddoppiare d dimezza C. Allinizio lenergia vale

0 E 2 A0 V 2 CV 2 Q2
U = Ad = = = .
2 d 2 2 2C
(a) Alla fine U 0 = Q2 /2C 0 = 2U quindi L = U U 0 = U . Infatti il campo elettrico rimane uguale,
ma occupa un volume doppio. I due piatti si attraggono, quindi occorre una forza F = L /(d) per
allontanarli.
In generale, quando uno modifica un capacitatore variando la capacita di dC tenendo la carica Q costante

Q2 1 Q2 V2
F ds = dL = dU = d = 2
dC = dC
2 C 2C 2

(b) Alla fine U 0 = C 0 V 2 /2 = U/2 quindi Ltotale = U U 0 = U/2 > 0. Questo sembra suggerire che i due
piatti si respingano, mentre invece uno si aspetta che si attraggano esattamente come nel caso precedente
(in quanto contengono cariche di segno opposto). Il punto e che Ltotale e il lavoro totale, somma di
due contributi: un contributo meccanico (legato alla forza necessaria per spostare le armature), ed un
lavoro ricevuto dalla batteria mano a mano che le cariche sulle armature diminuiscono. La carica finale
vale Q0 = C 0 V = Q/2. Una carica Q = Q/2 viene spinta dentro la batteria, che riceve un lavoro
Lbatteria = QV /2 = U . Lmeccanico = Ltotale Lbatteria = U/2 < 0.
Capitolo 2. Conduttori 25

In generale la batteria riceve un carica dQ e quindi un lavoro Lbatteria = V dQ = V 2 dC. Lenergia


nel capacitatore varia di V 2 dC/2. Quindi il lavoro meccanico vale

V2
F ds = dLmeccanico = dLtotale dLbatteria = dC
2
come nel caso (a).

Quindi in generale la forza e legata alla variazione della capacita C da F = (V 2 /2)(dC/ds), e tende ad aumentare
la capacita. In questo esercizio abbiamo solo ottenuto un risultato atteso in modo complicato. Il prossimo e piu
interessante.
Ad esempio se inserisco una barra conduttrice in un condensatore...

Esercizio 40: Conduttore in capacitatore


Un conduttore di spessore d viene parzialmente inserito in un capacitatore quadrato di spessore D e lunghezza
L  D mantenuto ad una differenza di potenziale V . Calcolare la forza sentita dal conduttore mobile.

bSoluzione: Il sistema e disegnato in fig. 3.1a. Possiamo vederlo come una capacita C0 = 0 L(L x)/D in
parallelo con una capacita C1 = 0 Lx/(D d) (costituita da 2 capacita in serie). Inserire un conduttore e un
pocome ridurre la distanza fra i piatti: per questo la capacita aumenta. Il valore preciso e

0 L2
 
xd
C = C0 + C1 = 1+
D L(D d)

Come visto in precedenza la forza vale

V 2 dC V 2 dL0
F = = .
2 dx 2 D(D d)

Fisicamente la zona dove agisce la forza e la punta del conduttore: sebbene in quella zona non sappiamo fare i
calcoli, sappiamo calcolare la forza totale. Determiniamo ora il segno delleffetto. Non mi pare possibile capirlo
in modo intuitivo, ma solo affidandosi al formalismo. Nel caso banale dellesercizio precedente la forza tende ad
attrarre i piatti, cioe ad aumentare la capacita. Quindi in questo esercizio il conduttore viene attratto dentro il
condensatore, perche questo aumenta la capacita.

Esercizio 41: Sfera conduttrice a terra


Una carica q e situata a distanza R dal centro di una sfera conduttrice a potenziale zero di raggio r.

bSoluzione: Serve una carica immagine q 0 = q r/R messa come in figura fig. 2.2, situata a distanza r2 /R dal
centro della sfera.
Un giorno qualcuno noto che il potenziale generato da due cariche = kq1 /r1 + kq2 /r2 vale zero su di una
sfera. Infatti = 0 a r1 /r2 = q1 /q2 i.e. r12 q22 + r22 q12 = 0 che e lequazione della sfera. La sfera e il luogo dei
punti per i quali le distanze fra 2 punti sono in rapporto fisso.
Torniamo al problema, che proviamo a risolvere usando un sistema di coordinate con origine nel centro della
sfera ed aggiungendo una carica immagine q2 = q 0 a distanza r2 dal centro. Per fissare il tutto basta imporre
= 0 nei 2 punti del conduttore lungo lasse:
q1 q2 q1 q2
+ = 0, + =0
R r r r2 R + r r + r2

sono risolte da r2 = r2 /R e q2 = q1 r/R. La sfera conduttrice ha carica totale q 0 . Se avesse carica diversa (e.g.
zero) o potenziale diverso, basterebbe aggiungere una ulteriore carica immagine q 00 nel centro.
Usando coordinate polari , il potenziale vale
 
1 r 1 1 r 1
(, ) = kq( ) = kq p p
r1 R r2 2 + R2 2R cos R 2 + (r2 /R)2 2(r2 /R) cos
26 Capitolo 2. Conduttori

Figura 2.2: Fig. 2.2a,b,c: linee di campo in presenza di una carica q a distanze varie da una sfera conduttrice
a terra. Fig. 2.2d: linee di campo per una sfera conduttrice isolata in campo elettrico esterno.

La densita di carica superficiale vale

q(R2 r2 )


() = 0 Er ( = r, ) = 0 =
=r
4r(R + r2 2rR cos )3/2
2

La carica totale indotta sulla sfera e q 0 , che e diversa da q. La forza attrattiva fra la sfera e la carica q vale
q1 q2 rR
F =k 2 = kq 2 2
r12 (R r2 )2

e decresce come 1/R3 per R  r. Il lavoro necessario a spostare la carica q rispetto alla sfera da distanza R a
distanza R0 vale L = qq 0 /80 (1/R0 1/R).

Esercizio 42: Sfera conduttrice isolata


Come lesercizio precedente, ma la sfera e isolata

bSoluzione: Occorre aggiungere una ulteriore carica immagine q 00 = q 0 in modo che la carica immagine
totale sia zero. Per fare in modo che la sfera rimanga a potenziale costante occorre mettere q 0 nel centro della
sfera. Per R  r la forza fra sfera e carica q decresce come 1/R4 . Il lavoro necessario a spostare la carica q
rispetto alla sfera e uguale a L = 12 [Vqq0 + Vqq00 ] cioe senza includere la variazione di energia potenziale
Vq0 q00 fra le due cariche immagini.

Esercizio 43: Sfera conduttrice in E costante


Una sfera conduttrice isolata di raggio r viene messa in un campo elettrico E0 esterno costante.

bSoluzione: Si puo trovare la soluzione in diversi modi, sviluppando ulterioremente esercizi precedenti.

Partendo dal problema precedente, posso generare un campo elettrico costante usando una carica q a
distanza R dalla sfera nel limite q, R tenendo costante E0 = q/40 R2 . In questo limite la carica
immagine q 0 = qr0 /R diverge e si avvicina al centro della sfera, dove si trova la seconda carica immagine
q 00 = q 0 , ma le due cariche immagine generano un dipolo finito p = q 0 d = 40 E0 r3 . Riassumendo:
fuori dalla sfera

E = E 0 + (campo generato da un dipolo p nel centro della sfera).

Verifichiamo che il potenziale (, ) e costante sulla superficie della sfera a = r

r3
 
p
(, ) = 2
E 0 cos = ( 2 )E0 cos
40
Capitolo 2. Conduttori 27

Figura 2.3: (a) Condensatori in serie. (b) Condensatori in serie.

La densita superficiale di carica vale




() = 0 = 30 E0 cos
=r

Abbiamo quindi ritrovato la situazione studiata a pagina 15: una sfera con carica superficiale () =
0 cos . Avevamo trovato che genera al suo interno un campo elettrico costante E = 0 /30 , che per
0 = 30 E0 e esattamente opposto al campo esterno E0 . In questo modo dentro la sfera si ha E = 0.
Avevamo anche trovato che allesterno genera il campo di un dipolo p = V 0 , che per 0 = 30 E0 vale
p = 40 E0 r3 in accordo con il risultato precedente.

Le linee di campo sono disegnate in figura 3.1d.

Esercizio 44: Tetraedro conduttore


4 triangoli equilateri conduttori, mantenuti a potenziali 1,2,3,4 vengono disposti in modo da formare la superficie
di un tetraedro. Quale e il potenziale nel centro?

bSoluzione: In generale deve essere una combinazione lineare dei 4 contributi. Infatti se so risolvere il caso
con solo 1 acceso (1 6= 0 e 2,3,4 = 0), e poi so risolvere
P il caso con solo il 2 acceso, sommando le due soluzioni
ho risolto anche il caso con 1 e 2 accesi. Quindi = i ci i . P
Poi, per motivi di simmetria, la risposta deve essere simmetrica in 1, 2, 3, 4. Quindi = c i .
Per finire c = 1/4 (cioe = (1 + 2 + 3 + 4 )/4) in quanto nel caso 1 = 2 = 3 = 4 il potenziale deve
avere il valore comune costante, siccome le 4 facce formano un tetraedro chiuso.

Esercizio 45: Condensatore sferico


Un condensatore e costituito da una sfere concentriche di raggi r1 ed r2 . Calcolare la capacita e discutere il
limite r2 .

bSoluzione:
Q 40
C= = 1
V r1 r21
Se r2  r1 il valore di r2 conta poco e si puo pensare ununica sfera come un condensatore di capacita C = 40 r1
avente laltro piatto ad infinito.

Esercizio 46: Condensatori in serie


Due condensatori di capacita C1 e C2 con cariche Q1 e Q2 vengono connessi come in fig. 2.3a. Come si
redistribuiscono le cariche?

bSoluzione: La corrente flusice lungo la resistenza, dissipando energia, fino a che i due condensatori hanno
equali differenze di potenziali. Imponendo
Q01 Q0
V = = 2 e Q1 + Q2 = Q01 + Q02
C1 C2
si trova Q0i = Ci (Q1 + Q2 )/(C1 + C2 ).
28 Capitolo 2. Conduttori

Esercizio 47: Effetto delle punte


Due sfere conduttrici cariche di raggi r ed R lontane sono connesse da un filo. Mostrare che il campo elettrico
attorno alla sfera piccola e piu grosso che attorno alla sfera grossa.

bSoluzione: Le cariche in un conduttore carico si respingono, e quindi cercano di andare il piu possibile lontane
le une dalle altre, generando una forte concentrazione di cariche sulle punte. La sfera piccola schematizza una
punta e consente di fare un calcolo esplicito.
Le cariche q e Q sulle due sfere si determinano imponendo che i potenziali sulle superfici delle due sfere siano
uguali:
Q q
=
R r
Questo corrisponde a quanto visto nellesercizio precedente: Qi Ci ri . Quindi il campo elettrico e grosso
attorno alla sfera piccola
E(r) q/r2 R
= = .
E(R) Q/R2 r
Il massimo campo elettrico che laria asciutta puo sopportare e e qualche MV/m (con campi elettrici piu forti
rendono laria conduttrice dando luogo a scariche). Mettendo delle punte su di un parafulmine ci si assicura
che una nuvola carica eletricamente si scarichi su di esse.

Esercizio 48: Sfera conduttrice bucata


Una sfera conduttrice scarica contiene, al suo interno ma non al suo centro, un buco con dentro una carica q.
Calcolare il campo elettrico generato.

bSoluzione: Nonostante lassenza di simmetria sferica, il campo elettrico esterno e uguale a quello generato
da una carica q al centro del conduttore. Infatti, lunica soluzione dellequazione di Poisson costante sulla sfera
e 1/r. Non esiste una soluzione semplice per il campo elettrico nel buco.

Esercizio 49: Carica dentro sfera


(Dal compito del 16/1/2004). Una carica puntiforme q e posta allinterno di un guscio conduttore sferico di
raggio interno R e raggio esterno R0 , a distanza d dal centro. Il guscio conduttore e posto a terra. Calcolare

a) Il potenziale ed il campo elettrico in tutto lo spazio.


b) La forza sulla carica q.
c) Mostrare che la carica totale indotta sulla sfera e pari a q.
d) Come cambia la risposta a) se il guscio conduttore e isolato?

bSoluzione:
a) E noto che due cariche q e q 0 = qR/d a distanze dd0 = R2 dal centro di una sfera producono potenziale
zero sulla sfera. Questo e il sistema di cariche immagini che ci serve per calcolare E nella zona interna. Il
fatto che il conduttore abbia spessore finito non complica il problema. Dentro il conduttore e nella zona
esterna E = 0.
b) La forza e attrattiva e vale F = qq 0 /40 (d d0 )2 .
c) Siccome fuori E = 0, la carica totale (q + carica indotta) e zero.
d) La carica totale ora e q. Sulla superficie interna si dispone una carica totale q distribuita in modo da
schermare, a r > R leffetto della carica puntiforme. Sulla superfcie esterna si dispone uniformemente una
carica totale q, generando un campo radiale E = q/40 r2 . Dentro E rimane come prima.
Capitolo 3

Dielettrici

La densita di polarizzazione indotta da un campo elettrico esterno P = 0 E induce una densita di cariche di
polarizzazione P = pol (e sui bordi una densita superficiale pol = P ). Separando la carica totale in
tot = free + pol e definendo D = P + 0 E E = P /( 1) il campo D soddisfa a D = free . Se
e costante un dielettrico e descritto dalle stesse equazioni del vuoto con 0  = 0 dove = 1 + . Se
varia bruscamente le condizioni di raccordo su bordi senza cariche libere sono: E= = 0 e D = 0 (cioe
1 E1 = 2 E2 ).

Esercizio 50: 2 dielettrici in condensatore piano


Calcolare la capacita di un condesatore piano ottenuto mettendo due diversi dielettrici fra due piatti conduttori
come mostrato in figura.

bSoluzione:

1 2
1

2
2

1) Chiamo ` lo spessore totale, diviso in `1 ed `2 . Nel primo caso conviene usare il campo D perche e costante:

D = = 1 E1 = 2 E2

Quindi la differenza di potenziale V e la capacita C = Q/V valgono

Q `1 `2 1 1 1 i S
V = E1 `1 + E2 `2 = ( + ) cioe = + dove Ci = .
S 1 2 C C1 C2 `i
Questa geometria corrisponde ad avere due condensatori in serie.

2) Nel secondo caso conviene usare il campo elettrico perche uguale nelle due zone, visto che E= = 0
lungo il bordo e che la differenza di potenziale e la stessa nelle due zone. Quindi la densita di carica
totale e la stessa nelle due zone; ma ci interessa la capacita che e definita in termini della carica libera
come C = Qfree /V . Chiamiamo 1 e 2 al diversa densita di carica libera nelle due zone. Siccome hanno
uguale area la densita media di carica media e = (1 + 2 )/2. Esse determinano il campo elettrico come
Ei = i /i . Imponendo E1 = E2 si ottiene

i Q/S S
i = e quindi V = Ei ` = C = (1 + 2 ) = C1 + C2 .
1 + 2 1 + 2 `
Questa geometria corrisponde ad avere due condensatori in parallelo.

29
30 Capitolo 3. Dielettrici

Esercizio 51: N dielettrici in condensatore piano


Ripetere lesercizio precedente mettendo N dielettrici di egual spessore `/N e costanti dielettriche i = 1 +
(2 1 )i/N . Ottenere il risultato nel limite N .

bSoluzione:
1) Quando sono in serie
N N Z `
1 X 1 X z dz ` ln(1 /2 )
= = =
C i=1
Ci S
i=1 i 0 (z)S S 1 2
dove (`) = 1 + (2 1 )z/`.
2) Quando sono in parallelo, scrivendo S = L2
N N Z L
X X i L x (x)dx L S 1 + 2
C= Ci = =
i=1 i=1
` 0 ` ` 2

dove (x) = 1 + (2 1 )x/L.

Esercizio 52: Condensatore in acqua


Un condensatore cilindrico di lunghezza L e diametri esterno ed interno D e d, mantenuto ad una differenza di
potenziale V , viene immerso verticalmente in una bacinella dacqua, di densita . Calcolare di quanto si innalza
il livello dellacqua dentro il condensatore rispetto al livello esterno.

bSoluzione: Abbiamo visto che inserendo un dielettrico fra le armature di un condensatore se ne aumenta la
capacita, e che quindi le forze elettriche Fel = dUel /dz tendono a far salire lacqua dentro il condensatore. Al
contrario la forza gravitazionale Fgrav tende a farla scendere. Lacqua salira fino ad un livello z tale che queste
due forze si bilanciano. Calcoliamole.

Se lacqua entra nel condensatore di un tratto z la capacita vale C(z) = 2(z + (L z)0 )/ ln(D/d) =
C(0) + 20 z/ ln(D/d). avendo definito  = (1 + )0 . La forza elettrica non dipende da z:
V 2 dC V 2 0
Fel = + = .
2 dz ln D/d

La forza gravitazionale cresce con z:


D 2 d2
Fgrav = m(z)g = z g
4
Imponendo Fgrav + Fel = 0 si trova
4V 2 0
z=
(D2 d2 )g ln(D/d)
cioe misurando z si puo ricavare . Numericamente viene z mm per D mm e V kV.

Esercizio 53: Carica davanti a semipiano dielettrico


Lo spazio e riempito da due semipiani dielettrici aventi a sinistra costante dielettrica 2 , ed a destra 1 . Una
carica si trova a destra. Trovare i campi elettrici.

bSoluzione: Provo: in 1 il campo di vuoto generato da q e da una q 0 immagine. In 2 il campo di vuoto di una
q 00 al posto di q.
(q + q 0 ) cos q 00 cos (q + q 0 ) sin q 00 sin
E=1 = , E=2 = , E1 = , E2 =
r2 r2 r2 r2
Capitolo 3. Dielettrici 31

L P

D d

Figura 3.1: Esercizi su forze.

Le condizioni di raccordo sono


1 2 21
q 00 = q + q 0 , 1 (q q 0 ) = 2 q 00 : q0 = q , q 00 = q
1 + 2 1 + 2

Per 2 si ritrova il conduttore. La carica q vale qvera /1 .

Se invece 1  2 E1 e piccolo, come intrappolare una carica.

Esercizio 54: Dielettrico in condensatore


Un dielettrico di costante dielettrica relativa e spessore d viene parzialmente inserito in un condensatore
quadrato di spessore D e lunghezza L  D mantenuto ad una differenza di potenziale V . Calcolare la forza
sentita dal dielettrico.

bSoluzione: Il sistema e disegnato in fig. 3.1a. Possiamo vederlo come una capacita C0 = 0 L(L x)/D in
parallelo con una capacita C1 , costituita da 2 capacita in serie: C10 = 0 Lx/(D d) e C100 = 0 Lx/d. Quindi
la capacita totale vale

1 0 L(L x) L0
C = C0 + C1 = C0 + = +x
1/C10 + 1/C100 D d d + D

Come visto in precedenza la forza vale

V 2 dC V 2 dL0 ( 1)
F = =
2 dx 2 D(d d + D)

Il dielettrico viene attratto dentro il condensatore. Per si ritrova il conduttore in condensatore studiato
a pagina 25, Abbiamo potuto trascurare gli effetti ai bordi e sulla punta del dielettrico mobile sebbene sia l che
si esercita la forza.
32 Capitolo 3. Dielettrici

Esercizio 55: Forza di conduttore su dielettrico


Un dielettrico di base quadrata a ed altezza h  a ha una polarizzazione uniforme P come in figura 3.1b. Viene
appoggiato su di un piano conduttore. Calcolare la forza risentita.

bSoluzione: La polarizzazione genera una densita di carica uniforme + sulla cima, e sulla base. (Per
determinare il segno basta ricordare pol = P = z Pz ). Lo si puo risolvere usando un dielettrico
immagine. Ma la forza dominante e quella generata dalle cariche nella base che inducono una carica + sul
conduttore, ed un campo elettrico E = /0 e quindi una forza attrattiva F = Ea2 /2 = P 2 a2 /20 . Stimiamo
il contributo delle cariche sul tetto approssimandole come una carica puntiforme q = a2 . Introducendo una
carica immagine q essa risente una forza q 2 /h2 40 . Quindi la forza totale vale

P 2 a2 a2
 
F 1+
20 2h2

Il contributo delle cariche in cima e trascurabile.

Esercizio 56: Dielettrico in campo esterno


Un dielettrico con costante dielettrica e immerso in un campo elettrico esterno E ext . Calcolare il campo
elettrico allinterno del dielettrico assumendo che esso abbia forma a) lunga e sottile; b) corta e larga; c) sferica.

bSoluzione:

a) Se il dielettrico e lungo e sottile, la condizione al bordo dominante e E= = 0, e quindi Ein = Eext .

b) Se il dielettrico e corto e largo, la condizione al bordo dominante e D = 0: dentro il dielettrico il campo


elettrico vale Ein = Din /kappa = Dout / = Eout /.

c) Se il dielettrico e sferico, verra una cosa intermedia ma il conto e piu compicato. Le equazioni da risolvere
sono, dentro il dielettrico:
E tot = E ext + E pol , P = 0 E tot (3.1)

cioe la polarizzazione e proporzionale al campo elettrico totale, che comprende un contributo generato
dalla polarizzazione.
Assumiamo che 1  1: in tal caso la polarizzazione e piccola E ext e E tot saranno quasi uguali, e
quindi in prima approssimazione la polarizzazione vale P ' 0 E ext , ed e quindi costante. Come discusso
a pagina 15) un P costante genera una densita di cariche superficiali = P cos e quindi, allinterno della
sfera, un campo elettrico E pol = P /30 uniforme. Il campo elettrico totale vale

E tot = E ext + E pol ' (1 )E ext
3

c)0 In generale E ext e E tot differiscono in modo significativo. Proviamo a vedere se una polarizzazione
P uniforme risolve il problema impostato in eq. (3.1). La polarizzazione genera un campo elettrico
E pol = P /30 = E ext /3 uniforme: quindi, dentro la sfera si produce un campo uniforme

E ext 3E ext
E tot = E ext + E pol = E ext E tot : E tot = =
3 1 + /3 2+

minore del campo esterno. Per  1 si ritrova lapprossimazione del punto precedente. Per  1 il
dielettrico diventa come un conduttore.

Il campo elettrico totale esterno alla sfera e quello esterno piu quello di un dipolo: il risultato e disegnato in
figura 3.2.
Capitolo 3. Dielettrici 33

Figura 3.2: Linee di campo (continue) ed equipotenziali (tratteggiate) per una sfera dielettrica in un campo
elettrico esterno costante. Le tre figure correspondono a costanti dielettriche = {1, 3, 30}.

Esercizio 57: Buco in dielettrico


Un dielettrico con costante dielettrica out e immerso in un campo elettrico esterno E ext . Il dielettrico contiene
un buco di forma a) lunga; b) corta; c) sferica. Calcolare il campo elettrico dentro il buco.

bSoluzione:

a) Ein = Eext .

b) Ein = Din = Dout = out Eout .

c) Consideriamo il problema generale di un oggetto dielettrico di costante in immerso in un dielettrico


esterno di costante out . Le condizioni al bordo che devono essere soddisfatte sulla superficie della sfera
sono
= =
out Eout = in Ein , Eout = Ein
Cioe conta solo il rapporto = in /out . Le soluzioni ottenute ai punti a) e b) di questo esercizio e di
quello precedente soddisfano a questa proprieta generale.
Per trovare la soluzione basta quindi sostituire 1/out nella soluzione c)0 dellesercizio precedente.

La stessa soluzione e riottenuta tramite un calcolo esplicito nellesercizio successivo.

Esercizio 58: Sfera dielettrica in dielettrico


Un dielettrico di costante dielettrica out contiene un buco sferico di raggio r e costante dielettrica in . Si studi
il sistema in presenza di un campo elettrico esterno Eext .

bSoluzione: Proviamo a trovare una soluzione assumendo che il campo interno sia Etot = Ein costante, e che
il campo esterno sia Etot = Eout = Eext + campo generato da un dipolo P . Ci sono 2 incognite: Etot e P . Le
condizioni al bordo che devono essere soddisfatte sulla superficie della sfera sono
r r
out Eout = in Ein , Eout = Ein

Esplicitamente
kP kP
out (Eext 2 ) cos = in Ein cos , (Eext + ) sin = Ein sin
r3 r3
da cui
3Eext kP out in
Ein = , = Eext
2 + in /out r3 2out + in
34 Capitolo 3. Dielettrici

Esercizio 59: Attrazione fra dielettrici


Perche un pettine attrae pezzettini di carta?

bSoluzione: Storicamente fu una delle prime manifestazioni dellelettricita (electron non e inglese ma greco,
e vuol dire ambra. Anche un pettine di plastica va benissimo). Il fatto che la carta attratta rimanga poi
appiccicata indica che centrano i dielettrici. Se fossero invece cariche libere si neutralizzerebbero appena si
toccano.
Un dielettrico in un campo uniforme non sente nessuna forza. Questo e ovvio per geometrie semplici (e.g.
cubo o cilindro orientato lungo il campo), e stato verficato in precedenza nel caso di una sfera, ed in generale e
dovuto al fatto che il dielettrico si polarizza lungo il campo. Un dipolo in campo elettrico costante non sente
forze.
Un dielettrico in un campo elettrico non uniforme viene attratto verso campi grossi: lo abbiamo visto nel
caso particolare del condensatore piano, dove erano gli effetti ai bordi a generare la forza attrattiva E 2 . La
seconda potenza non e specifica di questa geometria semplice, ed e dovuta al fatto che F = pol E e che la carica
di polarizzazione e a sua volta E.
In generale il dielettrico viene attratto verso campi grossi da una forza che e complicato calcolare in dettaglio.
Ma gli argomenti precedenti consentono di dire che in generale la forza e del tipo F E 2 .
Un pettine sfrutta leffetto delle punte per generare un campo elettrico abbastanza grosso ed abbastanza
dipendente dalla posizione in modo da generare una forza abbastanza grande da attrarre pezzetti di carta.
Capitolo 4

Correnti

E = j dove e la resistivita e = 1/ viene chiamata conducibilita. Per effetto Joule viene dissipata una
potenza W = j E = j 2 . E utile introdurre la corrente totale I e definire la resistenza R in modo che V = IR.
Compiti rilevanti: Compitino del 19 dicembre 2003 es. 1. Compitino del 17 gennaio 2003, es. 3.

Esercizio 60: Capacitatore piano imperfetto


Calcolare il tempo di scarica di un condensatore piano (area A, distanza tra i piatti d) contenente un materiale
di conducibilita e costante dielettrica .

bSoluzione: Attenzione: qui non indica la densita superficiale di cariche. Si puo ragionare in due modi:
1. Usando le equazioni fondamentali. Il campo elettrico E = Q/A genera una corrente J = E e quindi

Q = AJ = Q risolta da Q(t) = Q(0)et/ dove = .


2. Usando le formule valide per circuiti. La capacita vale C = A/d, la resistenza R = d/A. Quindi
= RC = /.
Notare che non dipende da A e d, cioe da quanto e grosso il condensatore. Questo rende piu semplice il
funzionamento delle cellule: il tempo di scarica non varia quando la membrana diventa piu spessa o grossa.

Esercizio 61: Scarica di sfera carica


Calcolare il tempo di scarica di una sfera di raggio a in un dielettrico di conducibilita e costante dielettrica .

bSoluzione: Esce una corrente radiale. Siccome ha divergenza zero le cariche flusicono verso distanza infinita
senza accumularsi. Facendo il calcolo direttamente ottengo:

flusso di J flusso di E
dQ z }| { z }| {
= 4r2 Jr = 4r2 Er = Q
dt 
e quindi Q(t) = Q(0)et/ con = /. Per t il materiale si e comportato come un conduttore, per
t  = / come un dielettrico.
Posso riscrivere la corrente i = Q come i = V /R dove V = Q/4a e la differenza di potenziale e R = 1/4a.
Avevamo visto che una sfera ha una capacita C = 4a. Quindi possiamo schematizzare il sistema come un
circuito (chiuso allinfinito) con costante tempo = RC = /.

Esercizio 62: Resistenza fra sfere concentriche


Calcolare la resistenza fra due sfere concentriche di raggi a e b in un materiale di resistivita .

35
36 Capitolo 4. Correnti

bSoluzione: Posso calcolarla indirettamente interpretando il sistema come una serie infinita di resistenze:
come visto nellesercizio precedente la resistenza di un guscio di spessore dr vale dR = dr/4r2 . Sommando
le resistenza di tutti i gusci in serie viene
Z
1 1
R = dR = ( )
4 a b
Se a  b conta solo il primo termine: lintegrale e dominato dalla zona vicino alla sfera piccola.

Esercizio 63: Sonda marina


Due sfere di raggi a vengono calate in mare a distanza d e connesse da un filo conduttore. Calcolare la resistenza
del circuito.

bSoluzione: Domina la zona vicino alle sfere (o attorno alla sfera piccola, se avessero dimensioni diverse):
quindi lelettrostatica consente e.g. di misurare localmente la salinita del mare. Se passa un branco di pesci fra
le sfere non me ne accorgo. Per pescare servira lelettrodinamica.
Si puo schematizzare il sistema come due resistenze R = /4a in serie. Per a = 25 cm e = 25 ohm cm
viene 2R = 0.27 ohm.

Esercizio 64: Semipiano dielettrico imperfetto


it
Un dipolo oscillante p = p0 e viene posto nel vuoto a distanza d da un semispazio x < 0 di costante dielettrica
 e conducibilita .

bSoluzione: In analogia allesercizio a pagina 30, provo una soluzioni con dipoli immagini

(p a x = d)+(p0 a x = d) per x > 0, zona 2



E=
(p00 a x = d) per x < 0, zona 1

Questo soddisfa le equazioni

D = free E =0 J = free J = E D = E

per x < 0 e x > 0, dove lunica carica e il dipolo p. Sul bordo (chiamdo w la densita di carica)
1 2 2 1 1 2 1 1
E= = E= , E E = w/0 w = E : E E = 1 E /0

cioe
00
p p0 = p00 (p + p0 p00 ) = p
0
Eliminando p00 trovo una equazione per p0

/0 1
p0 = (p p0 ) p
1+ 1+

Assumendo p0 = p00 eit (dopo un transiente) trovo

1 i0 (1 )/
p0 = p
1 + i0 (1 + )/

Un dipolo ruotante e descritto da p (1, i, 0).

Esercizio 65: Diodo termoionico


Gli elettroni escono dal catodo con velocita nulla a V0 = 0. Calcolare come la corrente dipende dalla differenza
di potenziale V .
Capitolo 4. Correnti 37

bSoluzione: Gli elettroni acquistano velocita m 2


2 v = eV (x) e generano una densita di corrente J = v. Se J e
piccola e non modifica V , abbiamo finito. A basso V la corrente e limitata dalla densita di carica degli elettroni.
Lintasamento massimo si ha quando gli elettroni schermano completamente il campo elettrico esterno dando
E = 0 al catodo (se E < 0 nulla esce e gli elettroni fuori vengono spazzati via). Utilizzando lequazione di
Poisson /0 = V 00 si ottiene r
00 2e
J = v = V 0 V
m
In condizioni stazionarie J e costante in x. Ottengo una equazione differenziale per V :

d V 02
 
00 j
V = : 2j V = 0
V dx 2
p
(j = J m/2e/0 ). La costante di integrazione vale zero in quanto V 0 = Ecatodo =. Integrando ancora
Z V Z x
0
p 1/4
p 4 3/4 p
V = 2 jV 1/4 : dV V = 2 j dx : V = 2 jd
0 0 3

ed, inserendo il valore di j,


3d2 J
r
3/2 m
V =
20 2e
cioe J V 3/2 : non segue la legge di Ohm.
Calcoli simili permettono di studiare giunzioni fra semiconduttori, usati per costruire diodi piu moderni.

Esercizio 66: Piatto dielettrico


Un piatto di dielettrico con costante dielettrica  e resistivita viene messo in un campo elettrico esterno Eext ,
che viene improvvisamente rimosso. Studiare cosa succede. Stessa domanda per un campo elettrico esterno
Eext = E0 eit lentamente variabile.

bSoluzione:
Dopo un podi tempo si forma una densita di carica = 0 Eext sul bordo destro e su quello sinistro.
Subito dopo che il campo Eext e stato rimosso rimane la che genera un campo E = Eext allinterno
del conduttore e quindi una corrente j = E/ che inizia a riequilibrare le cariche. Siccome E = 0
non si generano cariche di volume. Da = j, tenendo che j = 0 fuori dal conduttore segue che
= +j = /0 da cui (t) = (0)et/ con = . In questo primo problema il segno giusto e fissato
da ovvie considerazioni fisiche.
Come prima = 0 dentro il conduttore, e si accumulano cariche ai due bordi, lasciando un campo
elettrico interno Ein . Le condizioni di raccordo sono

0 Eext = Ein + = j = Ein / : + = 0 Eext

In condizioni stazionarie la soluzione e


0 E0 0 E0 1
= 0 eit con 0 = e quindi Ein =
1 + i  1 + 1/i

Se = 0 Ein = 0 (conduttore perfetto). Se  1 Ein = E0 / (dielettrico perfetto). Per valori intermedi


si ha un campo con modulo intermedio che oscilla con ritardo di fase = arctan . Il tutto corrisponde
a modificare il conto statico usando una costante dielettrica complessa = + 1/i0 .

Esercizio 67: Sfera dielettrica


Un sfera di raggio r composta da un materiale con costante dielettrica  e resistivita viene messa in un campo
elettrico esterno Eext = Eext Re (1, i, 0)eit lentamente ruotante.
38 Capitolo 4. Correnti

a) Mostrare che la carica libera sta solo sul bordo.


b) Scrivere le condizioni di raccordo.
c) Verificare che la sfera acquista una polarizzazione P (t) uniforme ruotante con ritardo di fase.
d) Trovare il momento delle forze M , ed il valore di per il quale M e massimo.

bSoluzione:
a) Dentro il dielettrico J D. Prendendo la divergenza segue : se allinizio = 0 la carica fluisce
senza accumularsi.
b) In generale (con fuori il vuoto)

E=in = E=out , Eout = Ein + /0 = Ein /

Assumendo un campo Ein costante ed un campo Eout = Eext + (campo generato da un dipolo P )

kP kP Ein
(Ein Eext ) sin = 0 (Ein Eext + 2 ) cos = = cos
a3 r 3 0

c) Assumendo condizioni stazionarie P = P (1, i, 0)eit , derivando la prima equazione e sostituendo la


riscrivo in forma analoga allequazione statica

P 1
(Ein Eext 2 ) cos = 0 +
r3 i0
Quindi la soluzione e analoga a quella ottenuta nel caso statico

k P 1 1
= Eext = Eext
r3 2 + 3 + i
avendo messo per semplicita = 1. [Il segno non viene]
d) M = Re P Re E massima per = 1 (a meno della riduzione nel modulo di P ) (nel sistema Terra-Luna
le maree avvengono con ritardo di fase provocando un trasferimento di momento angolare).
Capitolo 5

Circuiti

Risolvere le equazioni di Maxwell e complicato. E facile ottenere soluzioni approssimate che descrivono alcune
situazioni di interesse pratico (circuiti).

Esercizio 68: Resistenze in parallelo


Verificare che la corrente si ripartisce minimizzando la dissipazione per effetto Joule.

bSoluzione: Lenegia dissipata vale W = R1 I12 + R2 I22 . Chiamando I la corrente totale sia ha I2 = I I1 .
Quindi
dW R1 R2
= 2R1 I1 + 2R2 (I1 I) = 0 quindi R1 I1 = I = R= I
dI1 R1 + R 2
Che sia un minimo e non un massimo, lo si puo vedere nel caso R2  R1 : tutta la corrente flusice nella resistenza
piccola minimizzando leffetto Joule.

Esercizio 69: Resistenze su cubo


Calcolare la resistenza totale del circuito in fig. 5.1a, assumento che le singole resitenze abbiano un valore
comune R.

bSoluzione: E un esempio di circuito non decomponibile come combinazioni di serie e paralleli. Lo si potrebbe
risolvere scrivendo le equazioni di Kirchoff, pero si fa prima a dare la risposta ad occhio sfruttando la simmetria
del problema. Per motivi di simmetria le correnti si dividono come in figura. Quindi la differenza di potenziale
fra i due capi vale V = IR( 13 + 16 + 13 ) = I 56 R.
Se invece uno collegasse due spigoli opposti sulla stessa faccia del cubo, non passerebbe corrente nelle due
linee verticali degli altri due spigoli opposti...

Esercizio 70: Ponte di Wheatstone


Risolvere il circuito in fig. 5.1b.

bSoluzione: E un esempio di circuito non decomponibile: cioe non si puo evitare di applicare le leggi di
Kirchoff vedendolo come combinazioni di serie e paralleli. In pratica lo si usa con: R3,4 resistenze fisse note,
R2 resistenza ignota da misurare, R1 resistenza variabile nota, da scegliere in modo tale che la differenza di
potenziale VBD = 0, cioe che non passi corrente attraverso R.
Iniziamo dal caso R = : VBD = 0 (cioe il circuito e bilanciato) se R2 R4 = R1 R3 . In generale
 
R1 R4 R 1 R 3 R2 R4
VDB = V =V
R1 + R 2 R3 + R4 (R1 + R2 )(R3 + R4 )

39
40 Capitolo 5. Circuiti

I/3 B
I/3
R1 R2

A R C
I/6
R4 R3
D
I/3

Figura 5.1: (a) Circuito a cubo. (b) Ponte di Wheatstone.

In condizioni di bilanciamento R2 /R1 = R3 /R4 = r. Per studiare la sensitivita dello strumento (e quindi
calcolare lincertezza sperimentale su R2 ) consideriamo una variazione delle resistenze Ri Ri + Ri : essa
produce  
r R1 R2 R3 R4
VDB = 0 + + V.
(1 + r)2 R1 R2 R3 R4
Passiamo al caso di R finito. In pratica R compare perche un qualunque strumento che misura VDB lo fa
introducendo una resistenza R < fra i capi B e D. Risolviamo quindi il circuito completo utilizzando il
metodo delle maglie: ci sono 7 incognite: VA , VB , VC , VD e le tre correnti di maglia I1 (a sinistra, circolante in
direzione A B), I2 (a destra in direzione C B) ed I0 (sotto, in direzione A D C). Siccome contano
solo le differenze di potenziale si puo scegliere VA = 0, riducendo il numero di incognite a 6. Ci sono poi 6
equazioni, una per ogni tratto di circuito:
VB VA = R1 I1 , VB VC = R2 I1 , V C VA = 0
VD VA = R4 (I0 I1 ) VD VC = R3 (I0 + I2 ) VD VB = R(I1 + I2 ).
Dopo calcoli noiosi si ottiene
(R1 R3 R2 R4 )V
VDB = .
(R1 + R2 )(R3 + R4 ) + (R1 R2 R3 + R1 R2 R4 + R1 R3 R4 + R2 R3 R4 )/R
Vediamo quindi che per calcolare il valore centrale di R2 non serve tenere conto di R, che invece modifica la
sensibilita dello strumento.

Esercizio 71: Impedenze


1
Mostrare che per una corrente oscillante I = I0 eit vale ZR = R ZC = ZL = iL.
iC
bSoluzione: Le cadute di potenziale ai capi di resistenza, condensatore, induttanza valgono
Q I
RI = LI = iLI
C iC

Esercizio 72: Potenza dissipata


Calcolare la potenza dissipata su di una generica impedenza Z.

bSoluzione: La potenza dissipata vale W = V I dove V = ZI. Il prodotto non e una operazione lineare:
Re(z1 z2 ) 6= Re(z1 )Re(z2 ). Bisogna tornare ai numeri reali: scrivendo Z = R + iY
R 2 Z cos 2
hW i = h(I0 R cos t I0 Y sin t) (I0 cos t)i = I0 = I0
2 2
cioe solo la parte resistiva dissipa energia.
Capitolo 5. Circuiti 41

Esercizio 73: Filtro che taglia frequenze alte


Costruirlo.

bSoluzione: Ad esempio metto RC in serie e leggo il voltaggio ai capi di C: a grosso la sua impedenza
Z = 1/C decresce e la maggior parte della caduta di potenziale avviene su R.
VC 2

VC 1 1
= r = =
Vin 1 + iRC Vin 1 + (RC)2
Per R = 200 k si ha una riduzione di 3 dB a = 1 Hz (r = 100.3 = 0.512 1/2) se C = 0.8 F, che a = 50
Hz corrisponde a r = 0.4 103 i.e. 34 dB.

Esercizio 74: Filtro che taglia frequenze basse


Costruirlo.

bSoluzione: Metto RC in serie e leggo il voltaggio ai capi di R, oppure metto RL e leggo ai capi di L.

Esercizio 75: Pendolo accoppiato


Mostrare che due pendoli connessi da una molla k soddisfano alle stesse equazioni di due maglie LC con in
comune una capacita C 0 .

bSoluzione: Usando le correnti di maglia I1 e I2 le equazioni sono


Q1 Q1 Q2 Q2 Q2 Q1
V1 = LQ1 + + 0 = LQ2 + +
C C0 C C0
Usando le impedenze e risolvendo si trova
iV1 C 2 V1 22 12 I2 C
I2 = = i =
(CL 2 1)(CC 0 L 2 2C C 0 ) 2L ( 2 12 )( 2 22 ) I1 C + C CC 0 L 2
0

Avendo definito 12 = 1/LC e 22 = 1/LC + 2/LC 0 . Alla media delle frequenze I1 = 0.


Le equazioni del moto del pendolo sono
x1 x2
mx1 + mg + k(x1 x2 ) = F mx2 + mg + k(x2 x1 ) = 0
` `
che hanno la stessa forma con 1/LC g/`, 1/LC 0 = k/m.
Per trovare i modi normali provo una soluzione xi = xi eit
  
2 + g/` + k/m k/m x1
=0
k/m 2 + g/` + k/m x2
Il determinante vale zero per
1. 2 = 12 = g/`, x1 = x2
2. 2 = 22 = g/` + 2k/m, x1 = x2
Mettendo F (t) = F eit viene
F F 22 12 x2 k` 12 22
x2 = = = =
k (k m 2 + gm/`)2 /k 2m ( 2 12 )( 2 22 ) x1 k` + mg m` 2 2 2 12 22
che corrispondono alle soluzioni per i modi normali
F 1 F 1
x1 + x2 = x1 x2 =
m 2 12 m 2 22
Nel caso del circuito, il modo normale I1 = I2 (niente corrente su C 0 ) vede unimpedenza iL + 1/iC che vale
zero per = 1 ; il modo I1 = I2 vede unimpedenza 21 (iL + 1/iC) + (1/iC 0 ) che vale zero per = 2 .
42 Capitolo 5. Circuiti

R1 R1 R1 R1 R1 R1

R2 R2 R2 R2 R2 R2 R = R

Figura 5.2: Fig. 5.2a: circuito attenuatore. Fig. 5.2b: resistenza equivalente.

Esercizio 76: Attenuatore


Si determini la resistenza totale R del circuito infinito in fig. 5.2 ed i potenziali nei vari punti

bSoluzione: Siccome = + 1 per trovare la resistenza R equivalente al circuito si impone che R sia eguale
ad un passo della catena seguito da R:
p
1 R1 + R12 + 4R1 R2
R = R1 + : R= (5.1)
1/R2 + 1/R 2

Per trovare il potenziale dopo il primo passo, rimpiazziamo tutte le resistenze successive con la resistenza
equivalente R, ottenendo un circuito con 3 resistenze R1 , R2 ed R, di resistenza totale R: circola una corrente
totale I = V /R
La differenza di potenziale V 0 ai capi di R vale
R2
V0 =V
R + R2

Nei passi successivi la differenza di potenziale decresce in modo geometrico: V (n) = V /(1+R/R2 )n . Ad esempio,
per dimezzare la differenza di potenziale ad ogni passo serve R = R2 , cioe R2 = 2R1 .
In pratica non si puo costruire una catena con un numero infinito di passi: per terminare il circuito dopo
un numero finito di passi senza scompensarlo, basta terminarlo con una resistenza R.

Esercizio 77: Catena LC


Come nella fig. 5.2, con R1 Z1 = iL e Z2 = 1/iC.

bSoluzione: Pre semplificare la formula (5.1) divido ogni L in 12 L + 12 L e metto L/2 a sinistra:
 p
Z1 2
p(L/C) (L/2) se < 0
p
Zeff = Z = (Z1 /2)2 + Z1 Z2 = 2
2 i (L/2) (L/C) se > 0

dove 0 = 2/ LC. La cosa sorprendente e che a basse frequenze un circuito con solo L e C si comporti come
una resistenza; il motivo fisico e che lenergia sembra scomparire in quanto viene trasmessa attraverso la catena.
Abbiamo gia visto che il potenziale varia lungo la catena come
p
(L/C) (L/2)2 i(L/2)
= ei se < 0


p
Z 2 Z Z 1

(L/C) (L/2) 2 + i(L/2)
Vn = n V0 = = = p
Z + Z2 Z (L/2)2 (L/C) (L/2)


p <1 se > 0
(L/2)2 (L/C) + (L/2)
p
A bassa frequenza tutto funziona se si termina la catena finita con R = L/C. (Se si scambia L C e come
cambiare 1/: la catena risultante taglia le frequenze basse invece di quelle alte).
Se le differenze di potenziale e di corrente fra due elementi vicini sono piccole, valgono

V = LI I = Q = C V
Capitolo 5. Circuiti 43

cioe
dV L dI dI C dV d2 V L C d2 V
= = : 2
= 2
dx x dt dx x dt dx |x{zx} dt
v2

La rete LC fornisce una descrizione approssimata di una linea di trasmissione. Ad esempio per un cavo coassiale
0 20 1 R2
L = x ` C = x v= =c (` ln ).
2 ` 0 0 R1

Per x 0 0 : una linea ideale trasmette tutto, ed il cavo equivale ad una resistenza
r
0 `
R = lim Z = = 60 `.
x0 0 2
Parte II

Magnetostatica
Capitolo 6

Campi magnetici

Le equazioni di base sono F = q(E + v B) e le equazioni di Maxwell (0 = 4km , km = 107 Tesla m/A)

B = 0 j B =0

che implicano j = 0. In forma integrale


I
B ds = 0 (j) = 0 i (B) = 0

Campo magnetico generato I


0 r 0 r
B= qv 3 = i ds
4 r 4 r3
Forza prodotta da un campo magnetico: F = qv B = j B = i ds B. Un circuito chiuso in un campo
magnetico costante percorso da una corrente costante i sente F = 0 e momento M = m B dove m = iSn
Lenergia potenziale vale U = m B = i(B). Se il campo magnetico non e costante F ' (m B).

Unita di misura. Il campo magnetico viene misurato in Tesla = N/Am (Gauss = 104 Tesla). Il campo
magnetico terrestre sulla superficie vale circa 0.1 Gauss. Il massimo campo producibile e circa 10 Tesla. Il flusso
di B viene misurato in Weber = Tesla m2 = Volt sec.

Esercizio 78: Forza fra 2 cariche


Due elettroni si muovono parallelamente lungo traiettorie rettilinee a distanza a con velocita costante v  c.
Calcolare la forza elettromagnetica

bSoluzione: La forza e diretta lungo la congiungente. La forza elettrica respinge e quella magnetica attira
e2 0 e2 v2 1
F = ev ev = (1 2 ) c2 =
40 4 40 c 0 0
Il risultato sopra e sbagliato. Esistono altri effetti relativistici di O(v 2 /c2 ). Nel sistema in cui le cariche sono
in quiete F0 = e2 /40 e quindi la relativita dice che Fv = F0 /, invece che Fv = F0 / 2 Infatti i campi E e B
generati da cariche in moto vanno moltiplicati per .
Una carica puo avere velocita media cm/s 1010 c e quindi il suo campo magnetico e una correzione di
ordine 1020 . Questa soppressione puo venire compensata se ci sono NA 6 1023 cariche che si muovono nello
stesso verso (formando una corrente), messe assieme ad altrettante cariche di segno opposto (in modo che i loro
campi elettrici si cancellano). Siccome la materia e fatta in questo modo, ha senso studiare la magnetostatica.

Esercizio 79: Disco di Rowland


Un disco di raggio r = 20 cm con carica = 106 C/ m2 fa 200 giri al secondo. Stimare il campo magnetico
generato

45
46 Capitolo 6. Campi magnetici

bSoluzione: Quindi la corrente vale I 200 Q/ sec 105 A e, ad una distanza di 1 cm, genera un campo
magnetico
km I
B 1010 T
cm
(km = 0 /4 = 107 ) 105 volte minore del campo magnetico terrestre (circa costante attorno al disco). Il
grosso campo elettrico E /0 105 V/m (0 1011 C/Vm) genera una forza repulsiva di circa 1 N e veniva
schermato con un conduttore. Sui condensatori si puo accumulare Q C ma...??

Esercizio 80: Attrazione o repulsione?


Perche due cariche uguali si respingono, mentre due fili con correnti uguali si attraggono?

bSoluzione: Non esiste un modo semplice di vederlo sapendo solamente che u = 0 E 2 /2 + B 2 /20 . Invece di
cariche e fili consideriamo piani che rendono la geometria piu semplice.

La pressione elettrostatica su di un piano con densita di carica superficiale = 0 (E2 E1 ) vale

E1 + E2 0 2 0 2
p = = ( E ) = uE uE = E
2 2 2
avendo usato il fatto che E= e continuo. Per determinare il segno pensiamo a due piani xy paralleli con
densita di cariche : essi si attraggono, tendendo a minimizzare lenergia uE .

La pressione magnetostatica su di un piano con densita di corrente superficiale = (B=2 B=1 )/0 vale

B1= + B2= B2 B2
p = = ( = ) = uM uM =
2 20 20

avendo usato il fatto che B e continuo. Per determinare il segno pensiamo a due piani xy paralleli con
densita di corrente x : essi danno luogo ad un campo magnetico By nella zona interna, ma si respingono.

Quindi il segno e opposto nei due casi. La pressione e data non da u dove u = uE + uM ma da L dove
L = uE uM . La notazione L indica che questa sara la Lagrangiana Lorentz-invariante dellelettromagnetismo:
e noto che L = K V mentre H = K + V . La ragione fisica del ribaltamento del segno nel caso magnetico e
che mantenere correnti costanti costa energia, mentre le cariche rimangono automaticamente costanti.

Esercizio 81: Filo rettilineo


Calcolare il campo magnetico generato da un filo rettilineo di raggio a percorso da una densita di corrente
costante j.

bSoluzione: B ha solo una componente radiale. Fuori dal filo (r > a) la legge di Ampere fornisce 2r Br = 0 i
dove i = j a2 . Dentro il filo (r < a) i = jr2 e quindi Br = 0 jr/2 = 0 ir/2a2 .

Esercizio 82: Cavo coassiale


Calcolare il campo magnetico generato da un filo rettilineo di raggio a percorso da una densita di corrente
costante j, circondato da un cilindro di raggi b e b0 > b lungo cui scorre uniformemente una corrente totale
opposta.

bSoluzione: Per r < b e come nellesercizio precedente. Per r > b0 si ha B = 0. Per b < r < b0 si ha

r 2 b2
 
0 j 2
Br = a 1 02
2r b b2
Capitolo 6. Campi magnetici 47

Esercizio 83: Spira circolare


Calcolare il campo magnetico generato da una spira circolare di raggio a nel piano xy percorso da una corrente
i.

bSoluzione: Lungo lasse e facile integrare: B ha solo una componente lungo z

0 i 0 ia 0 ia2
Bz = 2a cos = cos =
4 r2 2 r2 2 (a2 + x2 )3/2

ds ed r sono ortogonali, e ds r forma un angolo con lasse z. Nel centro = 0 e B = 0 i/2a.


Quindi linduttanza della spira vale circa L 0 a. E complicato fare un calcolo preciso, che dipende dallo
spessore del filo.

Esercizio 84: Due spire circolari


Calcolare il campo magnetico generato da due spira circolare di raggio a a distanza d nel piano xy percorse da
una corrente N i.

bSoluzione:
0 N ia2 h 2 i
Bz = (a + x2 )3/2 + (a2 + (d x)2 )3/2
2
Il campo magnetico e molto uniforme fra le due spire quando d = a (spire di Helmholtz): infatti per tale valore
si ha d2 Bz /dx2 = 0 a x = d/2 e Bz (x = d/2) = (4/5)3/2 0 N i/a.
Assumiamo che il campo magnetico sia zero fuori e costante dentro. Un elettrone entra camminando lungo
lasse. Dentro percorre un arco di circonferenza con raggio di curvatura r = mv/eB e quindi viene deflesso di
sin = a/r. p p
Se deve essere = 45 per v = 2eV /m con V = 25 kV e i = 2 A servono n = (1.4/0 i) mV /e 200
avvolgimenti per spira.

Esercizio 85: Filo a U


Calcolare il campo magnetico nel centro del semicerchio generato da un filo percorso da una corrente i che forma
una U di raggio a.

bSoluzione: Raddoppiando i due mezzi fili ed il semicerchio, Bz e la meta del Bz prodotto da 2 fili rettilinei
infiniti e da un cerchio:
2 0 i 1 0 i
Bz = +
2 2a 2 2a
Notare che la configurazione di fili riflessa produce lo stesso B, in quanto esso e uno vettore assiale, non un
vettore.

Esercizio 86: Piano a U


Come prima, con il filo rimpiazzato da un piano.

bSoluzione: Nel centro equivale alla meta di due piani e di un solenoide. La cosa interessante e che il campo
magnetico e costante nella regione interna.
48 Capitolo 6. Campi magnetici

Esercizio 87: Solenoide rettilineo infinito


Calcolare il campo magnetico generato dentro un solenoide rettilineo infinito.

bSoluzione: Fuori vale zero, dentro e costante orientato come il solenoide. La legge di Ampere fornisce
B = 0 ni dove n e il numero di spire per unita di lunghezza.
Il flusso concatenato ad un solenoide abbastanza lungo da poter trascurare effetti ai bordi vale = BN S =
LI (dove N = n` e il numero di spire e S la loro area) con L = 0 SN 2 /`. Quindi lenergia necessaria per
accendere una corrente i vale U = LI 2 /2 (dimostrazione: V = = LI. Quindi la potenza vale W = V I =
1
LI I = d( 2 LI )/dt). La si puo calcolare anche come lintegrale della densita di energia u = B 2 /20 sul volume:
2

U = (0 nI)2 S`/20 .

Esercizio 88: Solenoide rettilineo finito


Dire qualcosa sul campo magnetico generato dentro un solenoide rettilineo semi-infinito.

bSoluzione: Usando la forza bruta, integro il campo di una spira, mettendo n spire per unita di lunghezza da
` a 0: Z 0
ia2
 
0 0 0 ni `+x x
Bz (x) = n dx =
2 (a2 + (x x0 )2 )3/2
p
` 2 a2 + (` + x)2 a2 + x2
| {z }
1 per `
Il principio di sovrapposizione consente di dire che: 1) Sul bordo B vale la meta che in un punto interno, come
vedo immaginando di completare aggiungendo laltra meta del solenoide. In generale Bz (x) + Bz (x) = Bz . 2)
La linea di campo di B che sfiora il bordo esce perpendicolare al solenoide, perche quando completo la somma
deve fare zero; le componenti orizzontali si sottraggono ma le eventuali componenti verticali si sommerebbero
(B e uno pseudovettore) e quindi devono valere zero. Chiaro
con un disegno. 3) La linea di campo che sfiora il
bordo, dentro il solenoide sta ad una distanza limite a/ 2 dal centro (il raggio che contiene meta del flusso),
con relazioni analoghe per le linee interne.

Esercizio 89: Solenoide toroidale


Calcolare il campo magnetico generato dentro un solenoide toroidale.

bSoluzione: B e circolare in quanto spire disposte simmetricamente rispetto al punto dove viene calcolato B
danno contributi che si cancellano alle componenti non radiali. Quindi, per la legge di Ampere, fuori B = 0 e
dentro 2rBr = 0 N i dove N e il numero totale di spire.

Esercizio 90: Sfera ruotante


Sulla superficie di una sfera omogenea di massa M e raggio R e distribuita uniformemente una carica Q. La
sfera ruota con velocita angolare . Calcolare il momento magnetico. Scrivere lequazione del moto in presenza
di un campo magnetico uniforme B.
Capitolo 6. Campi magnetici 49

bSoluzione: Un anello fra e + d ha raggio r = R sin superficie dQ = Q dS/S con dS = 2r R d e


ruota con velocita v = r. Quindi trasporta una corrente
dQ v Q
di = = sin d
2r 4
Il momento magnetico vale

QR2 QR2
Z Z
= 2
r di = sin3 d =
4 0 3
o anche
Q 5 2M R2
=g L dove g= e L=
2M 3 5
e il momento angolare. Una sfera uniformemente carica avrebbe g = 1. Elettrone e muone hanno ge =
2.002319304374.. e g = 2.0023318406...
Capitolo 7

Moto in campo magnetico esterno

Esercizio 91: Trottola magnetica


Trovare un modo di sospendere un dipolo magnetico in aria.

bSoluzione: La forza di un dipolo in un campo magnetico esterno e data da unequazione analoga al caso di
un dipolo elettrico: U = B e F = U , M = B.
Discussione preliminare. Avevamo visto che F = (pE) non funzionava per un dipolo elettrico orientato
lungo z in un Ez x (perche tale E ha rotore diverso da zero, e quindi non puo esistere in elettrostatica):
pensando il dipolo come 2 cariche e facile vedere che la forza deve essere zero. Nel caso magnetico (dove B puo
avere rotore diverso da zero) la formula funziona: la forza e prodotta dal fatto che il lato della spira dove Bz e
piu grosso sente una forza maggiore.
Applicata invece ad un dipolo orientato lungo B, e con un Bz (z), la formula dice che il dipolo e attratto
verso zone dove B e maggiore. Pensando al dipolo come ad una spira, e immediato vedere che non ce nessuna
forza. La formula sballa perche non esiste un campo magnetico con solo Bz (z): per avere B = 0 serve anche
un Br (r, z). A livello grafico, il campo magnetico deve incurvarsi verso lesterno. E la componente Br che
genera la forza, anche se non appare in U .
Unapplicazione e la trottola magnetica. Se si mette un dipolo sopra il campo magnetico generato da una
spira, questo si allinea con la spira e ne viene attratto. Quindi non rimane sospeso.
Se invece l oggetto con dipolo magnetico (e.g. la sfera dellesercizio precedente) viene anche fatto girare,
il possedere un momento angolare L gli impedisce di allinearsi a B. Esiste un momento delle forze che
produce la seguente equazione del moto:
dL Q
=B =g LB
dt 2M
che dice che L precede attorno al campo magnetico con frequenza di precessione p = g QB/2M , mantenendo in
media temporale la sua orientazione iniziale. Quindi se allinizio viene messo anti-parallelo a B il magnetismo
genera una forza repulsiva che puo tenere sospeso loggetto. Alla lunga lattrito rallenta la rotazione, e loggetto
si allinea e casca. E grazie allattrito che una bussola si allinea al campo magnetico terrestre.

Esercizio 92: Cilindro su piano inclinato


Un cilindro di raggio r e lunghezza `, appoggiato orizzontalmente su di un piano inclinato (di angolo ), e
percorso da una corrente i uniforme lungo la sua lunghezza. E presente un campo magnetico verticale. Per
quale valore di i il cilindro rimane fermo se a) lattrito tra piano e cilindro e trascurabile b) lattrito e tale che
il cilindro puo rotolare senza strisciare.

bSoluzione: Il campo magnetico produce una forza orizzontale, diretta verso il piano inclinato. La corrente in
un angolo d produce dF = `B di = `Bi d/2.

a) La componente della forza magnetica totale lungo il piano inclinato vale F = `Bi cos . La componente
della forza gravitazionale lungo il piano inclinato vale F = mg cos . Sono uguali se i = mg tan /`B.

50
Capitolo 7. Moto in campo magnetico esterno 51

b) Il momento rispetto allasse di appoggio deve essere zero. La gravita produce M = mgr sin . Il campo
magnetico produce
Z Z 2
d
M = y dF = r(cos + sin )`Bi = i`Br cos
0 2
Quindi serve la stessa i di prima (le due forze sono uniformi).

Esercizio 93: Ago magnetico


Una ago di momento magnetico e situato sopra la congiungente due fili paralleli orizzontali a distanza d.
I due fili sono percorsi da correnti i e i. La distanza fra ago e ciascun filo e r. Lago puo ruotare in un
piano ortogonali ai fili, con momento di inerzia I. Calcolare la posizione di equilibrio ed il periodo delle piccole
oscillazioni.

bSoluzione: Il campo magnetico e verticale


0 i h
Bz = 2
2r d/2

Il momento delle forze vale M = B e lequazione del moto e I = B sin (la posizione
p di equilibrio e
= 0 o = /2 a seconda del segno di B, il dipolo vuole allinearsi con B) da cui T = 2 I/|B|.

Esercizio 94: Carica in B costante


Studiare il moto di una particella di carica q in un campo B costante

bSoluzione: Risolvendo lequazione del moto


qB
mv = qv B v = v B B =
m
si trova che v ha una componente costante v = parallela a B, ed una componente v ortogonale a B che ruota
con frequenza B . Quindi fa una spirale. Chiamando a il raggio dellorbita, si ha v = aB i.e.
a q B
p = aqB = MeV 3 104
cm e Gauss


Nei problemi successivi studieremo moti in campi magnetici piu generali, che tendono a dare moti complicati:
le particelle spiraleggiano attorno a qualche traiettoria media. La cosa interessante da calcolare e la traiettoria
media, cioe la velocita di drift. Da questo punto di vista il moto in un campo magnetico costante e quindi
semplicemente un moto a velocita costante lungo le linee del campo, a meno di girellamenti.

Esercizio 95: Campo magnetico galattico


Stimare quando e grande, assumendo che lenergia cinetica in particelle sia comparabile allenergia in campo
magnetico.

bSoluzione: La galassia contiene, in media, un atomo di idrogeno per cm3 con velocita v km/sec.

mv 2 erg B2
1014
2 cm2 20

da cui B 1010 Tesla = 106 G. Essendo debole viene un raggio r = p/eB grosso (scritta in termini di
p = mv la formula rimane valida anche per particelle relativistiche, per le quali p = mv), che ha comunque
52 Capitolo 7. Moto in campo magnetico esterno

effetti importanti perche anche la galassia ha una grossa dimensione d Mpc: un protone di impulso p viene
deflesso
d deB d B 3 1019 eV
= ' 9
r p 1 Mpc 10 G E
Quindi si puo (o meglio potra) fare astronomia solo con i pochi protoni che arrivano con energia abbastanza
grande. Gli altri spiraleggiano nella galassia e la loro direzione di arrivo non da nessuna informazione sulla
sorgente che li ha generati.

Esercizio 96: Spettrometro


bSoluzione: Permette di selezionare particelle con dato q/m se uno ha un fascio monoenergetico.

Esercizio 97: Carica in B ed E costanti


Studiare il moto di una particella di carica q in campi E e B costanti.

bSoluzione: In generale possiamo assumere B = (0, 0, Bz ) e E = (0, Ey , Ez ). Lequazione del moto (nel limite
non relativistico), scritta in componenti e

mvx = qvy Bz mvy = q(Ey vx Bz ) mvz = qEz

La soluzione e
Ey qBz
vx = vrot cos(B t) + vy = vrot sin(B t) vz = qEz t (B = ) (7.1)
Bz m

Il moto lungo z (in generale, lungo B) non si mescola con gli altri ed e ovvio. Lungo x, y la carica oltre a
ruotare fa un drift costante lungo x (in generale lungo E B). La rotazione dipende da q, ma il drift no.
(Sub-esercizio: discutere il limite q 0). Il moto globale e illustrato in fig. 7.1a per cariche con segno opposto.
Se particelle cariche entrano in una zona dove i campi elettromagnetici sono costanti ed hanno solo componenti
Ey e Bz , quelle con vx = Ey /Bz viaggiano imperturbate. Sfruttando questo fenomeno si puo ottenere un fascio
monoenergetico (ad esempio utilizzabile nello spettrometro dellesercizio precedente).


Per ottenere in modo facile ed istruttivo la soluzione (7.1) conviene passare ad un nuovo sistema di riferimento
S 0 dove E e B sono paralleli, avendo solo una componente lungo z, tramite una trasformazione Galileiana
vx = vx0 + Ey /Bz (in generale con un boost B E/B 2 ). In tale sistema S 0 la particella gira attorno al campo
magnetico costante.
Teoricamente questo equivale a dire che sotto una trasformazione galileiana v = v 0 + i campi elettromag-
netici trasformano come

E0 = E + B + , B0 = B + (E B = E 0 B 0 )

Queste trasformazioni sono corrette solo allordine piu basso in . Ad ordini superiori il risultato non sarebbe
consistente perche le equazioni di Maxwell sono Lorentz-invarianti mentre quella di Newton e Galileo-invariante.
Einstein modifico F = ma in modo da renderla relativisticamente invariante, permettendo di discutere i termini
di ordine piu alto in , qui indicati con Se non e verificata la condizione E/B  c questi termini mancanti
diventano importanti. Se E/B > c vedremo che si puo invece passare ad un sistema dove B = 0. Fisicamente,
questo corrisponde ad avere un campo elettrico cos forte che B non ce la piu ad incurvare lorbita.

Esercizio 98: Fotomoltiplicatore in B, E


Studiare di quanto viene ridotta lefficienza di un fotomoltiplicatore con E = kV/cm se viene posto in un campo
magnetico B = Tesla a 45 gradi (figura 7.1).
Capitolo 7. Moto in campo magnetico esterno 53

L
z
B
y
x

Figura 7.1: Fig. 7.1a: esempio di moto in E, B costanti di due particelle con carica opposta. Fig. 7.1b:
fotomoltiplicatore in campo magnetico.

bSoluzione: Un fotomoltiplicatore e un aggeggio simile ad un televisore che accelera elettroni (generati da


ionizzazioni , . . . ) facendoli sbattere su di uno schermo in modo da renderli rilevabili. Nei rivelatori di
particelle a volte si mettono campi magnetici, che incurvano le traiettorie di particelle ed anti-particelle in
direzioni opposte e con raggi che dipendono dalla loro massa, in modo da poterle distinguere.
Il campo elettrico di un fotomoltiplicatore e orientato in modo da accelerare gli elettroni verso lo scher-
mo. Ma se ce un campo magnetico gli elettroni non vanno nella direzione desiderata. Capire dove vanno e
unapplicazione dellesercizio precedente. Allinizio gli elettroni hanno velocita zero, quindi nella (7.1) mettiamo
vrot = Ey /Bz . Una particella che parte da (x, y, z) = (0, 0, 0) segue il percorso

Ey Ey eEz 2
x= (tB sin tB ), y= (1 cos tB ), z= t
Bz B Bz B 2me
Numericamente la frequenza B ed il raggio di spiraleggiamento a valgono

eB Ey m vrot
B = = 1.7 1011 Hz, vrot = = 7 104 , a= = 4 m
me Bz s B
cioe a e miscoscopico. Quindi in pratica gli elettroni

(A) ruotano su circonferenze di raggio trascurabile;

(B) driftano lungo x con velocita vrot 2 104 c: e un effetto trascurabile.

(C) accelerano raggiungendo v 0.1c lungo B e non piu lungo E. Questo e leffetto importante.

A causa di (C) una frazione d/L degli elettroni (cioe quelli che vengono ionizzati nella zona in cima a destra del
fotomoltiplicatore in fig. 7.1) vanno a sbattere sulle pareti laterali invece di venir rilevati sullo schermo. Ridurre
il campo magnetico o aumentare quello elettrico non migliora la situazione, fino a quando a  d.

Esercizio 99: Ciclotrone


Mostrare che una particella libera di muoversi in un campo magnetico Bz ed in un campo elettrico oscillante
Re Ex eit = Ex cos t viene accelerata lungo x.

bSoluzione: Se E = 0 la carica gira nel piano xy: riottengo questa cosa nota usando un primo trucco
matematico. Quando ce roba che gira e utile introdurre z x + iy (attenzione: questo z non ha niente a che
fare con lasse z) in quanto girare nel piano xy con frequenza e raggio r viene descritto in modo piu compatto
come z = reit .1 In termini di z le equazioni del moto diventano

mx = q yBz my = q xBz : z = iq zBz


1 Questo trucco viene usato in calcoli piu complicati (meccanica quantistica,. . . ), per cui e utile vederlo allopera in questo

problema piu semplice, dove non da una grande semplificazione. Quindi se confonde le idee, conviene rifare i conti ritornando ad x
ed y.
54 Capitolo 7. Moto in campo magnetico esterno

e quindi z(t) = z(0)eiB t con B = qBz /m. Aggiungendo il campo elettrico lequazione del moto diventa

iEx q/m it
mz = iq zBz + qEx eit : z = e
B

dove abbiamo usato un secondo trucco matematico: abbiamo assunto z(t) eit il che, come noto, trascura
il transiente e fornisce solo lorbita limite.
Per = 0 si ritrova il drift a velocita costante vy = Ex /Bz precedentemente discusso: grazie alla i esso e
diretto lungo y.
Per 6= 0 la carica q gira, acquistando una grossa velocita se = B . Intuitivamente la particella gira nel
campo magnetico, ed ad ogni mezzo giro il campo elettrico viene riorientato in modo da essere sempre lungo il
moto della particella, che quindi viene accelerata lungo una spirale. Come al solito questa tecnica trascura il
transiente e Si puo verificare che i raggi limiti ottenuti dalle seguenti similazioni numeriche sono in accordo con
il valore atteso:

/ B = 0.8 / B = 0.9 / B = 1 / B = 1.1 / B = 1.2

Quindi il ciclotrone e un modo di accelerare particelle in uno spazio ridotto. Aggiungendo un termine di attrito
(che puo essere causato da vari effetti fisici e.g. lirraggiamento) il denominatore diventa B + i, risonante
ma finito per = B .
La frequenza di rotazione B non dipende dallenergia della particella; questa semplificazione non e piu
vero quando la particella acquista unenergia relativistica: in tale caso diventa necessario variare in modo
appropriato per ogni bunch.
Uno potrebbe pensare che sia piu complicato ma anche piu conveniente mettere un campo elettrico che gira
con la particella, in modo che E sia sempre diretto lungo v. In termini di numeri complessi un E ruotante nel
piano xy si scrive come E = Eeit (1, i, 0)/ 2. Il segno specifica la direzione di rotazione. Lequazione
del moto diventa
11
mz = iq zBz + q Eeit .
2
Cioe se metto senso di rotazione sbagliato non accelero nulla, e se lo metto giusto non guadagno quasi nulla.
Infatti il campo elettrico oscillante puo essere decomposto come sovrapposizione lineare di due campi che
ruotano in direzioni diverse: (1, 0, 0) = (1, i, 0)/ 2 + (1, i, 0)/ 2: uno e in risonanza e laltro non ha effetto.

Esercizio 100: Carica in B con direzione non uniforme


Moto in un B la cui direzione varia lentamente: studiare il moto di una particella di carica q in un campo
magnetico con B costante.

bSoluzione: Le equazioni del moto in coordinate cilindriche (, , z), dove B ha una componente B costante,
sono
a = 2 = B z, a = + 2 = 0, z = B

dove B qB /m. In prima approssimazione procede lungo le linee del campo facendo una spirale di raggio
a. Quindi = R e = v|| /R. Scriviamo la soluzione al primo ordine perturbativo in a/R: la 1a equazione del
moto fornisce z = v||2 /B R. Avendo assunto R  a la velocita di drift e piccola: z/v|| a/R.
Lo si puo capire in modo intuitivo notando che per far curvare la traiettoria media lungo le linee del campo
serve una forza diretta lungo . Abbiamo visto allesercizio precedente che il moto in B ed E ortogonali e un
drift lungo B E. In questo caso = z.
Capitolo 7. Moto in campo magnetico esterno 55

Esercizio 101: Carica in B con modulo non uniforme


Moto in un B il cui modulo varia lentamente in direzione ortogonale a B. Studiare il moto di una particella di
carica q in un campo magnetico B = (0, 0, Bz (x)).

bSoluzione: Per semplicita assumiamo due valori costanti B1 < B2 = B1 + B nei due semispazi x > 0 e
x < 0 La particella si muove lungo semicirconferenze di raggi ri = mv/qBi con frequenza = qB/m (dove
B (B1 + B2 )/2). Ad ogni giro si sposta di y = 2(r2 r1 ) e quindi drifta con velocita
2
v B2 B1 v r B2 B1 mv
vydrift 2(r2 r1 ) = B
2 B B r qB

In generale la direzione del drift e B B 2 ; il verso dipende da q ma la velocita di drift no.


In generale B = 0 implica che ne il modulo ne la direzione di B sono costanti, per cui si ha leffetto
combinato dei due drift discussi in questo esercizio e nel precedente. Nel caso del campo magnetico di un filo,
B 1/ entrambi gli effetti producono un drift lungo z, con velocita

v||2 + v
2
/2
v drift = B
B

dove il versore z e stato scritto in modo complicato in modo che la formula sia valida in generale per il moto
con a  in un B 2-dimensionale (i.e. nel problema concreto B non dipende da z).

Esercizio 102: Carica in B(t) uniforme


Studiare il moto di una particella di carica q libera di ruotare nel piano xy in un campo magnetico Bz (t) che
viene lentamente variato da B0 a B1 .

bSoluzione: Assumiamo che B vari di poco in un giro: quindi le orbite sono approssimabili come circonferenze.
Il loro raggio a e determinato da mv 2 /a = qvB. Allinizio la carica ha velocita v0 e quindi gira con raggio
a0 = mv0 /qB0 . Alla fine avra velocita v1 (da calcolare) e quindi girera con raggio a1 = mv1 /qB1 .
La forza magnetica non accelera le particelle, ma un campo magnetico Bz (t) induce un campo elettrico
E (t). Dallequazione E = B segue 2rE = = r2 B. Denotando con v il modulo della velocita, la
particella viene accelerata secondo

qa(t) v Bz d v2
mv ' qE = Bz = m cioe ' 0. (7.2)
2 2 Bz dt Bz

Quindi v 2 /Bz (o equivalentemente il flusso Bz a2 , o equivalentemente il dipolo magnetico = qav/2)


sono invarianti adiabatici. La parola adiabatico ed il ' ricordano che il tutto e vero solo nel limite di campo
lentamente variabile.


Riotteniamo lo stesso risultato in modo alternativo, approssimando la carica ruotante come un dipolo
magnetico. Questa approssimazione e possibile solo se la carica ruota veloce, cioe stiamo anche qui facendo
lapprossimazione adiabatica. Una carica produce una corrente i = q/2 e quindi un momento magnetico
q 2 q
= a2 i = a = L dove L = mav = ma2
2 2m
e il momento angolare.
La forza su di un dipolo magnetico dipende solo dal campo magnetico, secondo
e
L = B = L B.
2m
56 Capitolo 7. Moto in campo magnetico esterno

Una particella libera in un campo magnetico ruota sempre attorno ad esso, quindi L e B sono sempre paralleli,
quindi L rimane costante in accordo con i nostri risultati precedenti.


Nel limite opposto, in cui non dipende da B, lequazione del moto dice che L gira attorno a B con
frequenza L = eB/2m. In un linguaggio meno sofisticato, questo accade perche il campo magnetico fa girare
piu veloce la carica, con frequenza + L (o piu lenta: il segno sara tale che la variazione di moto genera un
campo magnetico che si oppone alla variazione del B esterno). Studiamo un caso particolare importante per la
teoria del magnetismo nella materia.

Esercizio 103: Atomo in B(t) uniforme


Studiare come reagisce un atomo di idrogeno quando viene acceso lentamente un campo magnetico esterno.

bSoluzione: Il problema e analogo a quello precedente, con la differenza che lelettrone gira risentendo anche
della forza di Coulomb, non solo del campo magnetico esterno. Possiamo quindi utilizzare ancora leq. (7.2),
tenendo conto che ora il legame fra a(t) e v(t) e dato da mv 2 /a = evB + e2 /40 a2 . Per semplicita, as-
sumiamo che il campo magnetico dia una piccola correzione ad v e quindi al raggio dellorbita a, che in prima
approssimazione rimane costante: questa e lunica differenza rispetto al caso precedente. Quindi
ea v(t)
mv = B (t) = (0) + L (t)
2 a
dove L = eB/2m e detta frequenza di Larmour, che stiamo assumendo essere una correzione piccola, L  .
E.g. per B = Tesla viene L = 0.9 1011 Hz, mentre gli atomi hanno frequenze tipiche 1016 Hz.

Esercizio 104: Carica in B non uniforme


Calcolare in approssimazione adiabatica la velocita di drift parallela al campo magnetico.

bSoluzione: Nei vari casi precedentemente studiati abbiamo trovato nuovi effetti: drifts ortogonali al campo
magnetico. Ora studiamo in dettaglio il vecchio moto lungo le linee del campo magnetico, che in molti casi
rimane il moto principale. Se il campo magnetico e uniforme, la carica procede con v|| costante. Vogliamo
vedere come procede in generale. Il risultato e:
v 2 = v||2 + v
2
= costante e 2
v /B = invariante adiabatico
dove la prima costante del moto e lenergia (ovvio!). Lesistenza di un invariante adiabatico la si puo capire
intuitivamente dai problemi precedenti, dove la stessa espressione era invariante adiabatico. Nel caso precedente
B dipendeva dal tempo (producendo un E), mentre ora dipende dallo spazio: ma dal punto di vista di una
particella che cammina e come se B dipendesse dal tempo.
2
Avendo capito intuitivamente perche v /B e invariante adiabatico, vogliamo dimostrarlo in modo rigoroso.
Mettendo lasse z lungo la linea di B si ha che una variazione di Bz (z) e accompagnata da un B = Bz0 (z)/2,
in quanto questo e necessario per avere B = 0. Per verificarlo basta usare la divergenza in coordinate
cilinidriche, o imporre che il flusso di B su di un appropriato cilindretto sia zero.
La componente B contribuisce allequazione del moto della carica q lungo z:
qv B v2
z = ' Bz0
m 2Bz
2
avendo usato v ' v e ' mv0 /qBz0 in approssimazione adiabatica. Verifichiamo che v /Bz e costante:
2
d v Bz 2 1 d 2 B0 2 2z z z v2 B 0
= v (v v=2
) = z zv = 2 (z + z ) = 0
dt Bz Bz Bz dt Bz Bz Bz 2 Bz
in quanto lultima espressione coincide con lequazione del moto.


Una conseguenza e lintrappolamento magnetico: se Bz (z) aumenta ad un certo punto v|| = 0 e la particella
deve tornare indietro. Nel campo magnetico terrestre questo fenomeno di intrappolamento tiene particelle
cariche nelle fascia di Van Allen.
Capitolo 7. Moto in campo magnetico esterno 57

Esercizio 105: Carica in quadrupolo magnetico


Studiare il moto di una carica q con velocita v quasi parallela allasse z in un campo magnetico B = (bxy).

bSoluzione: Avendolo scritto come gradiente di un potenziale magnetico Vm il campo magnetico soddisfa
automaticamente alla IV equazione di Maxwell nel vuoto, B = 0. Siccome 2 Vm = 0 soddisfa anche a
B = 0. Sperimentalmente e ottenibile nel seguente modo. Il ferro ha 1000 e quindi le linee di B
escono praticamente perpendicolari al materiale. Quindi lo si costruisce in modo che segua la forma delle linee
equipotenziali di xy. [Spostare nel capitolo 10]. Vale

Bx = by By = bx, Bz = 0

Lequazione del moto e


mx = qvbx, my = +qvby
Assumendo che la particella si muova circa lungo lasse z e venga deflessa poco possiamo approssimare d/dt =
v d/dz ottenendo
d2 x 2 d2 y qb
2
= k x, 2
= +k 2 y dove k2 =
dz dz mv
la cui soluzione e
1 1
x(z) = x0 cos kz + x00 sin kz, y(z) = y0 cosh kz + y00 sinh kz.
k k
Quindi un fascio di particelle viene focalizzato lungo lasse x e defocalizzato lungo lasse y. Piu precisamente,
assumendo che ` sia la lunghezza della zona dove B 6= 0 un fascio avente x00 = 0 passa per x(z) = 0 a
z ` = cot(k`)/k: cioe il campo magnetico si comporta come una lente di distanza focale f = 1/k sin k`. (La
differenza fra cot e 1/ sin e questione di definizione dovuta allo spessore finito della lente: il piano principale sta
a distanza (1 cos k`)/k sin k` dalluscita del quadrupolo).

Esercizio 106: Carica in quadrupoli magnetici


Studiare cosa succede quando un fascio di particelle attraversa due quadrupoli magnetici, ruotati di 90 luno
rispetto allaltro.

bSoluzione: Abbiamo visto che un quadrupolo magnetico focalizza un fascio lungo una direzione ma defocalizza
lungo laltra. Il segno diverso e sgradito ma inevitabile, perche dovuto a come deve essere fatto un campo
magnetico nel vuoto. In molti esperimenti e invece necessario focalizzare un fascio lungo entrambi gli assi, in
modo da concentrarlo. Infatti, incrociando due fasci, il numero di urti fra particelle aumenta se i fasci sono
stretti, analogamente a come il numero di incidenti aumenta quando una strada diventa piu stretta.
Scopo di questo esercizio [Christofilos, 1950; Courant et al., 1952] e mostrare che mettendo due quadrupoli
magnetici ruotati di 90 uno dopo laltro si riesce a focalizzare in entrambe le direzioni.
Conviene riscrivere leffetto di un singolo quadrupolo usando la formulazione matriciale dellottica geomet-
rica. In tale formulazione si studia levoluzione con z di (x, x0 ) e di (y, y 0 ). Cioe x e la distanza dallasse ottico,
ed x0 e linclinazione rispetto allasse ottico di un raggio. Nel caso dellottica si studiando raggi di luce, qui fasci
di particelle. Attraversando un magnete quadrupolare di lunghezza ` si ha
       
x(`) x(0) y(`) y(0)
= Mx = My
x0 (`) x0 (0) y 0 (`) y 0 (0)

dove le matrici di trasferimento sono


   
cos k` sin(k`)/k cosh k` sinh(k`)/k
Mx = , My =
k sin k` cos k` k sinh k` cosh k`

Per k 0 si ritrova lo spazio vuoto.


La formulazione matriciale dellottica e conveniente perche per studiare leffetto combinato di sistemi diversi
basta moltiplicare le loro matrici. Quindi attraversando due quadrupoli ruotati di 90 a distanza d fra loro si
58 Capitolo 7. Moto in campo magnetico esterno

z
1 2 3 4

a b

Figura 7.2: Lente convergente.

ha, assumendo per semplicita `  d, 1/k (lente sottile), le matrici di propagazione per raggi inclinati lungo x e
lungo y sono, rispettivamente date dai prodotti Mx M (d) My e My M (d) Mx , che valgono
       
1 `0 1 d 1 `0 1 + d +
'
k 2 ` 1 0 1 k 2 ` 1 d`2 k 4 1
Si ha focalizzazione in quanto lelemento 21 e negativo.
Senza assumere d  ` si troverebbe

(Mx M (d) My )21 ' k 4 `2 (3d + 2`)/3 (My M (d) Mx )21 ' k 4 `2 (` 3d)/3

cioe che serve d > `/3, e che la lente e astigmatica (fx 6= fy ).

Esercizio 107: Ottica geometrica matriciale


Viene riassunto lutilizzo delle matrici come formalismo conveniente per problemi di ottica geometrica.

bSoluzione: Lottica geometrica studia come si propagano raggi di luce. Un raggio che si propaga lungo lasse
ottico z nel piano xz e descritto da due quantita: la posizione x(z) e linclinazione x0 (z).
Attraversando una distanza ` vuota si ha ovviamente
     
x(`) x(0) 1 `
= M (`) M (`) =
x0 (`) x0 (0) 0 1
Attraversando una lente sottile di focale f cambia solo linclinazione del raggio. Esso varia di una quantita
costante, chiamata distanza focale inversa 1/f :
     
x x 1 0
= Mlente M=
x0 dopo x0 prima 1/f 1

Per il resto serve solo sapere come moltiplicare matrici: se X4 = M34 X3 e X3 = M23 X2 e X2 = M12 X1 ,
allora X4 = M34 M23 M12 cioe le matrici vengono moltiplicate in ordine inverso: la prima (M34 ) e quella che
corrisponde allultimo passo.Consideriamo ad esempio il sistema standard in fig. 7.2. La matrice di propagazione
da 1 a 4 e
       
1 b 1 0 1 a 1 b/f a + b ab/f
M14 = M (b) Mlente M (a) = =
0 1 1/f 0 0 1 1/f 1 a/f

I raggi disegnati convergono in un punto unico se lelemento 12 della matrice totale vale zero, cioe se 1/a + 1/b =
1/f . Quando questa relazione e soddisfatta si ha xC = (1 b/f )xA : quindi 1 b/f e il fattore di ingrandimento
della lente. Se esso e negativo, limmagine viene rovesciata.
Quindi lottica matriciale consente di riprodurre i risultati noti per una lente. Lottica matriciale e utile
perche consente di ottenere risultati non noti per sistemi arbitrari di lenti: basta moltiplicare le matrici delle
singole componenti.
Capitolo 8

Induzione magnetica

Esercizio 108: Circuito allungato


Un circuito rettangolare di reststenza R e lati ` fisso ed x variabile e immerso in un campo magnetico B
ortogonale. La lunghezza x viene variata x = vt. Calcolare la corrente indotta, la forza esterna F e la potenza
W necessarie, e la potenza dissipata nella resistenza.

bSoluzione:
a. = BLvt e quindi I = E/R = BLv/R.
b. Fext = BIL = B 2 L2 v/R in direzione opposta al moto.
c. Wext = F v.
d. W = I 2 R = Wext . Tutto il lavoro fatto viene dissipato tramite R.

Esercizio 109: Cilindro ruotante


Un cilindro conduttore viene fatto ruotare a velocita angolare parallela ad un campo magnetico costante B.
Un contatto strisciante di resistenza R connette il bordo con il centro. Quanta corrente vi passa?

bSoluzione:

Esercizio 110: Generatore


Come si produce la corrente ?

bSoluzione: Per trasformare energia meccanica in energia elettrica, si fa ruotare una spira in un campo
magnetico costante (generato e.g. da un magnete) o far ruotare un campo magnetico attorno ad una spira
ferma. In entrambi i casi vale E = , ma questa formula segue da due diverse equazioni di base.
Nel caso in cui la spira e ferma segue direttamente dalla seconda equazione di Maxwell. Nel caso in cui il
campo magnetico e fermo, segue dalla forza di Lorentz. Per vederlo mettiamo B lungo z e facciamo girare una
spira di lati x e y attorno allasse x:
Bz x

59
60 Capitolo 8. Induzione magnetica

La forza magnetica ha una componente lungo il circuito solo sui lati esterni, i cui punti hanno coordinata
y = 12 y cos (t) e quindi Fx /e = (v B)x = vy Bz che integrata vale E = 22 dt
d
y x sin = . (la fem
preduce una vx per cui per far girare la spira occorre compiere lavoro uguale allenergia elettrica prodotta).
E = non vale sempre:

Ondulatore

Macchina unipolare: una sfera magnetizzata verticalmente contiene


R un B M viene fatta ruotare. Fra
polo e contratto strisciante su equatore si crea una fem E drrB, calcolabile anche come variazione
del flusso di un circuito ruotante

Esercizio 111: Generatore in orbita


Il campo magnetico terrestre allequatore vale B0 = 0.5 G. Un satellite ruota con v = 7 km/ s su un orbita di
raggio R = 8000 km. Dal satellite pende un filo lungo L = 200 m. Calcolare la ddp ai suoi capi.

bSoluzione: vB = Lv(R0 /R)3 35 V. Facendo oscillare la lunghezza del filo si genera una corrente alternata.
R

Esercizio 112: Trasformatore


Discutere i trasformatori.

bSoluzione: Per trasmettere una data potenza W = V I da una centrale elettrica al luogo di consumo conviene
utilizzare un grosso V ed una piccola I, in modo da ridurre la potenza RI 2 dissipata per effetto Joule lungo la
linea di trasmissione. Ad esempio per trasmettere 10 kW a 100 V serve I = 100 A; se invece la trasmetto a 10
kV basta I = 1 A, riducendo di un fattore 104 la potenza dissipata nel filo.
Per motivi di sicurezza lenergia elettrica va pero venduta a basso voltaggio. E quindi essenziale che un
trasformatore sappia convertire il voltaggio senza perdite. Un trasformatore si basa sullinduzione magnetica,
che richiede correnti variabili. Per questo motivo le correnti alternate sono piu usate di quelle continue.
Le equazioni di base di un trasformatore sono

V1 1 = V1 L1 I1 M I2 = R1 I1 2 = L2 I2 M I1 = R2 I2

e la quantita interessante da calcolare e il fattore di conversione V2 /V1 = R2 I2 /V1 .


Nel limite ideale R1 = 0 ed R2 viene I2 0 e quindi V2 /V1 = M/L1 = N2 /N1 . Se I2 6= 0 il secondo
circuito reagisce sul primo. Per calcolare come assumo una fem alternata V1 = V1 eit ed uso il metodo delle
impedenze
V1 = (R1 + iL1 )I1 + iM I2 (R2 + iL2 )I2 + iM I1 = 0
da cui

V2 iM R2 W2 R2 M 2 2
= =
V1 (R1 + iL1 )(R2 + iL2 ) + 2 M 2 W1 R2 + IL2 (R1 + iL1 )(R2 + iL2 ) + 2 M 2

[i moduli quadri andrebbero presi con cura] La potenza trasmessa vale 1 se R1,2 0 e se il termine 2 (M 2
L1 L2 ) al denominatore vale zero. Questo
corrisponde ad avere un trasformatore che non disperde flusso. Il
massimo valore possibile di M e M = L1 L2 .
Assumendo accoppiamento perfetto e che R1 sia trascurabile
r
V2 L2 N2
= =
V1 L1 N1

E chiaramente importante avere L  R, altrimenti W2 /W1 L2 /R2 . Nel vuoto si ha L N 0 d dove d e la


dimensione del trasformatore ed N e il numero di spire. Per 100 Hz e d m viene L N 107 . Il ferro
migliora la situazione di un fattore /0 104 .
Capitolo 8. Induzione magnetica 61

Esercizio 113: Trasformatore con due spire


Non si usa il ferro. Due spire concentriche, di raggi A ed a.

bSoluzione: L 0 a, MAa ' 0 a2 /2A. MaA = MAa in modo non ovvio.

Esercizio 114: Induzione


Due circuiti hanno autoinduttanze L1 ed L2 e M . Studiare cosa succede quando si connettono i fili in vario
modo.

bSoluzione: In generale
E1 = L1 I1 M I2 E2 = L2 I2 M I1
con M > 0, avendo scelto stesse direzioni per I ed E. (E = , posso fare lo stesso esercizio con ).

Se li attacco in serie dritti E = E1 + E2 e I = I1 = I2 . Quindi ottengo un unico circuito con induttanza


L = L1 + L2 + 2M .
Se li attacco in serie a rovescio E = E1 E2 e I = I1 = I2 . Quindi ottengo un unico circuito con
induttanza L = L1 + L2 2M . Deve essere L > 0 e quindi M < (L1 + L2 )/2.
Se li attacco in parallelo dritti E = E1 = E2 e quindi
L2 M L1 M L1 + L2 2M
I1 = E , I2 = E , I1 + I2 = I = EL L=
L1 L2 M 2 L1 L2 M 2 L1 L2 M 2

da cui M < L1 L2 , che e una condizione piu stringente. Notare che e possibile avere |M |  L1 L2 ,
quando M < 0.
Se li attacco in parallelo a rovescio viene L = (L1 + L2 + 2M )/(L1 L2 M 2 ).
Se M = 0 le induttanze si combinano come le resistenze.

Esercizio 115: Trapano


Inventare un trapano

bSoluzione: Iniziamo da un esercizio ideale: supponiamo di avere un campo magnetico con simmetria cilindrica
Br = B0 (r0 /r). Verifica B = 0 per r 6= r0 .
Una spira di lunghezza `, nella quale viene fatta passare una corrente continua i, e libera di ruotare ra-
dialmente. La forza di Lorentz sul lato superiore vale F+ = `iB(r+ ) e tende a far girare la spira; tuttavia sul
lato inferiore si esercita una forza F = (r+ /r )F+ tale che il momento totale e zero. Se a uno piace com-
plicarsi la vita e pensare in termini di flusso tagliato questo accade perche il flusso tagliato dal filo superiore
( = B(r+ )`r+ ) e uguale ed opposto a quello tagliato da filo inferiore.
Quindi, per ottenere un trapano funzionante, si elimina il filo inferiore rimpiazzandolo con il contatto mobile
disegnato in figura

F
F
62 Capitolo 8. Induzione magnetica

Studiamo adesso come si fa ad ottenere il campo magnetico assunto. Non disponendo di un filo di monopoli
magnetici lo si puo costruire con un magnete non su tutta la circonferenza ma solo su di una parte. Questo e
ottenibile usando un magnete permanente. Nella parte di sotto il campo magnetico tira dritto e produce un
momento che tende a frenare la spira. Per evitare questo si puo invertire il segno della corrente i quando fa il
mezzo giro nella parte inferiore, utilizzando quindi una corrente alternata di frequenza uguale alla frequenza di
rotazione. Questa non e una buona soluzione, in quanto una presa di corrente da invece una corrente alternata
di frequenza fissa, 60 Hz. Conviene quindi avvolgere un solenoide attorno ai magneti in modo da fargli creare un
campo magnetico nella direzione giusta quando ci passa la spira, in modo che venga sempre accelerata. Siccome
far percorrere ad un magnete il suo ciclo di isteresi costa energia (a seconda di quanto e largo il ciclo di isteresi),
si mettono due o piu spire con correnti circolanti in senso opposto in modo da usare il campo magnetico quando
punta in entrambe le direzioni.


La rotazione del filo produce una v e quindi una forza di Lorentz lungo il filo, e quindi una fem indotta
E = qr+ B` che ad alta frequenza riduce la fem iniziale frenando la rotazione della spira Poi la spira produce
a sua volta un campo magnetico: si ha anche un effetto di auto-induzione

E = qBr+ ` = Ri + L di/dt I = Br+ `i

da cui, senza fem esterna,


d2 d (Br+ `)2
L 2
+R +q =0
dt dt I
dando oscillazioni smorzate.
Capitolo 9

Forze magnetiche fra circuiti

Hanno importanza soprattutto come esercizi in compiti.

Esercizio 116: Due circuiti lunghi


Si considerino i due circuiti rigidi in figura, con L  d. a) De-
terminare il coefficiente di mutua induzione. Due generatori di
corrente mantengono le correnti I1 ed I2 costanti. Le resisten- d I1
ze elettriche sono trascurabili. b) Determinare le forze esterne d
necessarie a mantenere i circuiti fermi come in figura. c) Lascian- d I2
do libero uno dei circuiti di muovesi, determinare la sua energia
cinetica a distanza infinita.
bSoluzione:
a) Dominano le forze fra i fili lunghi. Chiamandoli 1,2,3,4 (dallaltro in basso)
 
0 L I1 I2 1 2 0 L I1 I2
F = F14 + (F13 + F24 ) + F23 = +1 =
2 d 3 2 2 3d

La forza e repulsiva.
b) M < 0 in quanto i due circuiti sono esterni: uno prende il flusso dellaltro nella regione in cui B torna
indietro. Lungo il piano che contiene i due circuiti, a distanza x dal filo 2
  Z 2d
0 I1 1 1 0 L 4
B= + , (B) = L B dx = M I1 M = ln
2 x x+d d 2 3

c) Sfruttiamo la conservazione dellenergia: UI + L = UF + K. K e lenergia cinetica che si vuole calcolare.


UF ed UI sono le energie magnetiche, in generale date da U = Lij Ii Ij . Alla fine L12 = M = 0 mentre
allinizio M ha il valore calcolato al punto b). L e il lavoro fatto dai generatori per mantenere costanti
le correnti
2 Z 2 Z 2
X X di X
L = Ii Ei dt = Ii dt = Ii [iF iI ] = I1 [0 M I2 ] + I2 [0 M I1 ] = 2M I1 I2
i=1 i=1
dt i=1

Quindi K = (UF UI ) + L = (1 2)M I1 I2 = M I1 I2 > 0, in accordo con il fatto che la forza e repulsiva.
c0 ) Si puo anche procedere in modo meno intelligente intergando la forza ricalcolata a distanza r > d generica:

2d2
 
0 L 1 2 1 0 L
F = F14 + (F13 + F24 ) + F23 = I1 I2 + = I1 I2
2 2d + r d + r r 2 r(d + r)(2d + r)

R grande distanza r  d deve ridursi alla forza fra due dipoli magnetici i = LdIi . Integrando K =
A
d
F dr si ritrova il risultato precedente

63
64 Capitolo 9. Forze magnetiche fra circuiti

Esercizio 117: Rotazione di due spire circolari


[Dal compito del 19/9/03] Due spire conduttrici circolari coplanari diposte nel piano xy hanno raggi diversi in
modo trascurabile, sezione del filo trascurabile, resistenza R e coefficiente di autoinduzione L. La spira interna
e libera di ruotare attorno allasse x e inizialmente percorsa da una corrente i0 , mentre la spira esterna e fissa
e collegata ad un generatore ideale di corrente che eroga la corrente I. Le correnti scorrono nello stesso verso.
Allistante t = 0 ed in un tempo trascurabile rispetto al tempo caratteristico del sistema, la spira interna viene
ruotata di 90 e fermata.

a) Si calcoli la relazione tra il coefficiente di autoinduzione L delle spire e quello di mutua induzione M sia
al tempo t < 0 (quando le spire sono coplanari) sia a t > 0 (dopo aver effettuato la rotazione).

b) Supponendo trascurabile la caduta ohmica nella spira interna durante la rotazione, si calcoli la corrente
che circola in essa allistante dellarresto.

c) Si calcoli il lavoro fornito dal generatore di corrente durante la rotazione.

d) Sapendo che lenergia dissipata nella spira interna per effetto Joule durante il tempo transitorio di scarica
successivo alla rotazione della spira e LJ , si determini L.

e) Si calcoli il lavoro meccanico Lmecc speso per far ruotare la spira.

bSoluzione:

a) Allinizio M0 = L. Dopo la rotazione, M = M1 = 0, come si puo vedere da considerazioni di simmetria.

b) Durante la rotazione la f.e.m. ai capi della spira interna e: E = d/dt = Ri. Siccome la rotazione
avviene molto velocemente possiamo trascurare Ri, e quindi = M I + Li rimane costante. Quindi
Li1 = Li0 + M0 I cioe i1 = i0 + I.

c) La conservazione dellenergia consente di calcolare il lavoro totale Lgen + Lmecc = U1 U0 , ma non le


singole componenti.
Il lavoro compiuto dal generatore di corrente si ottiene facilmente integrando la potenza dissipata sia a
causa della presenza della resistenza, sia dovuta alla forza elettromotrice indotta:
Z t
Lgen = RI 2 EI dt = RI 2 t + I = RI 2 t + I(M i)

0

Per t 0 la dissipazione ohmica diventa trascurabile. Essendo (M i) = M0 i0 = Li0 , risulta che


Lgen = Li0 I. Verifica del segno: il sistema da solo tenderebbe ad aumentare I per opporsi alla variazione
di , che decresce. Siccome invece I viene mantenuta costante, il generatore riceve energia.

d) Durante il transiente successivo alla rotazione lenergia immagazzinata nellinduttanza si dissipa per effetto
Joule, quindi
L L
LJ = i21 = (i0 + I)2 .
2 2

e) Trascurando la dissipazione Joule durante la veloce rotazione, dalla conservazione dellenergia segue che

L 2 L L
Lmecc = U1 U0 Lgen = (i + I 2 ) (i20 + I 2 ) M0 Ii0 + LIi0 = I(I + 2i0 )
2 1 2 2

Calcolare il lavoro meccanico come integrale Lmecc = M d del momento delle forze sarebbe piu
R

complicato, ma consente di vedere subito che vale zero per I = 0.


Capitolo 9. Forze magnetiche fra circuiti 65

Esercizio 118: Una spira ed un dipolo


[Dal compito del 6/2/04] Un dipolo magnetico con momento magnetico

~ e posto al centro di una spira circolare di raggio a. Il sistema di assi


cartesiani e fissato in modo che lorigine sia nel centro della spira e gli assi
z
x e y nel suo piano. Il dipolo viene fatto ruotare con velocita angolare =()
costante nel piano x, z (vedi fig. 1)

a) Si determini la corrente che scorre nella spira sapendo che es-


sa ha una resistenza elettrica R (si trascuri il suo coefficiente di a
autoinduzione). y

b) Si determini il momento meccanico esterno necessario a mantenere


il dipolo in rotazione.
x Fig.1
c) Si mostri che la potenza meccanica media fornita e eguale alla
potenza dissipata per effetto Joule.
La spira sia connessa a un generatore di corrente costante I.
d) Se il dipolo ha massa m e si puo muovere lungo z, si determini lorientamento relativo della corrente nella
spira e del dipolo perche la forza lungo z sia di richiamo attorno al punto z = 0 e la frequenza delle sue
piccole oscillazioni.

bSoluzione:
a) Il dipolo magnetico e equivalente ad una spiretta di superficie s percorsa da corrente i tale che = si. La
corrente nella spira grossa dipende dalla variazione del flusso di mutua induzione della spiretta sulla spira.
E complicato calcolarlo, mentre e facile calcolare il coefficiente di mutua induzione M () dalla spira sulla
spiretta. Per via di un teorema generale i due coefficienti sono uguali. Ricordando che il campo magnetico
nel centro della spira vale Bz = 0 I/2a abbiamo
s () 0 s
M () = = cos .
I 2a
Quindi il flusso indotto dalla spiretta (dipolo) sulla spira e
0 s cos 0
S () = i M () = i= cos t
2a 2a
e non dipende dalla superficie s arbitraria della spiretta. La corrente I e data da
E(t) 1 S (t) 1 0
I= = = sin t
R R t R 2a
b) Il momento meccanico esterno M deve essere opposto al momento delle forze dovuto allinterazione tra
dipolo e campo della spira, M = B, che ha solo componente
2 20
My = x Bz = Bz sin t = sin2 t
4Ra2
c) La potenza W sviluppata dal momento delle forze esterne e
2 20 2
W = My = sin2 t
4Ra2
eguale alla dissipazione Joule WJ = E 2 /R.
d) Affinche la forza sia di richiamo il potenziale U (z) = B deve essere minimo a z = 0, quindi e B
devono essere concordi. La forza e:
Bz 0 Ia2 3 0 Ia2 z 3 0 Iz
Fz = mz = = = '
z 2
z 2(a + z )2 3/2 2 2
2 (a + z )5/2 2 a3
p
da cui la frequenza delle piccole oscillazioni, = 30 I/2ma3 .
66 Capitolo 9. Forze magnetiche fra circuiti

Esercizio 119: Monopolo magnetico


Studiare come reagisce una spira circolare di raggio a quando lungo il suo asse passa, a velocita v costante a)
un dipolo b) un monopolo magnetico.

bSoluzione: Entrambi inducono un flusso indotto e quindi una fem E = . Se la spira ha auto-induttanza
L trascurabile e resistenza R, misuro una corrente I = /R. Se invece la spira ha R trascurabile ed auto-
induttanza L ho E = LI e quindi misuro una corrente I = I0 (t)/L. In generale succedono cose piu
complicate. Nel seguito studio questi due casi particolari.
a) Dipolo magnetico. Usiamo lo stesso trucco usato alla domanda a) dellesercizio precedente. Il coeffi-
ciente di mutua induzione della spirona sulla spiretta (dipolo) e

0 a2 0 a2
M= s cos , D = iM = cos
2(a2 + z 2 )3/2 2(a2 + z 2 )3/2

dove e langolo che fa il dipolo rispetto alla spira e z = vt.


b) Monopolo magnetico. Un monopolo magnetico qM produrrebbe un campo magnetico a simmetria
sferica, Br = qM /r2 (la definizione della normalizzazione di qM e arbitraria). Non occorre fare lintegrale
per calcolare il flusso raccolto dalla spira, che e semplicemente dato da qM per langolo solido sotteso dalla
spira, 2[1 cos sotteso ]. Quindi  
z
M = 2qM 1 .
z 2 + a2

La figura mostra landamento temporale di e nei due casi.


flusso

fem

tempo tempo

Misurando una di queste quantita si ricava la velocita delloggetto che passa, e si distingue se e un dipolo
(linea continua blu) o un monopolo (linea tratteggiata rossa) Il monopolo magnetico produrrebbe un segnale
caratteristico, che nessuno ha mai visto.
Notare che i due risultati sono legati da dM /dz = D /d: infatti due monopoli magnetici qM a distanza
d = 0 cos /4qM piccola formano un dipolo magnetico . 1

Esercizio 120: Traslazione di due spire circolari


[Dal compito del 25/9/02] Una spira circolare conduttrice giace vincolata su di un piano parallelo a distanza z
1 Teorici ritengono che i monopoli debbano esistere, abbiano carica q 16
M h/2qE e forse massa M 10 mp , ma che dopo
linflazione ne siano rimasti troppo pochi per essere osservati. Un elettrone sullanello aquista impulso
Z
2qE qM
pr = qE Er dt = qE
2a a

Imponendo la quantizzazione del momento angolare L = apr = h si trova qE qM = h/2. Torna ma non sono sicuro sia corretto,
bisogna capire il significato di 6= 0. Probabilmente per rendere consistenti le eq. di Maxwell modificate bisogna aggiungere una
JM
Capitolo 9. Forze magnetiche fra circuiti 67

dal piano di una seconda spira. La resistenza, il coefficiente di autoinduzione e quello di mutua induzione della
spira superiore siano R, L2 , M . Il coefficiente di autoinduzione della spira inferiore sia L1 . Si invii nella spira
inferiore una corrente 
It/ per t <
i1 (t) =
I per t >
Si determini

a) La corrente i2 (t) nella spira superiore;

b) Lenergia totale dissipata nella resistanza R

c) La carica Q che attraversa R

Nellipotesi che la spira superiore di massa m sia libera di muoversi verticalmente lungo il suo asse parallelo al
campo gravitazionale g, che R = 0 e che sia abbstanza piccolo in modo che M resti costante per 0 < t < si
determini, ripetendo limmissione della corrente i1

d) la quota massima raggiunta dalla spira superiore, sapendo che in questa posizione il suo coefficiente di
mutua induzione e M 0 .

bSoluzione:
a) Per t < i2 e data dallequazione 0
2 `
di2 di1
L2 +M + Ri2 = 0, i2 (0) = 0
dt dt
2 =
risolta da
M I t/2
i2 (t) = (e 1)
R
dove 2 = L2 /R. Per t > si ha di1 /dt = 0 e quindi 2 =
i2 (t) = i2 ( )e(t )/2 . La figura mostra il risultato per -IM/L 2
0 2 3
diversi valori di /2 .

b) Lenergia dissipata vale



I 2M 2
Z
W = dt R i22 (t) = [ 2 + e /2 2 ]
0 R 2

Per 2  si ha i2 ( ) ' IM/R e W ' R (M I/R )2 0 dominata da 0 < t < .


Per 2  si ha i2 ( ) ' IM/R2 = IM/L2 e W ' L2 i22 ( )/2 dominata da < t <
2 .
c) La carica totale che attraversa la resistenza e calcolabile come
Z
Q= dt i2 = IM/R.
0

Il risultato e semplice e lo si puo alternativamente ottenere senza nessun calcolo usando i2 = E/R =
/R dove (t) = M i1 (t) e il flusso del campo magnetico generato dalla spira inferiore su quella superiore.
Si ottiene quindi Q = (i f )/R = M I/R.

d) Per R = 0 si ha E = 0 = quindi = M i1 +Li2 e costante. Siccome allinizio = 0 e siccome i1 raggiunge


velocemente il valore costante i1 (t > ) = I alla massima quota si ha i2 = M 0 I/L2 . Imponiamo adesso
la conservazione dellenergia, tenendo conto che un generatore genera la corrente i1 . Lenergia magnetica
vale
L1 2 L2 2 L1 2 M 2 (t) 2 1 I2
Umag = i1 + i2 + M (t)i1 i2 = I I , Umag = (M 2 M 02 )
2 2 2 2L2 2 L2
Lenergia fornita dal generatore vale

I2
Z Z
d
Lgen = i1 E1 dt = I (M i2 + L1 i1 ) = (M 2 M 02 )
dt L2
68 Capitolo 9. Forze magnetiche fra circuiti

Lenergia gravitazionale vale Ugrav = mg z. Imponendo Umag + Ugrav = Lgen si ottiene

M 2 M 02 I 2
z = .
2 mgL2
Come al solito il generatore contribuisce 2 volte la variazione di Umag .
Capitolo 10

Campi magnetici nella materia

La densita di corrente totale J viene divisa in corrente libera J L (usualmente chiamata J , con abuso di
notazione) e corrente di magnetizzazione J M = M . Per motivi storici si scrive B = 0 (H + M ) = H in
modo che la IV equazione di Maxwell diventa

H = J + JS

La magnetizzazione e approssimativamente legata ad H da M = m H (e quindi = 0 (1+m )). La precessione


di Larmour da il diamagnetismo, < 0. Lallineamento dei dipoli elementari da il paramagnetismo, > 0.
Le condizioni di raccordo sul bordo fra due materiali sono: B continuo, H= continuo quando non ci sono
correnti.

Esercizio 121: Sbarra magnetizzata


Una cilindro di raggio r ha una magnetizzazione uniforme M lungo lasse. Calcolare B. Se ne taglia una fetta
trasversale di spessore  r. Calcolare B nel centro del buco.

bSoluzione: Senza buco: la magnetizzazione M descrive una corrente J m = M . Nel caso del problema
JM e una densita di corrente superficiale = M , come si ottiene applicando Stokes ad un circuitino lungo
parallelo al materiale. Risolvendo B = 0 J M usando lo stesso circuitino si trova B = 0 M .
In modo piu matematico, ma anche piu veloce, dobbiamo risolvere B = 0 B cioe H = 0.
Quindi mettendo H = 0 (niente correnti libere) si ottiene B = 0 B. (I materiali ferromagnetici possono avere
avere M 6= 0 in assenza di un B esterno, realizzando in pratica questo esercizio).
Per tenere conto dellaggiunta del buco possiamo vederlo come laggiunta di una spiretta di corrente i = M ,
che nel centro genera B = 0 i/2r. Quindi B = 0 M (1 /2r). (B e continuo lungo il taglio, ma saperlo non
risolve il problema, in quanto tagliare il materiale modifica anche il campo allinterno del materiale, non solo
nel buco.).

Esercizio 122: Materiali ferromagnetici


N spire con corrente I sono avvolte attorno a materiali ferromagnetici (i.e.  0 ) secondo le 3 geometrie
disegnate in figura. Trovare B nel piccolo traferro.

69
70 Capitolo 10. Campi magnetici nella materia

bSoluzione: Il ferro ha = r 0 con r 103 105 (alcune leghe hanno r piu alto, alltre hanno cicli di
isteresi stretti e dissipano meno energia). Per risolvere problemi su materiali ferromagnetici occore sfruttare il
fatto che, in ottima approssimazione, questi intrappolano le linee del campo magnetico, tenendo costante il flusso
in out
di B. Alluscita dalle imboccature il campo magnetico e circa ortogonale al materiale, in quanto H= = H= e
out in
quindi B= = B= /r 0.

a) Senza buco il campo H = B/ e dato da 4LH = N I, cioe B = N I/4L. Nel vuoto si avrebbe B
0 N I/L: aggiungere il ferro ha lo stesso effetto di ridurre la dimensione: L Lr = L/r .
Aggiungendo un buco (o traferro) per un tratto d  L le equazioni diventano (4Ld)B/+dB/0 = N I.
Notare che anche un piccolo buco d  L puo ridurre B in modo significativo: e.g. un d 4L/r L/10000
lo dimezza circa. Il motivo e che la lunghezza effettiva del circuito varia da L/r a circa d + L/r : il
tratto nel vuoto non e soppresso da 1/r

b) Nel restringimento il flusso di B rimane costante. Se la superficie diventa 4 volte piu piccola, B diventa
4 volte piu intenso.

c) Invece di applicare il formalismo delle riluttanze provo a ragionare. Nella regione con le spire e come il
caso a). Nelle altre regioni bisogna capire come si divide il flusso del campo magnetico alle biforcazioni.
La condizione e che la circuitazione di H lungo la parte destra del circuito valga zero. Quindi B non
puo e.g. passare tutto nella sbarra intermedia, ma deve dividersi: 3/4 nella sbarra intermedia, ed 1/4 fa
il giro lungo. Quindi il campo magnetico nellinterferro e 4 volte piu debole che nel caso a).

Esercizio 123: Due bacchette


Ci sono due bacchette di ferro. Una emagnetizzata per la sua lunghezza, laltra no. Come si puo scoprire quale
delle due e magnetizzata?

bSoluzione: Chiedere allIll.mo Prof. Claudio.Scrucca@cern.ch.

Esercizio 124: Correnti parassite


[dal compito del 19/9/2003] Un lungo solenoide cilindrico e costituito da N spire per unita di lunghezza avvolte
su un nucleo di ferro di raggio R e lunghezza L  R. Il ferro ha permeabilita magnetica e conducibilita
elettrica . Nelle spire si fa passare la corrente alternata I = I0 cos t.

a) Calcolare il campo magnetico allinterno del solenoide.

b) Calcolare il campo elettrico indotto allinterno del solenoide.

c) Si spieghi perche il nucleo di ferro si riscalda e si calcoli la potenza dissipata per unita di lunghezza.

bSoluzione:

a) B = H = N I

b) Siccome E = B viene E = 12 rnI0 sin t

c) E induce una corrente J = E e quindi una potenza dissipata JE per unita di volume.

In generale un campo magnetico B(t) induce un campo elettrico E(t) e quindi delle correnti J = E che
dissipano energia e sono quindi dette correnti parassite. A frequenze alte bisogna tenere conto che B non e
piu uniforme: questo viene fatto nel prossimo esercizio, nel quale si sceglie la geometria piu semplice possibile
ma si deriva un risultato generale.
Capitolo 10. Campi magnetici nella materia 71

Esercizio 125: Correnti parassite

Un blocco di metallo ha permeabilita magnetica e conducibilita


. Viene applicato un campo magnetico oscillante di frequenza . Ez
Allesterno del blocco B e parallelo alla superficie: By = B0 cos t. x
Determinare la lunghezza di penetrazione del campo nel ferro e la
potenza dissipata dalla corrente indotta. (Si assuma che il blocco By
occupi il semipiano x > 0 e si trascuri la corrente di spostamento).

bSoluzione: Il campo magnetico By (x, t) = Hy (x, t) genera un campo elettrico Ez (x, t) come dettato dalle
equazioni di Maxwell

x Hy = ( H)z = Jz = Ez , x Ez = ( E)y = By = Hy

Eliminando Ez , si trova che Hy soddisfa allequazione di diffusione

Hy Hy
2
=
x t
Assumendo Hy h(x)eit si riduce a h00 = ih,
p risolta, nel semipiano x > 0 dove e costante, da
kx 2 2
h(x) = h(0)e con k = i /2 dove = 2/ viene chiamato lunghezzapdi pelle (la definizione
differisce di un fattore 2 da quella usata in un altro esercizio). Quindi k = (i 1) /2. Eliminando la
soluzione che diverge per x , si ottiene che il campo penetra per una lunghezza dellordine di :

Hy (x) = ReHy (0)e(i1)x/ eit = Hy (0)ex/ cos(x/ t)

La continuita di Hy al bordo x = 0 da la condizione al contorno Hy (0) = H0 = B0 /0 . Il campo elettrico vale


 
x Hy H0 x/
Ez = = e cos(x/ t) sin(x/ t)

ed e piccolo a basse frequenze: Ez /cHy 1/c  1. Esso genera correnti parassite J = E. La potenza
media dissipata per unita di volume e
dW 2 2x/
= hJz Ez it = hEz2 it = H e
dV 2 0
Integrando su x > 0 si trova la potenza dissipata per unita di superficie:
Z
dW dP B02
= dx =
dS 0 dV 0 0

Se e piccolo, viene dissipata poca potenza.

Esercizio 126: In pratica


Avvolgo N spire su di un tubo di ferro, ed accendo la corrente (220 V, 50 Hz). Sul tubo appoggio un anello. Si
muove o la forza magnetica e trascurabile?

bSoluzione:
Parte III

Elettrodinamica
Capitolo 11

Corrente di spostamento

La conservazione della carica = j aggiunge lultimo termine alle equazioni di Maxwell

E = /0 E = B B =0 B = 0 j + 0 0 E

Il nuovo termine j s 0 E viene chiamato corrente di spostamento. Una conseguenza e la presenza di onde

elettromagnetiche che viaggiano alla velocita della luce c = 1/ 0 0 , e.g.

Ez = sin(y ct), Bx = cos(y ct), tutto il resto = 0

Esercizio 127: Scarica di un filo


Un filo rettilineo va da z = 0 a z = ` di area S ha una densita uniforme di carica uniforme (t) = 0 et/ che
si scarica al capo z = `. Calcolare il campo magnetico.

bSoluzione: Iniziamo a calcolare la corrente j. Lequazione di continuita jz /z = equivale a i/z =


(dove = S) e quindi i(t, z) = z . E come svuotare un canale dacqua aprendo una chiusa: la corrente e
forte vicino alla chiusa e debole al capo opposto.
Siccome i dipende da z, se fosse B = 0 j la corrente concatenata dipenderebbe da quale superficie uno
sceglie nellapplicare il teorema di Stokes. La corrente di spostamento mette tutto a posto. La prima equazione
di Maxwell dice che Ez = z/0 (il campo elettrico vale zero a z = 0). La corrente di spostamento quindi vale
isz = S0 Ez = +z . Non nasce nessun campo magnetico.

Esercizio 128: Piano con carica ondulata


a) Trovare il potenziale generato da un piano con densita di carica (x) = 0 cos kx. b) Le cariche sono lasciate
libere di muoversi sul piano con conducibilita . Calcolare la loro evoluzione, la corrente, ed il campo magnetico
generato.

bSoluzione: a) Invece di integrare provo a risolvere 2 = /0 . Tento la soluzione

(x, z) = F (z) cos kx, 2 = [F 00 k 2 F ] cos kx

e quindi F (z) = c+ ekz + c ekz . La soluzione con le condizioni al contorno () = 0 e E = 0 = /0 e


0 k|z|
= e cos kx
2k0
Per k 0 (piano uniformemente carico) si riduce a cte 0 |z|/20 , in accordo con lesercizio a pag. 8.


Questo esercizio e piu importante di quanto sembra, in quanto una qualunque distribuzione di cariche
puo essere decomposta come somma di coseni con diversi k (trasformata di Fourier). Usando il principio di
sovrapposizione, abbiamo una soluzione per il problema generico.

73
74 Capitolo 11. Corrente di spostamento

Ad esempio una griglia di fili a distanza a avra un trasformata di Fourier diversa da zero per k 1/a. Ad
una distanza |z|  a i termini esponenziali diventano piccoli, e si ottiene il campo elettrico uniforme generato
dal modo k = 0, cioe dalla carica totale della griglia. Quindi e facile generare un campo elettrico uniforme.
Lilluminazione e descritta dalle stesse equazioni dellelettrostatica. Quindi una griglia di tubi al neon
produce una illuminazione costante.

b) Il campo elettrico Ex = x = 0 sin kx/20 genera una corrente Jx = Ex che redistribuisce le cariche.
La soluzione e (t) = 0 et/ cos kx, come si vede da
0
= J = E = : =
0

Quindi anche il campo elettrico decade esponenzialmente. Non viene generato nessun campo magnetico in
quanto la corrente di spostamento compensa la corrente

E
J + 0 E = E 0 =0

Vediamo quindi che questo accade in generale quando cariche sbilanciate sono libere di redistribuirsi secondo
J = E. Il prossimo esercizio mostra che questa cancellazione e piu generale.

Esercizio 129: Sfera radioattiva


Una sfera uniforme isolata di raggio a emette isotropicamente positroni da decadimento : n pe con velocita
v0 . Il tempo di decadimento della radioattivita e . Genera un campo magnetico?

bSoluzione: Il numero di neutroni liberi di decadere diminuisce come N = N0 et/ e quindi la sfera acquista
una carica Q(t) = e[N0 N ] > 0.
Iniziamo assumendo v costante e  r/v, cioe un decadimento cos lento da essere approssimabile ad un
processo costante. I elettroni generano una corrente Jr (r) = (e)(N )/4r2 , che non dipende da v. Il campo
magnetico e zero in quanto la corrente di spostamento cancella Jr : Jsr = 0 Er = eN /4r2 .
Per capire se questa cancellazione e un accidente del caso semplificato che abbiamo considerato, o se e invece
dovuta a qualche motivo piu profondo, consideriamo casi progressivamente meno semplici.
Se il decadimento e veloce, r/v0 , in generale Jr (r, t) = eN (t r/v)/4r2 : il numero di elettroni che
attraversano una superficie a distanza r al tempo t dipende da quanti ne erano stati emessi al tempo t r/v0 .
La cancellazione fra J e Js rimane perfetta in quanto Er (r) e determinato dalla carica totale dentro una sfera
di raggio r (che contiene la sfera radioattiva ed una nuvola di elettroni), eguale a e[N0 N (t r/v)]. Si ha
ancora J + Js = 0.
Il calcolo diventa ancora piu complicato se si tiene in conto che v non e costante, in quanto la forza di
Coulomb rallenta i positroni. Il calcolo e complicato, e potrebbe essere fatto con una tecnica analoga a quella
utilizzara per studiate il diodo termoionico. E facile vedere che J e Js si cancellano ancora, in quanto entrambe
proporzionali a N calcolato a qualche istante ritardato. Il ritardo non e piu r/v ma e dato da qualche formula
complicata che non e necessario calcolare.
Sebbene venga qualche i(r) = 4r2 Jr (r) complicata si ha sempre B = 0: e quindi naturale domandarsi quale
sia il motivo generale. Una corrente a simmetria sferica non puo generare un campo magnetico, che dovrebbe
avere solo una componente Br (r), ma questa da rotore zero. Prendendo la divergenza della IV equazione di
Maxwell si ottiene (J + J s ) = 0: la corrente di spostamento deve quindi cancellare J , e lunico modo
che ha di farlo e cancellare Jr .

Esercizio 130: Scarica di un condensatore


Un condensatore di area S = a2 e distanza fra i piatti d  a si scarica con costante tempo . Calcolare il
campo magnetico e la sua energia.
Capitolo 11. Corrente di spostamento 75

bSoluzione: La carica vale q(t) = q0 et/ i.e. i = q = q/ . Il campo elettrico vale 0 E = generando una
densita di corrente di spostamento uniforme js = = q/S . Notare che la corrente i entra in un piatto ed
esce dallaltro; la corrente totale di spostamento vale is = i. Quindi j s genera un campo magnetico ruotante

r2 0 js 0 ri
B = = r<a
2r 2S
(A grande distanza r  a il campo magnetico generato dalla corrente (che non si interrompe) i + is e circa
radiale B = 0 i/2r). Lenergia nel campo magnetico (usando i = S0 / )

B2 di2 0 0 E 2 a2 0 0
Z
UB 1 a
UB = dV = UE = V = = ( )2
20 16 2 UE 8 2 8 c
e trascurabile a meno che a c , in generale a meno che E vari significativamente nel tempo che la luce impiega
ad attraversare lapparato.

Esercizio 131: Condensatore in alternata


Studiare una capacita (costituita da due cerchi codnuttori a distanza d) alla quale viene applicata una differenza
di potenziale oscillante a frequenza . Si trascurino gli effetti di bordo e lirraggiamento.

bSoluzione: Il campo elettrico oscillante Ez = E0 eit genera un campo magnetico lungo B = B0 eit :

E iE0 r2 iE0 r
B = : B0 = =
c2 2
c 2r 2c2
che a sua volta genera una correzione al campo elettrico E0 E0 + E1
Z r
2 r2
E = B : E1 = i dr0 B0 (r0 ) = E0 2
0 4c

avendo definito E0 come il campo a r = 0. A sua volta E1 genera un campo magnetico B0 B0 + B1 1

i 1 r i 3 r3
Z
B1 = 2 dr0 r0 E1 = E0
c r 0 16c4
che genera un campo elettrico
r
4 r4
Z
E2 = i dr0 B1 (r0 ) = E0
0 64c4
che genera
i 5 r5 6 r6
B2 = E0 E3 = E0
64 6 64 62 c6
Quindi
 2n
it it
X (1)n r r 
Ez = (E0 + E1 + E2 + E3 + )e = E0 e 2
E0 eit J0
n=0
(n!) 2c c

La fig. 11.1a mostra J0 (x) confrontata con la sua espansione in serie ad ordine 0,2,4,6,8: J0 (x) = 1 x2 /4 +
x4 /64 x6 /2304 + . J0 (x) = 0 a x = r/c ' 2.4: J0 (2.4) ' 1 1.44 + 0.52 0.08 = 0.


Riotteniamo la stessa cosa usando le eq. di Maxwell in forma differenziale. Assumendo Ez = Eeit e
B = Beit dove E e B dipendono da e nelle equazioni di Maxwell in coordinate cilindriche si trova
E 1 i
= iB (rB) = 2 E
r r r c
1 Quando il gioco si fa duro conviene usare il rotore in coordinate cilindriche
Ez 1 (rB )
( z Ez (r)) = , ( B (r))z = .
r r r
76 Capitolo 11. Corrente di spostamento

1 2
0.8
1.8
0.6

|J0 (i1/2 x)|


0.4 1.6
J0 (x)

0.2
1.4
0
0.2 1.2
0.4
1
0 2 4 6 8 0 0.5 1 1.5 2 2.5 3
x x


Figura 11.1: J0 (x) e |J0 ( ix)| (linee nere) confrontate con la loro espansione in serie attorno a x = 0 (linee
tratteggiate, che corrispondono ad includere mano a mano ordini successivi).

e quindi, sostituendo nella seconda il B preso dalla prima, ed usando come variabile x = r/c

E r2 2 E0
r (r )= 2 E : x(xE 0 )0 = x2 E : E 00 + = E
r r c x
Se non ci fosse il secondo termine si avrebbe E 00 = E 0 , la cui soluzione e una funzione speciale chiamata
cos(x) che si trova su tutti i computer (o tavole). Con il secondo termine, che e di tipo attrito, la soluzione e
chiamata J0 (x) che si trova su molti computer (o tavole) ed assomiglia un cos(x) che si smorza.
Si puo fare la stessa cosa piu in generale. Prendo il rot della II equazione di Maxwell

E
2 E + ( E) ( E) = B = 0 J 2
t c
da cui
1 2 1
(2 2 2
)E = 0 J + (11.1)
c t 0
Nel vuoto, assumendo che E abbia solo una componente Ez (r) che oscilla a frequenza , riscrivendo in coordinate
cilindriche
1 2 1 Ez 2
(2 2 2 )E = 0 i.e. r + 2 Ez = 0
c t r r r c
e risolta da Ez J0 (r/c). La funzione di Bessell J0 compare perche siamo in simmetria cilindirica.

Esercizio 132: Cavita risuonante


bSoluzione: (Se racchiudo il condensatore formando una lattina aggiungendo la parete laterale dove E = 0
il campo elettrico interno risolve le equazioni di Maxwell. Cioe la cavita risuona alle frequenze = 2.405r/c,
5.52r/c... Ci sono altri modi con E orizzontale, come si vedrebbe piu facilmente per una lattina cubica. Vedremo
che si trasmettono campi che dipendono anche da z).

Esercizio 133: Effetto pelle


Ad un filo di resistivita e raggio a viene applicata una differenza di potenziale oscillante a frequenza ,
ottenendo una corrente alternata ed un campo elettrico parallelo al filo E(t) = E0 eit . Mostrare che la corrente
si sposta sul bordo, e che questo tende ad aumentare la resistenza effettiva.

bSoluzione: Oltre alla corrente J = E ce la corrente di spostamento Js = 0 E = i0 E. A bassa frequenza


la corrente normale e piu importante della corrente di spostamento (Js /J con = 0 /) che quindi
trascuriamo. Possiamo calcolare come si redistribuisce la corrente calcolando come si redistribuisce il campo
elettrico, in quanto J = E. Assumendo che leffetto sia piccolo, procediamo perturbativamente.
Capitolo 11. Corrente di spostamento 77

La J iniziale uniforme genera un campo magnetico B (r) = 21 0 reit Jz , che per induzione genera una
correzione al campo elettrico E1 parallelo ed opposto a quello iniziale. Utilizzando coordinate cilindriche
r
dEz 0 r r 2
E = B : = B = i Ez0 = i 2 Ez0 dove
dr 2 0

viene chiamata skin depth. Il campo elettrico, scritto in termini del suo valore Eext = Ez (r = a), e

1 ir2 /2 2
Ez (r) = Eext .
1 ia2 /2 2

Lapprossimazione perturbativa E1  E0 vale se  a. Nel rame /0 1018 sec1 . Per a mm si ha  a


fino a  105 Hz.
E interessante calcolare limpedenza per unita di lunghezza, definita come Z = Eext /I, dove I e la corrente
totale: Z a
1 ia2 /4 2
I= Jz 2r dr = a2 Eext .
0 1 ia2 /2 2
Per 0 si ha e si ritrova Z R0 = 1/a2 . In generale Z = R + iL ha una parte complessa (che
corrisponde allimpedenza dovuta a B ) ed una parte reale maggiore di R0 :

1 ia2 /2 2 1 + (a/2)2 i(a/2)2


Z = R0 2 2
= R0 .
1 ia /4 1 + a2 /8 2

Fisicamente, questo e dovuto al fatto che la corrente si concentra verso lesterno aumentando lintasamento e
quindi la resistenza. Quindi conviene lavorare a frequenze abbastanze basse che  a.


Se E1  E0 lesercizio finisce qui. A grandi questo potrebbe non essere vero; in tal caso o si continua
lo sviluppo perturbativo E0 , E1 , E2 , . . ., oppure si risolvono le equazioni di Maxwell in forma differenziale.
Nellequazione donda ricavata allesercizio precedente metto J = E, trascuro ed il termine E dovuto alla
corrente di spostamento, assumo Ez eit , e riscrivo per un Ez (r) in coordinate cilindriche:
 
1 Ez 00 Ez0 2i r 2i
(r ) = 0 Jz = i0 Ez : Ez + = 2 Ez : Ez (r) J0
r r r r

Abbiamo nuovamente ottenuto unequazione differenziale alla Bessel ma questa volta il coefficiente numerico
e immaginario. Infatti anche la correzione al primo ordine ad Ez era immaginaria. La funzione completa e
mostrata in fig. 11.1b ed e qualitativamente simile al risultato al primo ordine.
Consideriamo il limite  a, opposto a quello studiato con il metodo approssimato. Per  a si puo
trascurare il termine Ez0 /r nellequazione differenziale, per cui la soluzione approssimata diventa unesponenziale:
Ez (r) ' Eext er(1+i)/ . Fisicamente, significa che la corrente e grossa solo ai bordi del filo, e penetra per uno
spessore .
9
Come vedremo in seguito esistono le onde. A frequenze abbastanza grandi, > 10 Hz, hanno lunghezza
donda umana ed e possibile utilizzarle per trasportare energia dentro cavita metalliche (guide donda). Il
fatto che i campi penetrino dentro il metallo solo per un piccolo spessore e che quindi le correnti parassite
J = E dentro al metallo siano trascurabili diventa un vantaggio. In questo modo si riesce a trasportare grandi
potenze con poca dissipazione di energia per effetto Joule. Tecnologicamente risulta piu semplice trasportare la
corrente a basse frequenze in cavi di rame che ad alte frequenze in tubi di rami.

Esercizio 134: Filo conduttore interrotto


[Dal compito del 4/4/2003] Un filo conduttore rettilineo e cilindrico, di resistivita , raggio a e lunghezza `  a,
viene connesso ad un generatore, in modo che nel filo passa la corrente I = I0 cos t.

a) Si calcoli il campo magnetico in tutto lo spazio (assumendo un filo di lunghezza infinita) e il campo
elettrico per r < a nellassunzione di corrente lentamente variabile. Si discuta a posteriori la condizione
necessaria a questa approssimazione.
78 Capitolo 11. Corrente di spostamento

Si taglia via un tratto h  a del filo e si regola di nuovo il generatore in modo che passi la stessa corrente di
prima.
b) Si risponda di nuovo alla domanda a); come cambiano i campi prima e dopo linterruzione del filo?
c) Linterruzione del filo si puo schematizzare come linserimento in serie di una impedenza Z. Si stimi e
discuta il valore di Z in funzione di .

bSoluzione:
a) Per 0 si ha un campo elettrico uniforme E0 = I/a2 . La corrente genera un campo magnetico
B = 0 rI/2a2 per r < a. Come gia visto in esercizi precedenti, vi sono correzioni di ordine relativo
0 , che diventano significative ad alte frequenze.
b) Prendendo la divergenza della IV equazione di Maxwell B = 0 (J + 0 E) si impara che la somma
delle correnti elettrica e di spostamento si conserva, anche alla superficie di discontinuita. Quindi il
campo magnetico non cambia rispetto al punto a). La conservazione della corrente totale consente di
determinare il campo elettrico Ev nella regione di vuoto: da 0 Ev = J + 0 E0 segue Ev = E0 [1 + i/(0 )].
Infatti, si puo schematizzare il sistema come un condensatore inserito fra due resistenze: sulle superfici si
deposita una densita di carica tale che = j, per cui Ev = E0 + /0 .
c) La parte vuota si comporta come un condensatore di capacita C = 0 a2 /h. Al crescere della frequenza
questo condensatore ha una induttanza parassita L ' 0 h/8. Per trovare L si puo calcolare lenergia
magnetica contenuta nel condensatore
Z a
B2 (0 rI/2a2 )2 0 hI 2 LI 2
Z
UM = dV = dr 2r =
20 0 20 16 2
Quindi, limpedenza associata e Z = iL + 1/iC.
Linduttanza diventa rilevante solo ad alte frequenze, quando la nostra
approssimazione di corrente lentamente
variabile cessa di valere. Ad esempio Z = 0 per = 1/ LC = 2 2c/a: per valori di cos alti, I varia in
modo significativo nel tempo che la luce impiega ad attraversare il filo. A frequenze cos alte ci sono effetti
addizionali:
p il filo irraggia; la corrente non e piu uniforme a causa delleffetto pelle. Infatti, 0  1 implica
= 2 c0 /  c/, che per Z = 0 e comparabile al raggio a del filo.

Esercizio 135: Due cilindri cavi


[Da un compito del 1987] Due cilindri cavi coassiali di raggi a < b sono percorsi da correnti uguali I(t) = I0 sin t
in verso opposto, distribuite uniformemente sulle superfici. a) Trascurando la corrente di spostamento calcolare
B. b) Mostrare che E puo avere una sola componente non nulla. c) Calcolare E assumendo che valga zero sul
cilindro esterno. d) Calcolare la corrente di spostamento Is . e) Discutere come deve essere calcolato B nei tre
casi Is  I0 , Is < I0 e Is I0 . f) Caso numerico: a = 1 mm, b = 1 cm, I0 = 2 A, = 1000 Hz.

bSoluzione:
a) B = 0 per r < a ed per r > b. Nella zona intermedia 2rB = 0 I.
b) Per simmetria cilindica E puo dipendere solo da r. Siccome non ce carica Er = 0. Siccome Bz = 0 si ha
2rE = 0. Quindi lunica componente e Ez (r), generata da E = B.
c) Riscrivendo in componenti la II equazione di Maxwell si ottiene Ez /r = B /t = 0 I0 cos(t)/2r
per a < r < b. La soluzione con Ez (b) = 0 e

0 r>b
1
Ez (r) = 0 I0 cos(t) ln(r/b) a < r < b
2
0 I0 cos(t) ln(a/b) r < a

d) La corrente di spostamento j s 0 E vale


b
b2 a2 2
Z
Is = 0 Ez 2r dr = I
0 4 c2
Capitolo 11. Corrente di spostamento 79

e) Se Is  I0 il conto perturbativo fatto finora e accurato. Se Is < I0 si puo iterare B E B


E B . . . aggiungendo i termini perturbativi successivi. Se Is I0 occorre risolvere le equazioni di
Maxwell. Da un punto di vista matematico questo e analogo a calcolare 1/(1 ) epandendolo attorno ad
 = 0. Se   1 bastano pochi termini della serie di Taylor 1 +  + 2 + . Se  <
1 ne servono tanti. Se
 1 la serie perturbativa non funziona.
f) Siccome b/c  1 siamo nel caso Is  I0 .

Esercizio 136: Carica in moto


Una carica q si muove lungo lasse z con velocita v  c costante. Calcolare il campo magnetico che essa genera.
Spiegare in che modo tante cariche q che formano una corrente i continua producono approssimativamente un
campo magnetico che non dipende dal tempo.

bSoluzione: Senza includere la corrente di spostamento si ha j 6= 0 solo in coincidenza della carica, ed il


problema non ha senso. Includendo j s , essa e lunica sorgente di B in tutto lo spazio vuoto. Qui risolviamo
il problema con un calcolo esplicito. Un modo alternativo di risolvere il problema consiste nel notare che le
equazioni di Maxwell complete sono relativisticamente invarianti, calcolare i campi nel sistema rispetto al quale
la carica e ferma, ed applicare le trasformazioni di Lorentz dei campi.
Il campo magnetico ha solo componente B (z vt, r). E sufficiente calcolarlo a z = 0 per t ed r generici.
Integriamo la IV equazione di Maxwell, B = 0 0 E lungo la superficie di un anello di raggio r giacente
a z = 0. Si ottiene I
0 is
B ds = 0 0 (E) i.e. B (r) =
2r
dove is e la corrente di spostamento che attraversa lanello:

d q q d vt qva2
is = (j s ) = 0 (E) = (1 cos (t)) = =
dt 2 2 dt a2 + v 2 t2 2(a2 + t2 v 2 )3/2

Quindi B (z, r, t) = 0 qvr/4(r2 + (z vt)2 )3/2 . Per z = 0 e massimo a t = 0, cioe mentre q sta passando.
Una successione di cariche con densita produce in media una corrente continua i = v. Infatti il campo
magnetico generato vale Z +
dz 0 vr 0 i
B = 2 + (z vt)2 ]3/2
=
4[r 2r
che e la ben nota formula per il campo magnetico generato da una corrente i.
Capitolo 12

Onde e oscillazioni

Unonda elettromagnetica piana polarizzata linearmente nel vuoto e descritta da


E
E = E 0 sin(k r t), B = B 0 sin(k r t), c= =
B k
con E 0 , B 0 , k ortogonali. Frequenza: = /2. Periodo: T = 1/. Lunghezza donda; = 2/k = c/.
Densita e il flusso di energia:
0
u = (E 2 + c2 B 2 ) S = 0 c2 E B
2
Quindi, in media
1 c
hui = 0 E02 , hSi = 0 E02 = chui
2 2

Esercizio 137: Sorgenti di onde


D

Quattro sorgenti identiche disposte come in figura emettono onde di /2


lunghezza donda . Due ricevitori sono situati a distanza r  come R1 A /2 B /2 C
in figura. Calcolare il rapporto fra le potenze ricevute dai ricevitori.
Cosa cambia se B viene spenta? Se D viene spenta?
R2

bSoluzione: Si ha IE |EA + EB + EC + ED |2 :
2 2
I1 |eik(r/2) + eikr + eik(r+/2) + eik r + /4 |2 | 1 + 1 1 + 1|2
2 2 2 2
I2 |eik r + /4 + eikr + eik r + /4 + eik(r+/2) |2 |1 + 1 + 1 1|2
p
avendo usato k = 2, eik/2 = 1, e r2 + 2 /4 ' r, Quindi I1 /I2 = 0/4. Se B viene spenta I1 /I2 = 1/1.
Se D viene spenta I1 /I2 = 1/9. Quindi il ricevitore 1 non sa dire se viene spenta B o D, mentre il ricevitore D
puo dirlo.

Esercizio 138: Ricevitore di onde


Calcolare la f.e.m. attraverso un cicuito quadrato di lato /4 disposto nellasse xy, quando viene attraversato
da unonda elettromagnetica che si propaga lungo x di lunghezza donda ed ampiezza E0 polarizzata lungo y.

bSoluzione: Ci sono 2 modi. La circuitazione del campo elettrico vale


I

E = E ds = E0 [sin(k0 t) sin(k(0 + /2) t)] = E0 sin t
2
Il flusso del campo magnetico vale

/2 B 0 2
Z Z
= dSBz = dxB0 sin(kx t) = cos t
2 0 2

80
Capitolo 12. Onde e oscillazioni 81

quindi E = viene uguale a prima, come si verifica usando = 2c/ e E0 = cB0 .


I due E devono venire uguali, in quanto unonda piana e soluzione della II equazione di Maxwell E = B.

Esercizio 139: Luce solare


Calcolare i valori numerici del vettore di Poynting, i campi elettrici e magnetici, densita di energia, pressione
della luce solare, sapendo che essa fornisce unenergia K = 1366 J m2 s1 .

bSoluzione: La media del vettore di Poynting e uguale alla costante solare: hSi = K . Quindi pressione e
densita di energia valgono

hSi J N
hpi = chgi = hui = = 4.5 106 3 = 4.5 106 2 .
c m m

Il campo elettrico vale E0 = 1000V/m (usando 0 = 8.85 1012 C2 /N m2 e N m = V C). Il campo magnetico
vale B0 = E0 /c = 3.36 106 Tesla.
Il sole emetta luce gialla di frequenza 0.5 1015 Hz e quindi ha = c/ 0.6m. La luce e composta da
fotoni, ciascuno dei quali ha energia h 1019 J (la costante di Planck vale h = 6.62 1034 J/ s): quindi la
luce del sole ne contiene circa 1022 / m2 s. Dalla relazione h kB T si ricava la temperatura del sole. Nota la
temperatura TS = 5800 K ed il raggio RS = 7 108 m del sole, la potenza emessa puo venir calcolata teoricamente,
sapendo che un corpo nero a temperatura T irraggia per unita di superficie una potenza

dW 2 kB
4
8 W
= T 4 dove = 3 2 = 5.670 10
dS 60h c m2 K4

e detta costante di Stefan-Boltzmann. Quindi il sole irraggia una potenza WS = SS TS4 = 4RS2 TS4 =
4 1026 W. La terra e a distanza d 1.5 1011 m dal sole e riceve frazione /4 = R2 /4d2 = 6.8 105 = 1/14500
della potenza totale. La potenza per unita di superficie vale K = (RS /d)2 TS4 = 1366W/m2 .


E interessante proseguire calcolando la temperature dei pianeti, anche se questo non e solo un esercizio di
elettromagnetismo. Anche la terra e approssimabile come un corpo nero a temperatura TE , calcolabile sapendo
che in condizioni di equilibrio irraggia tutta lenergia p che riceve dal sole: tenendo conto che la terra e una sfera
r2 K = 4r2 TE4 da cui TE = TS (/4)1/4 = RS /2dTS 280 K, che e una buona approssimazione. E
buona perche per via di un teorema generale, emissivita e riflettivita sono uguali e quindi si cancellano. Non e
perfetta per via delleffetto serra: le zone interne dellatmosfera sono piu calde. La luna ha la stessa temperatura
media TS della terra, ma non ha atmosfera: quando il sole e a picco raggiunge Tday = 2TE 400 K, di notte
scende fino a T  Tday .
in piu della terra, e quindi la sua temperatura e circa 25% piu bassa, TM 230K.
Marte e dista dal sole il 50%
Ma a mezzogiorno raggiunge 2TM 320 K e lacqua si puo scongelare.


E interessante calcolare lenergia ottenibile da pannelli solari. Come detto sopra il sole a picco produce
K = 1.366kW/m2 : tenendo conto che pannelli solari hanno efficienza  0.1, la potenza prodotta vale
W = K = 1kW/8m2 . Lenergia totale prodotta in un anno vale E = K epsilon yr/2/3 = 200kWh/m2 , dove
1/2 tiene conto che di notte non ceil sole, ed 1/3 dellinclinazione del sole hcos2 i < 1 e delle nuvole. Siccome
un kW h costa circa 0.15Euro, il risparmio prodotto e di circa 30Euro/yrm2 (se lenergia viene utilizzata). Il
costo di installazione e di circa 1000Euro/m2 .

Esercizio 140: Vettore di Poynting


Verificare che il vettore di Poynting descrive veramente la variazione di energia in varie geometrie.

bSoluzione:
82 Capitolo 12. Onde e oscillazioni

1. Una capacita cilindrica (lato a, spessore h) a frequenze non troppo alte ha campo elettrico Ez (t) e la
corrente di spostamento genera un campo magnetico lungo dato da B = r2 Ez /c2 /2r = rEz /2c2 .
Lenergia contenuta in un raggio r vale
0 2
U ' (r2 h) E U = r2 h0 Ez Ez
2
Il vettore di Poynting vale
I
r
Sr = 0 c2 Ez B = 0 Ez Ez Sr = 2r hSr = U
2

in accordo con u + S = 0, che possiamo anche verificare usando S = (1/r)d(rSr )/dr.

2. Un filo resistivo di lunghezza h ha un campo elettrico costante Ez = V /h. La corrente j = E dissipa


una potenza U = h r2 jE e genera un campo magnetico B = 0 j r2 /2r. Quindi il flusso del vettore
di Poynting Sr = rjE/2 vale Sr = h r2 jE = U .
H

3. Un cavo coassiale porta corrente continua a tensione V su di una resistenza R, che congiunge le due
armature. Quindi ci passa una corrente i = V /R, che dissipa una potenza W = iV . Al suo interno
contiene un campo elettrico ed un campo magnetico dati da

V 0 i iV
Er = B = Sz = 0 c2 Er B =
r ln r2 /r1 2r 2r2 ln r2 /r1

Il flusso del vettore di Poynting vale


I Z r2
Sz = Sr 2r dr = iV = W
r1

4. Un solenoide rettilineo infinito contiene un campo magnetico Bz = 0 nI e quindi un campo elettrico


2 2 2
H r = Bz r/2. Il vettore di Poynting vale Sr = Bz Bz r/20 . U = r Bz /20 , U = r Bz Bz /0 ,
E
Sr = 2r Sr = U .

5. Paradosso di Feynman. Un disco libero di ruotare lungo lasse z contiene una carica q ed un solenoide
percorso da una corrente continua che genera un campo magnetico Bz . Si interrompe la corrente: la
variazione di Bz genera un Er che mette in rotazione il sistema. Che ne e della conservazione del momento
angolare?

6. Un elettrone in un atomo di idrogeno ruota a distanza a dal protone con velocita determinata da
me v 2 /a = qe2 /4a2 0 cioe

v2 qe2 1 r2 1
2
= 2 2
e2 = per a = 0.53A.
c 40 me c a a 137.2

La lunghezza re e detta raggio classico dellelettrone (ma non e il raggio dellelettrone), in quanto e il
raggio che avrebbe un elettrone se la sua massa fosse dovuta allelettromagnetismo:

qe2 qe2
U = me c2 per re = = 2.82 1013 cm
40 re 40 me c2

(per abbreviare spesso si usa e2 qe2 /40 invece di qe ). Lelettrone, ruotando attorno al protone, genera
anche un campo magnetico campo magnetico di tipo dipolare, B 0 /r3 dove il dipolo magnetico vale
= a2 i = a2 (qe /2) = (qe /2m)L (L = ma2 ). Quindi S Er Bz /0 . La densita di momento vale
g = S/c2 . Il momento angolare elettromagnetico vale

qe2 L v2
Z
re L
Lem dV r g a4 g(a) 2
L 2
L
mc 0 a a c 1372

ed e quindi una frazione trascurabile del momento angolare meccanico.


Capitolo 12. Onde e oscillazioni 83

Esercizio 141: Rilfessione di onde in una corda


Si connettono due corde con diverse densita lineari e 0 = n2 (attenzione a non confondere con la lunghezza
donda). Studiare cosa succede quando un onda trasversale arriva al punto di congiunzione.

bSoluzione: Metto la corda lungo lasse x. Unonda produce una distorsione y(x, t) rispetto al valore di
equilibrio y = 0. La legge del moto per y(x, t) e
2y 2y
ma = F : 2
= 2
t x
(compare y 00 perche la tensione
p non richiama una corda dritta, con y 0 costante). La tensione e la stessa
dovunque. La velocita v = /(x) = k/ cambia imrovvisamente sul punto di congiunzione, che mettiamo a
x = 0. Siccome v 0 = v/n, nellanalogo elettromagnetico n sara lindice di rifrazione.
Non serve studiare un pacchetto donda. Cerco la soluzione di onda piana: oltre allonda incidente y =
a ei(kxt) aggiungo una componente Riflessa ed una T rasmessa
 i(kxt)
e + R ei(kxt) per x < 0
y=a i(k0 xt)
Te per x > 0

E intuitivamente ovvio che al punto di congiunzione fra le due corde y e y 0 sono continui. Matematicamente
segue dal fatto che lequazione donda contiene due derivate rispetto a x (e quindi rimarra vero in casi meno
intuitivi, come nelle equazioni di Maxwell o Schroedinger). Imponendo la continuita si trova
2k 2 k k0 n1
1+R=T k(1 R) = k 0 T T = = R= =
k + k0 1+n k + k0 n+1
Si ha T > 1 se n < 1: questo e sensato, e dovuto al fatto che nel passare da una corda grossa and una corda
piccola lampiezza dellonda si amplifica.
Non si deve amplificare la sua energia. La potenza media dellonda incidente (che corrisponde al vettore di
Poyinting nellanalogo e.m.) vale WI = vu = 12 v(a)2 : infatti 14 (a)2 e la densita di energia cinetica media,
che e in media eguale allenergia potenziale. La potenza riflessa vale WR = WI R2 e quindi quella trasmessa deve
valere WT = WI WR .1 Infatti, facendo il calcolo esplicito sfruttando il fatto che W v a2 (1/n) n2 a2
si ottiene
WT 4n
= T 2n = <1
WI (1 + n)2
Come giusto viene WT /WI 1, massimo per n = 1.

Esercizio 142: Rifrazione


Fare incidenza normale, mostrando che la cons. energia richiede un onda riflessa. Calcolare forza sentita da
separazione vuoto/dielettrico I T + R mostrando che si ottiene 2 : 1 : 0 per riflettente:assorbente:trasmettente

bSoluzione:

Esercizio 143: Rifrazione


Due dielettrici sono separati dal piano x = 0. Unonda elettromagnetica incide lungo k = (kx , ky , 0) con E
polarizzato lungo z. Cosa succede?

bSoluzione: In un dielettrico i campi e.m. si propagano in modo simile al vuoto, con lunica differenza che
c c/n:
1 1 c2 E c
c2 = 2, =c =
0 0 0 n B n k
1 Nella zona dove ci sono due onde, la potenza media e la somma delle potenze media delle due onde: W = W W . Fisicamente,
I R
questo e chiaro in quanto usare onde piane e un modo utile per descrivere un problema nel quale si invia un pacchetto donde, che
viene parzialmente trasmesso e riflesso. Per vederlo matematicamente con onde piane, occorre notare che il termine di interferenza
fa zero, se mediato sullo spazio e sul tempo, usando formule tipo 2 sin(kx t) sin(kx t) = cos(2kx) cos(2t).
84 Capitolo 12. Onde e oscillazioni

Le condizioni al bordo fra due dielettrici sono B 1 = B 2 e E1= = E2= , 1 E1 = 2 E2 . Assumo che il piano di
separazione sia il piano yz a x = 0, e che londa si propaghi nel piano xy con il campo elettrico polarizzato
lungo z. Per prima cosa, le condizioni al bordo, dovendo essere vere per ogni t, implicano che londa incidente,
londa riflessa (nel mezzo 1) e quella trasmessa (nel mezzo 2) hanno la stessa frequenza. Dovendo poi essere
vere per ogni y, si ha che tutte le onde hanno lo stesso ky : ky kyi = kyr = kyt . Siccome i moduli dei vettori k
sono dati da ki = ni /c si ha ki /kt = n1 /n2 e quindi si impara che londa trasmessa si inclina:

ky ky
sin t = sin i = : n2 sin t = n1 sin i
kt ki
Per quanto riguarda le ampiezze dellonda le condizioni di raccordo sono

Ei + Er = Et (da E o da Bx )
kx Ei kx Er = kxt Et (da Ey0i + Ey0r = Ey0t o da Byi + Byr = Byt usando B = k E/)

e quindi
kx kxt 2kx
Er = Ei Et = Ei
kx + kxt kx + kxt
che e identica a quanto ottenuto nellesercizio precedente su onde due corde con un capo in comune.
Allo stesso modo funziona anche il rapporto fra potenze trasmesse ed incidente. La potenza trasmessa vale
WT = vu dove u e la densita media di energia. In un dielettrico essa vale

 B2  
u = h E2 + i = h [E 2 + B 2 v 2 ]i = E 2
2 20 2 2

Tenendo conto che v 1/n e che u E 2 n2 E 2 si ha nuovamente

WT 4n
= T 2n = < 1.
WI (1 + n)2

Esercizio 144: Riflessione da un metallo


Unonda piana di frequenza con polarizzazione lineare parallela alla superficie incide sulla superficie di un
metallo di conducibilita .

a) Determinare la forma dellequazione delle onde elettromagnetiche nel metallo.

b) Studiare la penetrazione dellonda nel metallo.

c) Calcolare la riflettivita del metallo (rapporto tra intensita riflessa ed incidente)

d) Trovare il flusso di energia verso linterno del metallo e confrontarlo con la potenza dissipata per effetto
Joule.

bSoluzione:

a) Mettendo J = E e = 0 nella (11.1) si ottiene

1 2 E
(2 )E = 0
c2 t2 t

Per onde monocromatiche E(x, t) = E(x)eit lequazione si riduce a

2
(2 + )E = i0 E
c2
Con metodi simili si trova che il campo magnetico soddisfa alla stessa equazione, ottenuta rimpiazzando
E B, che ha solo componente By .
Capitolo 12. Onde e oscillazioni 85

b) Supponiamo che il conduttore si estenda per x > 0 e che E sia polarizzato lungo z. Quindi soddisfa
lequazione (per x > 0)
2 2
( 2 + 2 )Ez = i0 Ez
x c
La soluzione e quindi Ez = ei(qxt) Ez (0) dove q soddifsa la relazione di dispersione
2
 2
q1 q22 = 2 /c2
q 2 = 2 + i0 i.e.
c 2q1 q2 = 0
dove q = q1 + iq2 . q2 descrive lo smorzamento dellonda, provocato dalle correnti parassite. La soluzione
esplicita per q non e illuminante, anzi fa piuttosto schifo. Numericamente il rame ha 0 0 c2 = /0 =
6.6 1018 Hz, nel range di frequenza dei raggi X, circa tre ordini di grandezza maggiore delle frequenza della
luce visibile. Studiamo due casi limite.
Ad alte frequenze  0 q1 ha circa il valore di vuoto, q1 ' /c e leffetto della conducibilita e
descritto da un piccolo q2 ' 0 c. Quindi
Ez (x, t) = Ez (0)eikxit e0 cx dove k = /c = 2/.
Cioe londa si propaga in maniera simile a quanto fa nel vuoto, ma smorzandosi su una distanza
tipica 1/q2  . Per il rame 1/0 c 0.4 1010 m.
p
A basse frequenze  0 si ha q1 ' q2 ' 0 /2 1/ e ritroviamo il caso dellesercizio di
pagina 70: londa si smorza in qualche lunghezza donda. Questo e tipicamente il caso di metalli alle
frequenza della luce visibile, che non penetra nel metallo e viene quindi riflessa (a meno di un piccolo
assorbimento).
c) Per x < 0 i campi soddisfano lequazione donda nel vuoto, con soluzione generale
Ez (x < 0, t) = EI eikxit + ER eikxit .
EI ha il significato fisico di onda incidente, mentre EI quello di onda riflessa. Bisogna calcolare ER /EI .
Ad alte frequenze tutta londa entra (ER ' 0) e dissipa la sua energia per effetto Joule.
A basse frequenze il campo dentro il metallo e
Ez (x > 0, t) = ET eitix/ ex/
Siccome E risolve unequazione di secondo grado in x, per raccordare le soluzioni occorre che Ez e
Ez /x siano continue a x = 0, cioe
i1
EI + ER = ET , ik(EI ER ) = iqET ' ET

da cui
ER k 1 i ET 2k
= , =
EI k + 1 + i EI 1 + k + i
Poiche lintensita delle onde sono proporzionali ai moduli quadri dei campi, la riflettivita R e il
modulo quadro dellultima espressione:

1 + (k 1)2 + 0 20
R= = <1
1 + (k + 1)2 + 0 + 20
Nella figura questa espressione approssimativamente valida per  0 (linea tratteggiata) e con-
frontata con il risultato completo (linea continua):
1
0.8
0.6
R

0.4
0.2
0
10-4 10-3 10-2 10-1 1 10 102 103 104
/0

Per k  1 si ha R ' 1, cioe tutta lenergia viene riflessa. Infatti se la lunghezza di penetrazione e
trascurabile londa non puo dissipare energia per effetto Joule, e deve quindi tornare indietro.
86 Capitolo 12. Onde e oscillazioni

d) A basse frequenze abbiamo ottenuto WR /WI = R2 . Quindi la potenza trasmessa dovrebbe valere WT =
T WI con
4k
T =1R=
1 + (k + 1)2
e questa potenza dovrebbe venir dissipata dalle correnti parassite nellinterno del metallo. Verifichiamolo.
Dentro il metallo

eix/it
 
2kEI
Ez (x > 0) = EI 2kex/ Re = ex/
(1 + k) cos(x/ t) + sin(x/ t)
1 + k + i 1 + (k + 1)2

La potenza media Wdiss dissipata per unita di superficie e


Z Z
2(k)2 EI2 (k)2 Ei2
Wdiss = dx hEz2 t i = 2
dx e2x/ =
0 1 + (k + 1) 0 1 + (k + 1)2

avendo usato h(A cos +B sin)2 it = (A2 + B 2 )/2. Usando k 2 = (/c)(2/0 ) = 2/(0 c) = 20 c
otteniamo il risultato atteso
4k 0 cEI2
Wdiss = = T WI .
1 + (k + 1)2 2

Esercizio 145: Onde adiabatiche


Una corda uniforme di massa m e lunghezza ` e appesa nel campo di gravita g. Come si propagano le onde?
p
bSoluzione: La tensione ad un punto z e = mgz/`, quindi la velocita delle onde e v(z) = / = gz. Se
 ` le onde si propagano adiabaticamente, cioe con v = v(z), e quindi
Z `
dz p p
t(` z) = = 2[ `/g z/g]
z gz
p
Un corpo che cade impiega un tempo t0 (` z) = 2(` z)/g.

Esercizio 146: Guida donda


Guida donda rettangolare disposta lungo z e di lati a lungo y e b lungo x

bSoluzione: Come un cavo coassiale senza filo centrale, in modo da evitare elettroni che irraggiano. Usare
un tubo grosso vuoto conviene quando si devono trasportare grosse potenze. Le condizioni al bordo su di un
conduttore perfetto sono E= = 0 e B = 0. Provo la soluzione
n
Ey = E0 sin kx x ei(kz zt) kx =
a
Soddisfa a E = y Ey = 0. Metto n = 1. Per ottenere unonda trasversa rispettando B = 0 aggiungo

Bx = B0 sin kx x ei(kz zt)

Tuttavia ha divergenza diversa da zero: e possibile mostrare in generale che le onde in una cavita singola non
sono trasverse (le componenti trasverse soddisfano a 2 V = 0: in un cavo coassiale V puo essere diverso sui
due bordi, con un bordo solo lunica soluzione e V = cte). Occorre quindi aggiungere un campo longitudinale.
Mettiamo un Bz
kx
Bz = i B0 cos kx x ei(kz zt)
kz
I fattori relativi fra Bz e Bx sono tali che B = z Bz + x Bx = (ikz ikz )Bz = 0, in particolare B= e
costante sul bordo, dove E = 0. Le equazioni E = B, componenti x e z implicano kz E0 = B0 . Quindi
le linee di B circolano attorno al massimo di Ey .
Capitolo 12. Onde e oscillazioni 87

Per finire, per via della relazione fra E0 e B0 , lequazione ( B)y = Ey /c2 implica
p p
kx2 + kz2 = 2 /c2 i.e. kz = (/c)2 (/a)2 = (/c) 1 (c /)2

che puo essere immediatamente derivato dallequazione donda per Ey .


Sotto la frequenza critica c = c/a kz diventa immaginario, il che significa che londa si attenua come e|kz |z .
Questo viene chiamato modo TE in quanto Ez = 0. Esistono altre onde TM con Ez = E0 sin(nx/a) sin(my/b)
con n, m 1 che quindi ha una frequenze di cut-off maggiore: di solito si lavora in modo che solo TE10 possa
propagarsi. Quindi necessariamente cm i.e. microonde.
Le velocita di fase e di gruppo sono
c d p
vf = =p >c vg = = c 1 (c /)2 < c
kz 1 (c /)2 dkz

La densita di energia si muove con velocita vg . Infatti

1 0 E02 k2 B 2 k2 + k2
   
1 0 0
huix,y,t = + (1 + x2 ) 0 = 1 + z 2 2x E02 = E02
2 2 2 20 kz 2 8 /c 4
R Ra
La componente x del vettore di Poynting, Sx , e diversa da zero, ma Sx 0 dx sin kx x cos kx x = 0. Lungo z

11 2 0 kz
hSz ix,y,t = 0 c E0 B0 = E02 c2 = u vg
22 4
La lunghezza donda nella guida vale
2 0 2c
g = =p 0 =
kz 1 (0 /2a)2

Accoppiatore unidirezionale: due buchi separati di /4, che diventa /2 (fuori fase) o 0 (in fase) a seconda
della direzione di propagazione.
Un modo per capire fisicamente lesistenza della frequenza di taglio e mettere un filo nel centro della guida;
per avere E = 0 sui bordi si aggiungono infinite immagini. Avevamo visto che se nel filo ceuna carica costante
il campo muore esponenzialmente. Se invece la carica oscilla i campi si possono sommare costruttivamente per
via del ritardo di fase; in direzioni tali che la differenza di distanza fra due fili a sin e uguale a (n 1/2)0 .
Prendo n = 1. Sommando la lunghezza donda nella guida vale g = 0 / cos che equivale alla formula di
prima. Si ha g > 0 . Questo e quello che si ottiene anche ragionando in termini di raggi di luce che rimbalzano
con angolo riflettendosi fra i bordi nella guida, se si tiene conto che ad ogni riflessione i campi si invertono.
La velocita di gruppo e ridotta in modo corrispondente.

Esercizio 147: Cavita risuonante


Stimare il Q

bSoluzione:
Energia immagazzinata
Q 0
Potenza dissipata
cioe u(t) e0 t/Q e E(t) ei0 t) t/2Q e quindi lo spettro di energia e
1
|E()2 |
( 0 )2 + (0 /2Q)2
p
La stima e Q V /S 1000 dove = 20 c2 / e la lunghezza di pelle.

Esercizio 148: Pressione di radiazione


Una sfera di raggio R si trova a distanza r dal sole. a) Si calcoli la forza sulla particella dovuta alla radiazione
solare, assumendo che questa venga tutta assorbita. La sferetta abbia densita = 1g/ cm3 e sia soggetta anche
88 Capitolo 12. Onde e oscillazioni

alla attrazione solare. b) Si determini il raggio R0 per cui tutte le sferette con raggio inferiore sono espulse dal
sistema solare.

bSoluzione: La pressione di radiazione e diretta lungo la direzione dellonda


Z
prad = u Frad = prad dS = R2 u.

cioe conta solo la dimensione dellombra, e non la forma delloggetto. Se invece di essere perfettamente assorbente
fosse perfettamente riflettente la componente radiale della forza diventerebbe 1 2 maggiore a seconda della
sua forma. Avevamo visto che hui = (d2 /r2 )4.5 106 N/ m2 , dove d e la distanza della terra dal sole.
Alla stessa distanza la forza di gravita produce unaccelerazione a = GM/d2 = 0.006m/s2 e quindi una forza
Fgrav = ma = 43 R3 a. Si ha Fgrav < Frad per R < 3hui/4a = 0.57 103 kg/ m2 . Quindi, per = 1000kg/ m3 ,
si ha R0 = 5.7 107 m.

Esercizio 149: Velocita di gruppo


Illustrare in un caso semplice la velocita di gruppo.

bSoluzione: Consideriamo la sovrapposizione di due onde con eguali moduli di E


   
k1 k2 1 2 k1 + k2 1 + 2
E = A[sin(k1 z 1 t) + sin(k2 z 2 t)] = 2A cos z t sin z t
2 2 2 2

Linviluppo delle due onde produce unonda lunga che si muove con velocita vg = /k ' d/dk. Definendo
vf = /k c/n(k) si ha vg = c/(n + dn/d).

Esercizio 150: Pulsar


Una pulsar emette brevi impulsi a radio frequenze. Sapendo che 1 = 400 MHz arriva t = 1 s dopo 2 = 1000
MHz, e che n2 = 1 Ne e2 /0 me 2 con Ne 3 104 / m3 calcolare la distanza della pulsar.

bSoluzione: Fra la pulsar e la terra la presenza di elettroni liberi rende il mezzo dispersivo. Riassumo la
derivazione della frequenza di plasma. Unelettrone libero si muove secondo me x = qe E generando un dipolo
p = ex = E con = e2 /me 2 . La densita di polarizzazione del mezzo e quindi P = Ne p, per cui
s
2  P Ne e2 p 2 Ne e2
n =1+ =1 2
=1( ) dove p = = 104 Hz
0 0 E 0 m 0 me

e detta frequenza di plasma in quanto e anche la frequenza delle oscillazioni meccaniche del plasma.
Mostriamo adesso che frequenze basse viaggiano piu lente, arrivando con un ritardo t = D/vg1 D/vg2
(dove vg e la velocita di gruppo la velocita di fase ha il comportamento opposto). Ricordando che n c/vf =
ck/ i.e. k = n/c
1 dk 1 d(n) 1 dn 1 p2
= = = (n + ) ' (1 + )
vg d c d c d c 2 2
avendo approssimato n ' 1 p2 /2 2 in quanto n 1 1011 . Quindi, ricordando = 2

2c t
D= 5000 ly
p2 /12 p2 /22

Esercizio 151: Interferenza


(dal compito del 20/6/2003) Una nave percorre una rotta parallela alla costa alla distanza di circa 100 km da
questa e alla velocita di 18 nodi. Un marinaio a bordo della nave sta ascoltando un programma musicale, sulla
frequenza di 1200 kHz, trasmesso da una stazione situata sulla costa, in direzione perpendicolare alla rotta.
Capitolo 12. Onde e oscillazioni 89

Laltezza del segnale varia regolarmente col tempo apparendo e scomparendo e lintervallo tra il massimo ed
il minimo e 2 minuti. Nei momenti di massima intensita, il segnale ricevuto dallantenna e stimato a circa 12
mV/m, pari a circa 8 volte il livello di rumore. Si fa lipotesi che una seconda stazione costiera vicina alla prima
abbia iniziato a trasmettere per errore in fase e sulla stessa frequenza. Inquadrando i fenomeni nellambito
dellesperimento di Young stimare, fornendo i risultati numerici: a) La distanza d tra le due stazioni; b) La
potenza emessa da ciascuna delle due stazioni.
Note: 0. Si trascuri ogni effetto dovuto alla sfericita della Terra. 1. Un nodo e pari ad un miglio nautico
(circa 1.8 km) allora. 2. Si consideri solo la cosiddetta portante come unonda monocromatica. 3. Si intende
che il segnale scompare quando e inferiore al livello di rumore. 4. Si ipotizza, salvo verifica, che la distanza
tra le stazioni sia molto minore della distanza tra queste e la nave.

bSoluzione: a) La lunghezza donda del segnale ricevuto e = c/ = 250 m mentre la distanza tra due massimi
(o minimi) del segnale e a = 2vt = 2160 m.
Le onde emesse da due sorgenti a distanza d ricevute ad angolo sono in fase se n = d sin = da/D.
Quindi la distanza tra due massimi consecutivi vale a = D/d (formula di Young) da cui d = D/a = 11.6 km.
b) Sia E0 il valore del campo elettrico corrispondente alla soglia del rumore; se E1 e E2 sono le ampiezze dei
campi elettrici emessi dalle due stazioni, le condizioni sui massimi e minimi di intensita sono

E1 + E2 = 8E0 , E1 E2 = kE0

con k [0, 1]. Risolvendo il sistema si ottiene:

E1 = (8 + k)E0 /2, E2 = (8 k)E0 /2

Lintensita media dellonda ricevuta puo essere messa in relazione con la potenza della stazione emittente e con
il campo elettrico allantenna.
W 1
I= 2
= c0 E 2 , W = (2c0 ) D2 E 2 = (1/60)D2 E 2
4D 2
Ne segue che la potenza delle due stazioni vale

W1 = (8 + k)2 D2 E02 /240, W2 = (8 k)2 D2 E02 /240

Sostituendo i valori numerici, al variare di k, si ha:

6 kW < W1 < 7.6 kW, 6 kW > W2 > 4.6 kW

Il valore esatto dipende da k, che non e noto.


Capitolo 13

Irraggiamento

Una carica q in moto non relativistico irraggia


qe 1 dW e2 2 2 e2 2
E= n (n arit ), cB = n E = a sin2 W = a
40 c2 r d 4c3 3 c3
dove e langolo fra a e la direzione di osservazione n, e e2 qe2 /40 . In media temporale corrisponde ad una

forza 2e2 x/3c 3
. Un dipolo elettrico p(t) irraggia una potenza W = 2p2 /3c3 40 .

Esercizio 152: Atomo di idrogeno


Calcolare quanto dovrebbe irraggiare.

bSoluzione: Un elettrone in un atomo ruota con accelerazione a = 2 rA , quindi in un giro irraggia


2rA 4 e2 v 3
E = W = ( ) Eatomo 3
v 3 rA c
dove = v/c 1/137. e fa 1017 giri al secondo. Questa formula stima bene la vita media dei livelli eccitati, ma
lo stato base ha vita media infinita.
Modello classico per la stabilita dellatomo di idrogeno: se invece di immaginare lelettrone come una carica
puntiforme lo si pensasse come 2 cariche messe ai punti opposti della traiettoria, la potenza irraggiata sarebbe
zero, in approssimazione di dipolo. Con n carice e/n verrebbe ridotta di un fattore n. Nel limite n
lelettrone viene spalmato lungo la sua traiettoria e non irraggia piu, anche se i singoli pezzi lo farebbero, per
via di interferenza distruttiva.
Siccome E  Eatomo se cadesse, spiraleggerebbe lentamente su orbite circolari: a ciascun istante
e2 me v 2 e2 e2
= me 2 r : U= =
r2 2 r 2r
Quindi
e2 2e2 e2 2
r = U = W = ( )
2r2 3c3 me r2
cioe
d(r3 ) e2
= 4re2 c : r3 (t) = r03 4re2 ct re = 2.8 1013 cm.
dt me c2
Questo effetto estato osservato nel caso analogo della gravita, discusso in seguito: due masse in rapida
rotazione una attorno allaltra irraggiano onde gravitazionali.

Polarizzazione Una carica in moto circolare corrisponde ad un dipolo elettrico rotante p (1, i, 0)ei . I
campi di dipolo elettrico in zona di radiazione (r  c) sono dati da
1 prit n
B= ; E = cB n
40 rc3
Quindi B (x + iy) n: se si osserva lungo lasse z, essendo (x + iy) z = i(x + iy) la polarizzazione e
circolare. Se invece n = x la polarizzazione e lineare. Infatti guardando dallaltro si vede una carica che gira,
guardando di taglio una carica che oscilla.

90
Capitolo 13. Irraggiamento 91

Esercizio 153: Scattering elettrone/nucleo


Un elettrone con velocita iniziale v  c urta frontalmente un nucleo di carica Ze. Calcolare la polarizzazione
della radiazione emessa, e lenergia totale irraggiata, assumendo e verificando che sia una piccola frazione
dellenergia cinetica.

bSoluzione: Secondo la formula generale E e polarizzato linearmente nel piano (n, a) ed ortogonale ad n, e
B gira attorno ad a. Laccelerazione e a = F/me = Zqe2 /40 r2 . Quindi, applicando la formula generale

dW qe2 a2 2 e2 A qe6 Z 2
= sin2 , W = a2 = 4 A=
d 16 2 0 c2 3 40 c3 r 96m2e (c0 )3

Lenergia irraggiata vale



8 me v05
Z Z
W Eirr v0
Eirr = W dt = 2 dr = ( )3
rmin v 45 c3 Z T0 c

Alla fine viene Z al denominatore in quanto se Z  1 lelettrone rimane lontano dal nucleo. Per fare il conto
si usa la conservazione approssimata dellenergia
s r
2Zq 2
v(r) = v02 = v02 .
40 me r r

Lintegrale vale

v05 16 v05
Z Z
dr dx
= =
3 15 3
p p
/v02 r4 v02 /r 1 x4 1 1/x

avendo usato la variabile di integrazione adimensionale x = r/rmin . A parte il fattore numerico 16/15 il risultato
2 2
segue in modo semplice dal fatto che lintegrale e dominato da r > /v0 , cosicche w v0 . Il fattore numerico e
calcolabile usando y = 1 1/x come variabile di integrazione.
Per fare il conto con scattering frontale abbiamo messo una forza repulsiva. In realta la forza e attrattiva e
lo scattering non e frontale. Ma il risultato qualitativo rimane lo stesso.

Esercizio 154: Scattering ee


Si puo stimare lenergia irraggiata mettendo Z = 1 nella risposta dellesercizio precedente?

bSoluzione: Nellesercizio precedente il nucleo era molto piu pesante dellelettrone e quindi rimaneva circa
fermo. A prima vista avere due particelle di massa uguale cambia solo qualche fattore di O(1) in quanto
irraggiano entrambe. Sbagliato. Ce una differenza qualitativa importante. Il dipolo elettrico totale dei due
elettroni e proporzionale al loro impulso
e
p = er 1 + er 2 = P
me

e quindi e costante. In approssimazione di dipolo non ce irraggiamento. Entrambi gli elettroni accelerano, ma
ce uninterferenza distruttiva fra i loro contributi. Siccome sono messi in posizioni diverse hanno diversi tempi
ritardati e la cancellazione non e totale: ce un irraggiamento da quadrupolo Qij = qe [3xi xj r2 ij ]
P

1 2 p2
 2
Q

W = + +
40 3 c3 60c5

per cui alla fine Eirr /T0 (v0 /c)5 invece che (v0 /c)3 .
92 Capitolo 13. Irraggiamento

Esercizio 155: Onde gravitazionali


Stimare la potenza irraggiata in onde gravitazionali

bSoluzione: Gli esercizi precedenti invitano ad una digressione sullirraggiamento di onde gravitazionali, dove
si ha un fenomeno analogo allirraggiamento con 1/0 G e q m:

P 2
 2
Q

W G 3 + 5 +
c c
Nel caso gravitazionale lirraggiamento da dipolo e sempre zero in quanto il momento e costante. (In linguaggio
profondo ma per ora incomprensibile il gravitone ha spin 2, mentre il fotone ha spin 1). Lirraggiamento da
quadrupolo puo essere stimato come
2
Wint c5
Wirr dove W0 = = 3.62 1052 Watt
W0 G
ha il significato fisico di potenza interna (non sferica) del sistema. Ad esempio due corpi di masse M
e Wint Q
in orbita a distanza R luno dallaltro sentono una forza F GM 2 /R2 ed hanno velocita data da M v 2 R/G:
quindi Wint F v v 5 /G = W0 (v/c)5 < W0 in quanto v < c (quando si raggiunge v c il sistema collassa in
un buco nero). La costante universale W0 ha quindi il significato fisico di massima potenza possibile.
Nel caso del sistema Terra/Sole, bisogna tenere conto che hanno masse diverse:  = MT /MS 106 . Questo
produce Wint = W0 (v/c)5 . Essendo v/c 104 si ha Wirr 2 (v/c)10 W0 Watt, meno di una lampadina. A
differenza dellanalogo elettromagnetico nellatomo di idrogeno non e un fenomeno preoccupante.

Esercizio 156: Scattering elettrone/fotone


bSoluzione: Se a = qe E/m si ottiene
W 40 c(2/3)(e2 /mc2 )2 hE 2 i 8 2
= = = r
S 0 c2 hEBi 3 e
re = e2 /mc2 = 2.82 1013 cm e il raggio classico dellelettrone. Vale solo se si puo trascurare il rinculo
dellelettrone (cioe raggi ).
Se illumino un atomo di idrogeno il protone non conta nulla (a energie maggiori di 13.6 eV, mentre a energie
minori conta solo lo stato legato).
Se illumino un atomo di elio ho 2 elettroni che possono irraggiare in fase. In generale Z 2 or Z.

Esercizio 157: Polarizzazione della CMB


bSoluzione: da fare

Esercizio 158: Unantenna


bSoluzione: Unantenna che contiene una corrente oscillante I = I0 eit (12|z|/`) contiene una carica =
R `/2
j = I/z = (z/|z|)2I0 eit /L e quindi produce un dipolo elettrico oscillante p = `/2 z dz = I0 `eit /2i.
Secondo la formula di Larmor un singolo dipolo elettrico p irraggia in direzione rispetto al dipolo
dW hp2 i sin2 I02 (k`)2
= W = Rrad Rrad = 5(k`)2 ohm
d 4c3 40 2 6c 40
Segnali TV hanno (10100) m, quindi se un albero si interpone fra lantenna ricevente e quella trasmittente
non e un problema serio.
Capitolo 13. Irraggiamento 93

Esercizio 159: Due antenne


Due antenne piccole rispetto a situate a distanza /4 irraggiano con dipoli uguali eccetto una differenza di
fase di 90 . Calcolare dW/d ed il momento irraggiato

bSoluzione:
Due dipoli irraggiano un campo elettrico
2
E = E 1 + E2 = E 1 (1 ei ) = + cos = (1 + cos )
2 4 2
Quindi
2 I02 L2
 
dW dW1 i 2 dW1 2
= |1 e | = 2(1 cos ) = sin 1 + sin( cos )
d d d 16c3 2
Cioe non e simmetrica. Viene quindi anche emesso un momento

dpz 1 dU 2
= (1 )
dt c dt 12
che per W 105 W vale Fz 104 N.

Esercizio 160: Dipolo magnetico


Pulsar ruotante

bSoluzione: Una pulsar ruota facendo girare anche il suo dipolo magnetico . Quindi irraggia W = 22 /3c3 =
2 4 2 /3c3 riducendo lenergia cinetica rotazionale U = I 2 /2 (I 25 M R2 ). Se e misurato si ricava

3 M R2 c3 2
2 = B 1015 Gauss
5 3 R3

per M M , R 10 km, T 10 s, T 1010 .


Capitolo 14

Relativita

Introducendo A = (, A) le equazioni E = A/c e B = A diventano


0

E 0
F = A A = x

Ey B z 0

Ez By Bx 0
Le equazioni di Maxwell sono
4
F = J J = (c, J )
c
Le trasformazioni di Lorentz dei campi sono
0 0

Ex0 = Ex Bx0 = Bx
0 0 0 0 0 0
F = F (x ) = i.e. Ey = (Ey Bz ) By0 = (By + Ez )
0

0 0 1 0

Ez0 = (Ez + By ) Bz0 = (Bz Ey )
0 0 0 1
L = 41 F F = (E 2 B 2 )/2 e  F F E B sono invarianti di Lorentz. Per unonda valgono zero.
(Sporco trucco: F E + iB fa rotazioni con angolo complesso, quindi F 2 e invariante).
Non useremo le seguenti formule. Quadri-corrente di una carica puntiforme q in moto X ( ) arbitrario:
Z
J = q d V (x X ( )).
 
1
Per moto rettilineo uniforme a velocita v si riduce allovvio J = q (x vt)(y)(z), come puo verificare
  v
1
trasformando J0 = q (x0 )(y 0 )(z 0 ).
0
Il vettore di Poynting fa parte del tensore simmetrico energia impulso T che trasforma come
0 0 0
T00 = 2 (T00 2T0x + Txx 2 ), Txx = 2 (Txx 2T0x + T00 2 ), T0x = 2 [T0x (1 + 2 ) (T00 + Txx )]
0 0 0
T0y = (T0y Txy ), Txy = (Txy T0y ), Tyy = Tyy
e lo stesso per y z.

Esercizio 161: Contrazione di Lorentz


Verso il 1900 si discuteva il seguente problema: assumendo che la materia sia tenuta assieme da forze elettro-
magnetiche, e sapendo come queste si trasfromano in diversi sistemi di riferimento determinare in che modo la
materia si ingrossa o rimpicciolisce se vista da un sistema in moto.

bSoluzione: Una volta capito, il problema diventa banale. Siccome lelettromagnetismo ed il resto della fisica
trasformano in modo ben definito sotto trasformazioni di Lorentz, la distanza fra due punti di un oggetto forma
un quadrivettore X , che si trasforma come un quadrivettore indipendentemente dalle forze complicate F
che lo tengono assieme. E.g. e ovvio che sotto rotazioni la distanza e un invariante: il resto non e molto piu
profondo.

94
Capitolo 14. Relativita 95

Esercizio 162: Che cosa e lelettromagnetismo


Mostrare che lelettromagnetismo e lunica teoria relativistica di un campo vettore.

bSoluzione: In meccanica classica (ma specialmente in meccanica quantistica) un modo conveniente di de-
scrivere una teoria consiste nello scriverne la Lagrangiana L (per quanto riguarda la presente discussione una
Lagrangiana e sostanzialmente lenergia cinetica). Se ad esempio la teoria ha una qualche simmetria puo non
essere ovvio vederla dalle equazioni del moto. che la Lagrangiana. Ci interessa il caso in cui la simmetria e
linvarianza di Lorentz: quindi la Lagrangiana deve essere uno scalare.
Nel caso una teoria relativistica di un campo scalare la naturale Lagrangiana e L = ( )2 , con il segno
fissato ad essere + in modo che lenergia cinetica sia positiva, L = +2 + .
Per un vettore A apparentemente si ha L = ( A )2 , ma questa teoria non ha senso: infatti per
qualunque segno o la componente A0 o le componenti A1,2,3 hanno energia cinetica con segno sbagliato. Lunica
teoria sensata e data da L = ( A A )2 . Questa forma speciale corrisponde ad una nuova simmetria di
gauge A A + , necessaria (specialmente a livello quantistico) per giustificare la speciale forma sensata.
La simmetria di gauge vincola altri termini addizionali:

Il fotone deve avere massa zero. Infatti il termine Lorentz-invariante m2 A2 /2 (m ha dimensioni lunghezza1 )
viene proibito. Questo sarebbe un termine di massa per il fotone. Infatti in sua presenza il quadri-vettore
donda che compare in eiKX soddisferebbe a K 2 = m2 , e quindi /c > m. Quantisticamente il quadri-
impulso e P = hK , per cui hm e una massa. Nel limite statico ( = 0) si avrebbe k = im, cioe la forza
di Coulomb sarebbe erm /r2 .

Laccoppiamento alla materia J A rispetta questa simmetria se J = 0: la carica elettrica deve essere
conservata.

Quando viene sviluppata anche una teoria della materia J diventa e linvarianza di gauge diventa una
simmetria locale di : U(1) nel caso dellelettromagnetismo, SU(2) per le interazioni elettro-deboli, SU(3) per
quelle forti, e Poincare per la gravita.

Esercizio 163: Forza fra 2 cariche bis


Due elettroni si muovono parallelamente lungo traiettorie rettilinee a distanza a con velocita costante v  c.
Calcolare la forza elettromagnetica

bSoluzione: Modo 1: Nel sistema in cui le cariche sono in quiete F0 = e2 /40 e quindi la relativita dice che
Fv = F0 /
Modo 2: Nel sistema di quiete esiste solo E. Trasformando i campi trovo che nel sistema in cui le cariche si
muovono Ey0 = Ey e Bz0 = Ey : la forza di Lorentz e

e2 0 e2 v2 F0
Fv = ( ev ev) = (1 2 ) =
40 4 40 c

Per calcolare il segno basta ricordare che fili con correnti uguali si attirano.
Negli acceleratori di particelle si riesce ad accelerare fasci di particelle cariche, perche a v c la forza
repulsiva di Coulomb e compensata da quella magnetica.

Esercizio 164: Scattering debole bis


Una carica q viaggia lungo lasse x con energia E e grande parametro dimpatto b verso una carica Q, ferma
nellorigine e di massa cos grande che rimane a riposo. Calcolare il piccolo angolo di deflessione e verificare che
limpulso acquistato da q e uguale ed opposto a quello acquistato da Q.

bSoluzione: La formula dp/dt = F = q(E + v B) e vera anche relativisticamente, dove p = mv e limpulso


relativistico.
96 Capitolo 14. Relativita

Per grande b la carica q viene deflessa di poco e si ha (b) ' pq /p  1 con


Z + Z +
qQ b dt qQ
pq = Q
dt qE (x = vt, y = b, 0, t) = =
40 [(vt)2 + b2 ]3/2 20 vb
Il calcolo e stato gia effettuato a pagina 13, mostrando che uno puo fare lintegrale col il teorema di Gauss.
Limpulso acquistato da Q si calcola in modo analogo, in termini del campo elettrico generato dalla carica
q in moto relativistico: partendo dal sistema S 0 dove q e ferma (x0 = (x vt)) le trasformazioni di Lorentz di
E dicono che E e innalzato da un fattore :
Z + Z +
qQ b dt
pQ = dt Q E q
(0, 0, 0, t) = = pq
40 [(vt)2 + b2 ]3/2

Esercizio 165: Carica in E e B ortogonali bis


Estendere lesercizio di pagina 52 al caso di moto relativistico.

bSoluzione: Avevamo visto che una carica in E e B ortogonali spiraleggia driftando a velocita costante,
indipendente dalla carica e dalla massa. Questo diventa ovvioriassorbendo il drift tramite una trasformazione di
Lorentz con velocita E/B. Nel nuovo sistema E 0 = 0 e B 0 = B 2 E 2 (se B > E) come segue immediatamente
dal fatto che E B = 0 ed E 2 B 2 sono invarianti di Lorentz. Se E > B si puo andare in un sistema dove
B 0 = 0 ed E 0 = E 2 B 2 tramite un boost di velocita c2 B/E. Fisicamente questo e dovuto al fatto che se il
campo elettrico e troppo grosso, E > B, il campo magnetico non riesce ad incurvare la traiettoria.

Esercizio 166: Filo in moto


Un filo rettilineo infinito disposto lungo lasse x ha sezione A e contiene n elettroni per unita di lunghezza in
moto con velocita v, e n protoni fermi. Il filo viene messo in modo con velocita lungo lasse x. Calcolare i
campi E e B.

bSoluzione: Nel sistema S dove il filo e fermo il vettore quadricorrente J = (, j) vale J = (0, i/A) dove
i = nev. Questo produce E = 0 e B = 0 i/2r.
1. Un primo modo di calcolare i campi nel sistema S 0 rispetto al quale il filo si muove con velocita . Uno
puo pasticciare nel tradurre le trasformazioni di Lorentz dei campi in coordinate cilindriche, per evitare
rogne e bene tenere in conto che il sistema ha simmetria cilindrica e che E 2 B 2 e E B sono invarianti.
Il risultato e
Er0 = B , B0 = B (14.1)

2. Un secondo modo consiste nel trasformare il quadrivettore J, ottenendo


0 = jx , jx0 = jx (14.2)
da cui e immediato riottenere la (14.1): jx0 e quindi B0 diventa volte piu grosso, e la densita di carica
lineare 0 = A0 = nev genera il campo elettrico Er0 = 0 /20 r calcolato a pagina 7.
3. Un terzo modo piuttosto rognoso, ma che consente di capire da dove salta fuori la carica, consiste nel
trasformare le singole particelle. Nel sistema S 0 i protoni hanno carica +e, velocita e densita n0+ = n,
perche la lunghezza si contrae. Nel sistema S 0 gli elettroni hanno carica e, velocita v 0 = ( + v)/(1 + v)
(formula di addizione delle velocita) e densita n0 = n(v 0 )/(v). Per calcolare n0 conviene considerare
il sistema S 00 rispetto al quale gli elettroni sono fermi e n00 = n/(v), con (v) = (1 v 2 )1/2 . Quindi la
densita di carica del filo nel sistema S 0 vale
(v 0 )
0 = + () + = ne()v()
(v)
(utilizzando + = = ne e (v 0 ) = (v)()(1 + v)) in accordo con la (14.2) e quindi con la (14.1).
Questo esercizio illustra che tutto e consistente, e che utilizzare le leggi di trasformazione dei campi e molto piu
rapido.
Capitolo 14. Relativita 97

Esercizio 167: Forza prodotta da filo in moto


Una particella di carica q e in quiete a distanza r dal filo dellesercizio precedente. Calcolare la forza che agisce
sulla carica.

bSoluzione:
1. Fr0 = qEr0 dove Er0 e stato calcolato in vari modi allesercizio precedente.
2. Alternativamente si puo calcolare la forza nel sistema S dove il filo e fermo e la carica in modo con velocita
lungo lasse x. La forza di Lorentz e diretta lungo r e vale Fr = qB . Trasformando la forza al
sistema S 0 si ottiene Fr0 = Fr in accordo con il risultato precedente.

Esercizio 168: Onda vista da sistema in moto


Unonda si muove lungo K = (1, nx , ny , 0) con B = (0, 0, Bz ) e E = B(ny , nx , 0). Come diventa se vista da
un sistema in moto lungo lasse x con velocita ?

bSoluzione: Siccome K e un quadri-vettore


ny 0 1 1
K 0 = ((1 nx ), (nx ), ny , 0) i.e. 0 = nx =
nx0 (nx /ny )
Il campo magnetico diventa
B 0 = (0, 0, Bz0 ) = Bz (1 ny )(0, 0, 1)
0
Per finire il campo elettrico deve diventare E 0 = B 0 (n0y , n0x , 0), dovendo essere E 0 = k B 0 .

Esercizio 169: Riflessione da specchio in moto


Unonda elettromagnetica di frequenza si muove in direzione n = (nx , ny , 0) con il campo elettrico polarizzato
lungo lasse z. Uno specchio e situato a x = 0 nel piano yz.
1) Calcolare direzione e campi elettromagnetici dellonda riflessa.
Lo specchio viene ora messo in moto con velocita costante v lungo lasse x.
2) Calcolare direzione, frequenza e campo elettrico dellonda elettromagnetica incidente rispetto al sistema
di riferimento in cui lo specchio e fermo.
3) Calcolare direzione, frequenza e campo elettrico dellonda riflessa rispetto al sistema in cui lo specchio e
in moto.

bSoluzione: Al bordo con una superficie riflettente E= = B = 0: quindi se mando unonda, londa riflessa
ha E= e B invertiti. Ricordo la riflessione da uno specchio fermo:
kxr = kxi , kzr = kzr , E r = E i
Poi E B e E 2 B 2 sono invarianti...

Esercizio 170: Aberrazione relativistica


Siccome i fotoni emessi da dietro fanno un viaggio piu lungo un cubo che viaggia a velocita viene visto
ruotato di un angolo 90 per 1 (e con velocita appartente > c). Leffetto geometrico banale e quello di
contrazione si sommano a dare questo. NON viene visto contratto (Einstein trascurava leffetto banale). Una
sfera rimane una sfera.

bSoluzione: Si tiene conto di questo effetto quando si osservano i getti emessi da un nucleo galattico attivo.
98 Capitolo 14. Relativita

Esercizio 171: 0 2
Dedurre la partita del 0 dalla misura della polarizzazione della luce emessa nel suo decadimento.

bSoluzione: Il 0 decade in due onde elettromagnetiche (fotoni ); per la conservazione dellimpulso le due
onde hanno eguali intensita se il 0 e a riposo. Possono pero avere diversa polarizzazione: chiamiamo langolo
fra i campi elettrici E 1 e E 2 delle due onde. Ha interesse calcolare in funzione di quanto valgono gli invarianti
di Lorentz: scalare e pseudo-scalare:
E 2 B 2 F F , E B  F F
dove E = E 1 + E 2 e B = B 1 + B 2 .
1 Se = 0 (fig. a sinistra)si ha E 1 = E 2 ; tenendo conto che i due hanno k opposti i campi magnetici sono
anti-paralleli: B 1 = B 2 . Quindi E 2 B 2 6= 0 e E B = 0.

E2
B2

0 E1 E2
0 E1
B1
B2 B1

2 Se = /2 (fig. a destra) i campi sono ortogonali: E 1 E 2 e B 1 B 2 . Questo produce E 2 B 2 = 0 e


E B 6= 0.
3 Per generico si ha E 1 = (1, 0), B 1 = (0, 1), E 2 = (cos , sin ), B 2 = (sin , cos ). Questo produce sia
E B sin che E 2 B 2 cos .
Sperimentalmente misurando le polarizzazioni dei due fotoni emessi in decadimenti 0 si trova che
esse sono ortogonali (cioe = /2 come nel caso 2) e che quindi il decadimento del 0 genera un campo
elettromagnetico che ha diverso da zero linvariante pseudo-scalare E B. Andando avanti si concluderebbe che
il 0 e uno pseudo-scalare accoppiato allelettromagnetismo tramite interazione Lagrangiana 0  F F
e che alcune interazioni fondamentali violano la parita (cioe un esperimento da risultati diversi dello stesso
esperimento costruito in modo speculare).

Esercizio 172: GZK


Un protone urta frontalmente un fotone di energia E = 1012 mp c2 , dove mp e la massa del protone.
1. Quale energia Ep deve avere il protone affinche il processo p sia possibile, se m = 1.3mp ?
2. La viene prodotta e successivamente decade in un protone p0 ed in un fotone 0 . Quali energie hanno
le particelle di decadimento, nel sistema di quiete della ?
3. Nel sistema di quiete della , p0 e 0 vengono emessi in direzione ortogonale alle velocita delle particelle
p e originarie. Quali energie Ep0 ed E 0 hanno le particelle di decadimento nel sistema iniziale?

bSoluzione: E un problema di soglia piu difficile del solito perche il sistema rilevante non e ne il CM ne il lab
(ma e il sistema in cui le stelle sono ferme).
1. Predendo il modulo quadro di Pp + P = P trovo
m2 m2p
2(Ep pp cos ) =
E
Il protone e ultra-relativistico: pp Ep . La minima Ep e richiesta quando cos = 1 (scontro frontale).
Quindi protoni con energia maggiore di Ep > (m2 m2p )/4E = 5.5 1011 GeV vengono assorbiti.
Capitolo 14. Relativita 99

2. Da P = Pp0 + P0 si ha P02 = (P Pp0 )2 cioe, nel sistema del CM in cui la e ferma 0 = m2 + m2P
0 0 0 0
2m EpCM /c2 , da cui EpCM /c2 = (m2 +m2p )/2m . Procedendo in modo simile, o usando ECM +EpCM =
2 0 2 2 2
m c si trova ECM = (m mp )c /2m .

3. Facendo una trasformazione di Lorentz E 0 = (ECM 0


+ vCM p0xCM ) = ECM
0 0
dove ECM sono le energie
calcolate nel CM al punto 2. Il fattore vale E /m c Ep /m c 10 . Qualitativamente Ep0
2 2 11

E0 m . Quindi nel processo (ogni volta che langolo di decadimento nel CM e diverso da 0) il fotone
ha acquistato energia a spese del protone.

Questo problema e attualmente anche uno dei problemi della cosmologia. Luniverso e riempito da radiazione
cosmica di fondo (fotoni con energia E 2.73 K) e da raggi cosmici (protoni). I protoni possono venir
distrutti tramite il processo p . Quindi in questo problema abbiamo mostrato che non dovrebbero esistere
raggi cosmici di energia superiore a 5 1010 GeV (numero ufficiale ottenuto da calcolo esatto nel 1966: Greisen-
Zatsepin-Kuzmin (GZK) cutoff), o piu precisamente, tenendo conto della densita di fotoni che possono
arrivare sulla terra solo se prodotti entro d = 1/n 10 Mpc dove n 400/ cm3 e p 1028 cm2 . (In
realta il decadimento dominante e N invece di p, ma questo rafforza il bound) Il problema e
che raggi cosmici (protoni?) sopra il GZK cutoff sono stati osservati, mentre non si osserva nessuna possibile
sorgente abbastanza vicina. Una possibile spiegazione e che intensi campi magnetici extra galattici facciano fare
giri bislacchi ai protoni, per cui non arrivano ne dalla stessa direzione ne allo stesso tempo dei fotoni emessi
dalla sorgente che non si riesce a vedere e che quindi puo essere morta nel frattempo. Altra possibile soluzione
e che la relativita sia sbagliata: nessuno la ha mai testata ad E/mc2 tanto alti. Altra possibilita e che siano
prodotti vicino alla Terra dal decadimento di particelle relitte di enorme massa.

Esercizio 173: Effetto Compton


Mandando raggi X su elettroni fermi nel 1923 Compton trovo 0 () = +0.024A(1cos ). Si mostri che questa
e la relazione cinematica per e e considerando il fotone come una particella di energia E = h = hc/.

bSoluzione: Scrivo P + Pe = P0 + Pe0 con

P = (E/c, E/c, 0, 0), Pe = (me c, 0, 0, 0), P0 = (E 0 , E 0 cos E 0 sin )/c

(mostrare che il numero incognite = equazioni +1, come dovrebbe essere). Siccome Pe0 non mi interessa ricavo
direttamente il risultato riscrivendo la conservazione del quandri-impulso come Pe0 = (P P0 + Pe ) e prendendo
il modulo quadro:
m2e = m2e + 0 + 0 + 2me (E E 0 ) + 2EE 0 (cos 1)

cioe
1 1 1 cos
0
= + , 0 = Compton (1 cos )
E E me c2
dove lultima equazione e stata ricavata usando lequazione quantistica (compatibile con la relativita!) Compton =
h/me c2 = 2.4 1012 m. E = h significa che non posso affievolire la luce sotto un certo limite.
Insieme alleffetto fotoelettrico questo processo mostro in modo diretto che la luce ha anche natura parti-
cellare, cosa per la prima volta proposta in modo troppo conservatore da Planck in un tentativo di spiegare lo
spettro di corpo nero.

Esercizio 174: Esperienza d Fizeau


bSoluzione: Usando la formula di addizione delle velocita

v 0 = (v + c/n)/(1 + v/nc) t = (Ln/c)(1 + v/nc)/(1 + vn/c) T0 [1 (v/c)(n 1/n)]

Quindi = 2 t = (2L/)(v/c)(n2 1).


100 Capitolo 14. Relativita

Esercizio 175: Iraggiamento da elettroni relativistici


Quale e la massima energia raggiungibile da un acceleratore di elettroni?

bSoluzione: Siccome W = dE/dt = dE 0 /dt0 e uno scalare di Lorentz, la generalizzazione relativistica e


2 2 
2 e2 dP dP 2 e2
 
dp 1 dE
W = = 2
3 m2 c3 d d 3 m2 c3 d c d

dove d = dt/ e P = (E/c, p) = m(c, v). Specializziamo questa formula generale alle due tecnologie possibili
di acceleratori di particelle: lineare e circolare.
In un acceleratore lineare, usando dE = v dp (che segue da 0 = d(E 2 /c2 p2 ) = 2(E dE/c2 p dp)) la
formula puo essere riscritta in modo uguale a quella non relativistica
2 2
2 e2 2 e2 2 e2 1 dE
 
dp dE W dE/dx
W = = = 1
3 m2 c3 dt 3 m2 c3 dx dE/dt 2 3
3 m c v dx mc2 /re

In un acceleratore si riescono a produrre campi elettrici tanto intensi da accelerare un elettrone da fermo a
relatvistico in 10 cm: cioe si raggiunge dE/dx = me c2 /` con ` 10cm. Quindi la frazione di energia persa per
irraggiamento e re /` 1013 : completamente trascurabile. Il problema e che per accelerare elettroni fino ad
energie mai raggiunte prima (E 106 me c2 ) serve una lunghezza 106 ` 100km, il che costa 1010 e.
In un acceleratore circolare domina il termine |dp/d | = = (v/R) mv = m 2 a, con a = v 2 /R.
Quindi W e dato dalla formula non relativistica maoltiplicata per 4 e

dE W 2e2 4 3
= =
dx v 3R2
Quindi il massimo raggiungibile accelerando elettroni in un acceleratore circolare di raggio R e con gradiente
dE/dx vale s s r
4 dE/dx|max R 5 R
max = 6 10
e2 /R2 re ` 5 km

avendo usato e2 /R2 = me c2 re /R2 . LEP con R di qualche km ha raggiunto 2 105 con una corrente di qualche mA.
La parte dellacceleratore che costa piu energia e il raffreddamento dei magneti superconduttori che producono
campi magnetici di 1 Tesla, necessari per far girare gli elettroni. Notare che il max non dipende dalla massa
della particella: per raggiungere energie elevate conviene usare particelle pesanti: Emax mc2 max . Nel 2007
nellex anello di LEP, LHC accelerera protoni fino a max 104 .

Esercizio 176: Miraggi


bSoluzione: Ricordo che in unita c = 1 k = n con n grosso in zone piu dense.
Quando la luce entra in zone poco dense rimbalza.
La fase dellonda K X e uno scalare e quindi K = (, k) = (1, n) e un quadrivettore. Se kz 0 londa
torna indietro. Se londa va storta cioe kx 6= 0 kz2 = (n)2 kx2 questo succede non per n = 0 ma per n = kx /
che e tipicamente di ordine uno per angoli di ordine uno.
Altro modo: vado nel sistema dove kx0 = 0. Sfruttando linvarianza di K 2 e di kz in tale sistema 02 = 2 kx2
i.e.
k0 k2 n2 2 kx
n02 = ( z0 )2 = 2 z 2 = 0
kx 1 2

Potrebbero piacerti anche